You are on page 1of 101

MICROBIOLOGY AND PARASITOLOGY QUESTIONS

1. A newborn presents with lethargy, purpura, chorioretinitis and pulmonary infiltrates.


Which of the following could be a cause of this syndrome?
(A) Amebiasis
(B) Candidiasis
(C) Cytomegalovirus
(D) Human papillomavirus
ANS: C Medical Microbiology by Jawetz.,23rd edition, page 445

2. Which one of the following is a subcellular structure that is found only in Gram
negative bacteria?
(A) endospores
(B) lipopolysaccharide
(C) mitochondria
(D) phosphatidylethanolamine
ANS: B Medical Microbiology by Jawetz., 23rd edition, page 28

3. Which one of the following statements best describes a function of the peptidoglycan
layer in bacteria?
(A) The ability of bacteria to survive changes in the osmolarity of their environment is
due to the physical properties of the peptidoglycan layer.
(B) The peptidoglycan layer contains all the enzymes responsible for the synthesis of
membrane phospholipids.
(C) The peptidoglycan layer protects bacteria from the effects of ultraviolet light.
(D) The peptidolgycan layer renders Gram negative bacteria resistant to detergents.
ANS: A Medical Microbiology by Jawetz., 23rd edition, page 22
4. Which one of the following structures is responsible for the passage of low molecular
weight solutes through the outer membrane of Gram negative bacteria?
(A) capsules
(B) endospores
(C) flagella
(D) porins
ANS: D Medical Microbiology by Jawetz., 23rd edition, page 27

5. Which one of the following is a component of the cytoplasmic membrane of Gram


positive bacteria?
(A) Braun’s lipoprotein (Murein lipoprotein)
(B) capsules
(C) lipopolysaccharide
(D) phosphatidylglycerol
ANS: D Medical Microbiology by Jawetz., 23rd edition,page 25

6. The inhibition of bacterial protein synthesis by the macrolide antibiotic


erythromycin is directly due to which one of the following mechanisms?
A. Covalent modification of the ribosomal S12 protein
B. Inhibition of the “charging” of transfer RNA (tRNA)
C. Inhibition of the formation of a stable 70S initiation complex
D. Inhibition of the translocation step
ANS: D Medical Microbiology by Jawetz., 23rd edition,page 186

7. Your patient, suffering from a bacterial infection, receives a penicillin injection


and almost immediately experiences respiratory distress and loses consciousness.
This reaction is MOST LIKELY mediated by
A. complement.
B. CD8+ cytolytic T cells.
C. macrophages.
D. mast cells.
ANS: D Medical Microbiology by Jawetz., 23rd edition,page 140
8. The complement component, C3b, plays an important role in the immune
response by functioning to increase
antibody recognition.
A. chemotaxis.
B. opsonization.
C. TCR signaling.
ANS: C Medical Microbiology by Jawetz., 23rd edition,page 137

9. Which one of these statements explains the origin of Rh antibodies in the blood
of an
Rh-negative mother?
A. They are naturally occurring in the blood of all Rh-negative persons.
B. They are received through mis-matched blood transfusions.
C. They are synthesized by the mother in response to Rh-positive cells from her first
child.
D. They are transferred from the Rh-positive father during intercourse.
ANS: C Medical Microbiology by Jawetz., 23rd edition,page 135

10. Which one of the following bacterial pathogens requires selective culture
conditions for reliable isolation from the stool, including media with antibiotics and
incubation at 42 C in increased CO2?
A) Campylobacter jejuni
B) Enterohemorrhagic Escherichia coli
C) Salmonella sp.
D) Shigella sp.
ANS: A, Campylobacter jejuni Medical Microbiology by Jawetz., 23rd edition,page 273

11. Which one of the following is considered facultatively intracellular for human
macrophages?
A) Enteroinvasive Escherichia coli
B) Enterotoxigenic Escherichia coli
C) Salmonella typhi
D) Shigella sp.
ANS: C, Salmonella typhi Medical Microbiology by Jawetz., 23rd edition,page 258

12. A 20-year-old female presents with severe pelvic pain, fever, nausea and
vomiting. She was on the second day of her menstrual cycle. A purulent cervical
discharge was detected upon physical examination. Oxidase positive, Gram-negative
diplococci were isolated on Thayer Martin agar from the endocervical swab. What is the
MOST LIKELY identity of this organism?
A. Bordetella pertussis
B. Chlamydia trachomatis
C. Haemophilus ducreyi
D. Neisseria gonorrhoeae
ANS: D, Neisseria gonorrhoeae Medical Microbiology by Jawetz., 23rd edition,page 295

13. A large outbreak of food poisoning occurred at a company picnic. Eighty people
reported explosive vomiting and diarrhea within 3 hours after eating. All of the victims
recovered within 24 hours without medical intervention. No known bacterial or viral
pathogens were isolated from the stool of affected individuals. A survey of the food
eaten at the picnic suggested that potato salad was the common source of the
poisoning. The individual who prepared the salad had a sty on her left eye. Which one
of the following is the MOST LIKELY cause of this outbreak?
A. Clostridium perfringens
B. Salmonella typhimurium
C. Staphylococcus aureus
D. Staphylococcus epidermidis
ANS: C, Staphylococcus aureus Medical Microbiology by Jawetz., 23rd edition,page
223

14. Gram-positive cocci and polymorphonuclear leukocytes are observed on Gram


stain of sputum from a 60-year old alcoholic with pneumonia. Culture of the sputum
reveals alpha-hemolytic, catalase-negative bacteria. Which one of the following
laboratory tests is the MOST APPROPRIATE next step in the definitive identification of
this organism?
A. bacitracin sensitivity test
B. beta-lactamase production test
C. ELEK test
D. bile solubility test

ANS: D, bile solubility test Medical Microbiology by Jawetz., 23rd edition,page 236

15. A 19-year old female is admitted to an outpatient clinic with suprapubic pain,
dysuria and an urgency to urinate frequently. She is otherwise healthy. A clean catch
midstream urine specimen was collected and sent to the laboratory. Numerous
neutrophils are detected in the urine sample. The bacteriology laboratory reports that
less than 102 Gram-positive cocci and greater than 105 Gram-negative bacilli were
recovered per milliliter of urine. Which one of the following is the most likely cause of
her infection?
A) Escherichia coli
B) Klebsiella pneumoniae
C) Pseudomonas aeruginosa
D) Staphylococcus aureus
ANS: A, Escherichia coli Medical Microbiology by Jawetz., 23rd edition,page 252

16. A 35-year old male park ranger in rural New Mexico arrives to an emergency
room with fever and signs and symptoms of septic shock. An enlarged lymph node is
discovered under his arm upon physical examination. Gram-negative coccobacilli that
demonstrate bi-polar staining were seen in both lymph node aspirate and spinal fluid
from the patient. Which one of the following is the most likely etiologic agent of this
patient’s illness?
A) Brucella abortus
B) Borrelia burgdorferi
C) Chlamydia trachomatis LGV
D) Yersinia pestis
ANS: D, Yersinia pestis Medical Microbiology by Jawetz., 23rd edition,page 292

17. A 40-year old male hog farmer complains of intermittent fever, night sweats and
headaches to his physician. The patient is hospitalized and blood and bone marrow
specimens are collected. The physician requests that the laboratory incubate the
cultures for at least 6 weeks. Nine days after admission, Gram negative coccobacilli that
require increased CO2 for growth are isolated from the blood and bone marrow
cultures. The patient is most likely to be suffering from which one of the following
diseases?
A) Brucellosis
B) Bubonic plague
C) Legionnaire’s disease
D) Rocky Mountain spotted fever
ANS: A, Brucellosis Medical Microbiology by Jawetz., 23rd edition,page 284

18. Which one of the following diseases is most likely to respond favorably to
treatment with a -lactam antibiotic?
A. Legionellosis
B. Mycoplasmosis
C. Syphilis
D. Trachoma
ANS: C, Syphilis Medical Microbiology by Jawetz., 23rd edition,page 334

19. A 20-year old woman comes to your clinic complaining of an irritating vaginal
discharge. Upon examination, you find that the cervix and vaginal walls appear normal.
The discharge is thin and milky, pH 5.5, and has a fishy odor when treated with
potassium hydroxide. You are unable to detect pseudohyphae, buds, or flagellates
upon microscopic examination. A Gram stain reveals numerous curved Gram-negative
rods, epithelial cells, and clue cells, but relatively few Gram-positive rods or WBCs.
Cervical and vaginal cultures are negative for STDs. What is your diagnosis?
A. Bacterial vaginosis
B. Cervicitis
C. Trichomoniasis
D. Vaginal candidiasis
ANS: A, Bacterial vaginosis Medical Microbiology by Jawetz., 23rd edition,page 316

20. A 75-year old man in a nursing home is treated with a broad spectrum antibiotic
for bed sores. Two days later he develops bloody diarrhea. A stool specimen from the
patient is positive in an ELISA test for a necrotizing cytotoxin, and his treatment is
switched to vancomycin. The agent MOST LIKELY to be responsible for this clinical
syndrome is
A. Bacteroides fragilis.
B. Campylobacter jejuni.
C. Clostridium difficile.
D. Escherichia coli O157:H7.
ANS: C, Clostridium difficile Medical Microbiology by Jawetz., 23rd edition,page 209

21. Which one of the following toxins is responsible for scalded skin syndrome?
A. Alpha hemolysin
B. Exfoliatin
C. Hyaluronidase
D. Staphylococcal enterotoxin
ANS: B, Exfoliatin Medical Microbiology by Jawetz., 23rd edition,page 225

22. A 50-year old cancer patient with severe neutropenia became septic two weeks
after being admitted to the hospital. An oxidase-positive, Gram-negative rod that did not
ferment glucose was isolated from the patient’s blood. Which one of the following
organisms is the most likely cause of the patient’s sepsis?
A. Enterobacter cloacae
B. Escherichia coli
C. Klebsiella pneumoniae
D. Pseudomonas aeruginosa
ANS: D, Pseudomonas aeruginosa Medical Microbiology by Jawetz., 23rd edition,page
262

23. Blood culture from a 12-year old with acute osteomyelitis yields Gram-positive
cocci that are b hemolytic and catalase positive. Which one of the following laboratory
tests is the MOST APPROPRIATE next step in the definitive identification of this
organism?
A. Coagulase test
B. Optochin sensitivity test
C. Slide agglutination test for capsular antigen type
D. Test for capacity to grow in bile esculin
ANS: A, Coagulase test Medical Microbiology by Jawetz., 23rd edition,page 225

24. Activated macrophages are the effector cells in the expression of a protective,
cell-mediated immune response against
A. Corynebacterium diphtheriae.
B. Haemophilus influenzae type b.
C. Listeria monocytogenes.
D. Streptococcus pneumoniae.
ANS: C, Listeria monocytogenes Medical Microbiology by Jawetz., 23rd edition,page
152

25. A patient recovering from a crushing leg injury suffered during an a motorcycle
accident develops a temperature of 100o F, a rapid pulse, and extreme pain at the site
of his the wound two days after the accident. The skin overlying the patient’s sutured
wound is brownish- white, shiny, and studded with vesicles. Gram stain of the watery
exudate from the wound reveals Gram-positive rods, Gram-negative rods, and Gram-
positive cocci. The Gram-positive rods are MOST LIKELY
A. Bacillus cereus.
B. Bacteroides fragilis.
C. Clostridium perfringens.
D. Corynebacterium ulcerans.
ANS: C, Clostridium perfringens Medical Microbiology by Jawetz., 23rd edition,page
208

26. A 13-year old and her 10-year old brother are seen at an emergency room with
complaints of double vision, difficulty swallowing, and progressive muscular weakness.
A detailed history reveals that both children attended a church picnic three days earlier
but neither child has gastroenteritis. The MOST LIKELY presumptive diagnosis is
A) botulism.
B) Chinese rice syndrome.
C) Guillan-Barré syndrome.
D) salmonellosis.
ANS: A, botulism Medical Microbiology by Jawetz., 23rd edition,page 206

27. Epidemics of bacterial meningitis in young, previously healthy adults are MOST
LIKELY caused by
A. Escherichia coli K-1.
B. Haemophilus influenzae type b.
C. Listeria monocytogenes.
D. Neisseria meningitidis group A.
ANS: D, Neisseria meningitidis group A. Medical Microbiology by Jawetz., 23rd
edition,page 303

28. A 55-year old woman develops endocarditis following oral surgery. Gram-
positive, alpha-hemolytic, catalase-negative cocci are isolated from a blood culture.
The MOST LIKELY agent is
A. Enterococcus faecalis (group D streptococci).
B. Staphylococcus aureus.
C. Streptococcus pyogenes (group A streptococci).
D. Viridans streptococci.
ANS: D, Viridans streptococci. Medical Microbiology by Jawetz., 23rd edition,page 236

29. A 2-month old infant is admitted to the hospital with fever, lymphocytosis, and
bouts of violent coughing that often end in vomiting. The infant’s mother reveals that
the child has not as yet received any immunizations. The attending physician swabs
the infant’s nasopharynx and requests that the swab be plated on routine media as well
as a special medium (Bordet-Gengou) that contains blood and glycerol. No organisms
are found on blood or chocolate agar, but small Gram-negative rods are isolated on
Bordet-Gengou agar. The MOST LIKELY cause of the infant’s illness is
A. Bordetella pertussis.
B. Chlamydia pneumoniae.
C. Moraxella catarrhalis.
D. Mycoplasma pneumoniae.
ANS: A, Bordetella pertussis. Medical Microbiology by Jawetz., 23rd edition,page 282

30. Which one of the following viruses establishes a chronic persistent infection in
humans?
A. Hepatitis A virus
B. Hepatitis C virus
C. Herpesvirus
D. Poliovirus
ANSWER: B, Hepatitis C virus Medical Microbiology by Jawetz., 23rd edition,page 475-
476

31. Which one of the following parasitic infections is MOST LIKELY to produce life-
threatening anemia in children?
A. Amoebic dysentery
B. Babesiosis
C. Hookworm disease
D. Malaria
ANSWER: D Malaria Medical Microbiology by Jawetz., 23rd edition,page 677

32. Which one of the following viruses is transmitted via the gastrointestinal route?
A. Delta-associated agent
B. Hepatitis A
C. Hepatitis B
D. Hepatitis C
ANSWER: B, hepatitis A. Medical Microbiology by Jawetz., 23rd edition,page 474

33.A 33-year-old pregnant woman delivered a baby boy via a normal vaginal delivery.
Neither she nor the father has a history of genital or oral herpes. Although she had
chickenpox as a child, she was exposed to a child with chickenpox about 10 days
before delivery. There was no evidence of vesicular skin lesions on either the mother
or the child at the time of delivery. Three days after birth, the baby now develops
vesicular skin lesions on his back, accompanied by a fever. Which one of the following
viruses is MOST LIKELY to be the cause of the baby’s infection?
A. Cytomegalovirus
B. Herpes simplex virus type 1
C. Herpes simplex virus type 2
D. Human herpesvirus 6
ANSWER: C, Herpes simplex virus type 2 Medical Microbiology by Jawetz., 23rd
edition,page 434

34. Which one of the following viruses is MOST LIKELY to infect and cause rapid cell
lysis in a fibroblast cell line culture?
A. Hepatitis B virus
B. Herpes simplex virus
C. Human papilloma virus
D. Norwalk virus
Answer: B, Herpes simplex virus Medical Microbiology by Jawetz., 23rd edition,page
437

35. Six months after returning from travelling in North Africa and the Middle East, a
graduate student experiences two episodes of intense headache, shaking chills, and
high fever, followed by sweating and exhaustion. The episodes occurred 48 hours apart
and each lasted about 6 hours. He appears at your clinic the day after the second
episode. Laboratory test results reveal low hematocrit (35%), slight hypoglycemia, and
an intraerythrocytic parasite on Giemsa-stained blood films. What is your diagnosis?
A. Babesiosis
B. Falciparum malaria
C. Ovale malaria
D. Vivax malaria
ANSWER: D, Vivax malaria Medical Microbiology by Jawetz., 23rd edition,page 678

36. Examination by dark-field microscopy of scrapings from a genital ulcer of 19-year


old female reveals mobile, spiral-shaped organisms against a black background. The
MOST LIKELY diagnosis is
A) chancroid.
B) Chlamydia trachomatis infection.
C) gonorrhea.
D) syphilis.
ANS: D, syphilis Medical Microbiology by Jawetz., 23rd edition,page 333
37. To accurately determine the cause of repeated bouts of otitis media that have
begun to affect a toddler’s hearing, inner ear fluid is obtained by careful puncture of her
tympanic membrane with a sterile needle. Gram stain of the aspirate reveals short
Gram-negative rods and polymorphonuclear leukocytes. No organisms are isolated on
blood agar but colonies are observed on chocolate agar. The MOST LIKELY cause of
this infection is
A. Haemophilus influenzae non typable.
B. Haemophilus influenzae type b.
C. Klebsiella pneumoniae.
D. Streptococcus pneumoniae.
ANS: A, Haemophilus influenzae non typable. Medical Microbiology by Jawetz., 23rd
edition,page 279-280

38. Gram-positive cocci are observed on a Gram stain of purulent discharge from a skin
lesion of a 7-year-old. Culture of the exudate reveals beta-hemolytic, catalase-positive
organisms. Which one of the following laboratory tests is the MOST APPROPRIATE
next step in the definitive identification of this organism?
A) Bacitracin sensitivity test
B) Bile solubility test
C) Coagulase test
D) Optochin sensitivity test
ANS: C Medical Microbiology by Jawetz., 23rd edition,page 227

39. Which one of the following descriptions is correct about cholera gravis?
A) Associated with the formation of colonic ulcers
B) Death may occur due to dehydration, acidosis, and shock
C) Most common clinical manifestation of Vibrio cholerae infection
D) Occurs in response to plasmid-borne heat stable (ST) toxin
ANS: B Medical Microbiology by Jawetz., 23rd edition,page 270-271

40. Which one of the following species or serovars of Chlamydia can be transmitted as
a zoonotic infection?
A) Chlamydia pneumoniae
B) Chlamydia psittaci
C) Chlamydia trachomatis serovar C
D) Chlamydia trachomatis serovar D
ANS: B Medical Microbiology by Jawetz., 23rd edition,page 365

41. Gram stain of synovial fluid from the knee of a 20-year-old woman with arthritis
reveals Gram-negative cocci. To isolate the MOST LIKELY responsible agent, the
specimen should be plated on
A) Bordet-Gengou agar.
B) Chocolate agar.
C) Eosin Methylene Blue (EMB) agar.
D) MacConkey agar.
ANS: B Medical Microbiology by Jawetz., 23rd edition,page 709

42. The formation of colonic ulcers and absesses during Shigella dysentery is due to
A) ability of bacteria to survive stomach acidity because of the production of urease.
B) bacterial survival within macrophages.
C) invasion of enterocytes and cell-to-cell spread of the bacteria.
D) production of a superantigen by the bacillus.
ANS: C Medical Microbiology by Jawetz., 23rd edition,page 255-256

43. An 8-year-old boy living in a wooded area of Virginia suddenly develops fever,
headache, and myalgia. On physical exam, a rash is noted on his wrists and ankles.
Within hours, the rash spreads to his trunk. Numerous arthropod bites are observed on
the child’s extremities. No organisms are isolated from 3 sequential blood cultures. The
child is treated with tetracycline and recovers. Serological studies on acute and
convalescent serum samples from the child are MOST LIKELY to reveal that the
etiologic agent was
A) Borrelia recurrentis.
B) Coxiella burnetti.
C) Franciscella tularensis.
D) Rickettsia rickettsii.
ANS: D Medical Microbiology by Jawetz., 23rd edition,page 352

44. Which one of the following organisms is the MOST COMMON cause of bacterial
traveler’s diarrhea?
A) Campylobacter jejuni
B) enterotoxigenic Escherichia coli
C) Salmonella cholerasuis
D) Shigella sonnei

ANS: B Medical Microbiology by Jawetz., 23rd edition,page 252

45. Dissemination of Salmonella typhi from the intestine to extraintestinal sites requires
survival of this pathogen in which one of the following host cells?
A) Colonic enterocytes
B) Gastric epithelial cells
C) M cells
D) Macrophages

ANS: D Medical Microbiology by Jawetz., 23rd edition,page 257-258


46. Gram stain of cerebrospinal fluid from a 3-year-old child with fever, petechiae, stiff
neck and positive Kernig and Brudzinski’s signs reveals PMNs and a few Gram-
negative cocci. The MOST LIKELY agent is
A) Escherichia coli.
B) Haemophilus influenzae type b.
C) Neisseria meningitidis.
D) Streptococcus agalactiae (group B streptococci ).

ANS: C Medical Microbiology by Jawetz., 23rd edition,page 302


47. Listeria monocytogenes infection of an adult is MOST LIKELY to be acquired by
A) ingestion of contaminated food.
B) inhalation of contaminated aerosals.
C) inoculation with a contaminated needle.
D) person-to-person contact.
ANS: A Medical Microbiology by Jawetz., 23rd edition,page 217

48. What is the mechanism of action of the antifungal drug fluconazole?


A. Binds to ergosterol resulting in membrane disruption
B. Inhibits ergosterol synthesis
C. Inhibits protein synthesis
D. Inhibits RNA synthesis
ANS: B Medical Microbiology by Jawetz., 23rd edition,page 652-653

49. A 10-year-old with a dog complains of a “bump” on the back of his head. He
says its redness and itching have increased over the last three weeks. The lesion is
several centimeters in diameter, with hair involvement and scaling. Examination of
several of his hairs in a KOH preparation reveals hyphae in and around the hair shafts.
What is your diagnosis?
A. Actinomycosis
B. Candidiasis
C. Pityriasis versicolor
D. Tinea capitis
ANSWER: D, Tinea capitis Medical Microbiology by Jawetz., 23rd edition,page 632

50. You are assigned to a hospital in Phoenix, Arizona. Several patients have been
admitted in the past week with cough, chest pain, fever, and pneumonitis that has not
responded to antibacterial therapy. You discover that most of the patients are new
arrivals to Arizona who had been working outdoors during the dusty wind storms that
had occurred over the past few weeks. The causative organism is observed in a KOH
preparation of sputum. What is your presemptive diagnosis?
A. Blastomycosis
B. Coccidioidomycosis
C. Histoplasmosis
D. Psittacosis
ANSWER: B, Coccidioidomycosis Medical Microbiology by Jawetz., 23rd edition,page
637-638

51. A 65-year-old male develops endocarditis following urinary catheterization. Gram-


positive, catalase-negative cocci are isolated from a blood culture. The MOST LIKELY
agent is
A) Enterococcus faecalis (group D streptococci).
B) Staphylococcus aureus.
C) Staphylococcus epidermidis.
D) Streptococcus pyogenes (group A streptococci).
ANS: A Medical Microbiology by Jawetz., 23rd edition,page 235

52. A 10-year-old child of an instructor at the U.S. Air Force Academy develops marked
swelling in her right inguinal lymph node with fever of 104.1EF. An aspirate from the
painful, swollen node reveals Gram-negative rods. A direct immunofluorescent assay on
the same aspirate provides a definitive diagnosis. The child is treated with streptomycin,
and the aspirate is sent for culture. The culture yields white colonies on MacConkey
agar grown at room temperature. The MOST LIKELY agent is
A) Brucella canis.
B) Enterohemorrhagic Escherichia coli.
C) Francisella tularensis.
D) Yersinia pestis.
ANS: D Medical Microbiology by Jawetz., 23rd edition,page 291

53. A 22-year-old develops an abcess in his peritoneum following rupture of his


appendix. Gram stain of the exudate from his foul-smelling abcess reveals numerous
polymorphonuclear leukocytes, Gram-positive cocci, Gram-positive rods, and Gram-
negative rods. Aerobic culture of the exudate at 37o C on blood and MacConkey agar
plates yields only enterococci (group D streptococci). The Gram-negative rods are
MOST LIKELY
A) Bacteroides fragilis.
B) Escherichia coli.
C) Pseudomonas aeruginosa.
D) Shigella dysenteriae.
ANS: A Medical Microbiology by Jawetz., 23rd edition,page 309

54. Several people become ill within two hours after leaving a summer reunion picnic.
The majority of people complain of vomiting and diarrhea without fever, and recovered
within 24 hours after the onset of symptoms. The contaminated food is determined to be
a bowl of coleslaw prepared by someone with a stye on her left eyelid. Which one of the
following is the MOST LIKELY source of the food poisoning?
A) Clostridium botulinum
B) Clostridium perfringens
C) Staphylococcus aureus
D) Shigella sonnei
ANS: C Medical Microbiology by Jawetz., 23rd edition,page 226

55. A 35-year-old female presents with fever, headache and a non-productive cough.
The patient is prescribed penicillin and sent home. Two weeks later, she returns with
similar symptoms. A Gram stain and routine culture of the patient’s sputum reveal only
normal mouth flora. Sputum is then cultured on specialized medium containing sterols,
and, after two days, tiny colonies were visible under a microscope. Which one of the
following organisms does the physician suspect is the cause of the patient’s
pneumonia?
A) Corynebacterium diphtheriae
B) Haemophilus influenzae type b
C) Legionella pneumophila
D) Mycoplasma pneumoniae
ANS: D Medical Microbiology by Jawetz., 23rd edition,page 345

56. A 70-year-old male, who is a chain-smoker, complains of fever, diarrhea, cough and
chest pain. Evidence of pneumonia is seen by chest X-ray. Due to the inability of the
patient to produce a sputum specimen, a transtrachial aspirate is obtained. No
respiratory bacterial pathogen is isolated from aspirated fluid cultured on blood agar
plates. However, poorly staining Gram-negative rods are isolated on buffered charcoal-
yeast extract (BCYE) agar after three days of incubation. The physician had already
begun treatment with erythromycin due to the detection of bacilli in the aspirated fluid
using a fluorescently-labeled monoclonal antibody specific for the causative agent. The
MOST LIKELY pathogen responsible for this pneumonia is
A) Haemophilus influenzae.
B) Legionella pneumophila.
C) Mycoplasma pneumoniae.
D) Neisseria meningitides.
ANS: B, Legionella pneumophila Medical Microbiology by Jawetz., 23rd edition,page
313

57. Immunopathologic delayed-type hypersensitivity responses to eggs in tissue,


resulting in granulomas, obstruction and fibrosis is characteristic of
A. acute trichinosis.
B. . schistosomiasis..
C. filariasis.
D. onchocerciasis.
Answer: B Medical Microbiology by Jawetz., 23rd edition,page 141-142

58. Which one of the following fungi exhibits thermal dimorphism?


A. Aspergillus fumigatus
B. Candida tropicalis.
C. Cryptococcus neoformans
D. Histoplasma capsulatum
Answer: D Medical Microbiology by Jawetz., 23rd edition,page 640

59. An otherwise healthy 35-year-old female florist presented to her primary care
physician complaining of a persistent lesion on the third digit of her left hand. This had
been present for several weeks and showed no sign of resolving. Upon examination,
the patient was found to have a mild fever (1000F), the aforementioned lesion and two
nodules on her left forearm. A biopsy of the lesion was performed, and the pathology
report indicated the presence of numerous polymorphonuclear leukocytes and a rare
elongated budding yeast. What is the MOST LIKELY etiologic agent?
A. Blastomyces dermatitidis
B. Sporothrix schenckii
C. Histoplasma capsulatum
D. Nocardia asteroides
Answer: B Medical Microbiology by Jawetz., 23rd edition,page 632-633

60. Mode of transmission of Trichomonas vaginalis is:


Ingestion of infested beef
Skin contact with larvae
Mosquito bites
Sexual
ANS:D Medical Microbiology by Jawetz., 23rd edition,page 663-664

61. Mode of transmission of Onchocerca volvulus is:


Exposure to infested water
Louse bites
Skin contact with larvae
Blackfly bites
ANS: D Medical Microbiology by Jawetz., 23rd edition,page 690

62. Mode of transmission of Leishmania mexicana is:


A. Sexual
B. Mosquito bites
C. Fecal-oral
D. Sandfly bites
ANS: D Medical Microbiology by Jawetz., 23rd edition,page 665

63. Mode of transmission of Enterobius vermicularis is:


A. Sexual
B. Blackfly bites
C. Louse bites
D. Fecal-oral
Ans. D Medical Microbiology by Jawetz., 23rd edition,page 689

64. Oxygen requirements of Neisseria gonorrhoeae is:


A. Aerobic
B. Facultative anaerobe
C. Microaerophilic
D. Obligate anaerobe
Ans. A Medical Microbiology by Jawetz., 23rd edition,page 295

65. Mode of transmission of Borrelia burgdorferi is:


A. Direct contact with animals
B. Conidia inhalation
C. Sexual
D. Tick bites
Ans. D Medical Microbiology by Jawetz., 23rd edition,page 336

66. Therapy of Vibrio cholerae is:


A. TMP-SMX
B. ORS
C. Ceftriaxone
D. Erythromycin
Ans. B Medical Microbiology by Jawetz., 23rd edition,page 271

67. Family of Measles virus is:


A. Paramyxoviridae
B. Rhabdoviridae
C. Orthomyxoviridae
D. Flaviviridae
Ans. A Medical Microbiology by Jawetz., 23rd edition,page 550

68. Family of Hepatitis A virus is:


A. Papovaviridae
B. Hepadnaviridae
C. Parvoviridae
D. Picornaviridae
Ans. D Medical Microbiology by Jawetz., 23rd edition,page 466

69. Which of the following statements refers to Toxoplasma gondii:


A. Sandfly gut: promastigote -> amastigote
B. Three major serogroups. After-infection immunity only for the serougroup of the
strain involved.
C. Primary infection is usually asymptomatic. Congenital infection results in
cytomegalic inclusion disease.
D. ELISA helps distinguish from dispar - non-pathogenic.
Ans. C Medical Microbiology by Jawetz., 23rd edition,page 685

70. Mode of transmission of Leishmania donovani is:


A. Sandfly bites
B. Sexual
C. Exposure to infested water
D. Spore ingestion
Ans. A Medical Microbiology by Jawetz., 23rd edition,page 665

71. Phenotypic characteristic of Schistosoma hematobium is:


A. Eggs have a small lateral spine
B. Eggs have a terminal spine
C. Non-enveloped
D. Four suckers and circle of hooks on scolex
Ans. B Medical Microbiology by Jawetz., 23rd edition,page 695

72. Therapy of Taenia solium is:


A. mebendazole
B. TMP-SMX
C. Praziquantel
D. Suramin
Ans. C Medical Microbiology by Jawetz., 23rd edition,page 691

73. Phenotypic characteristic of Streptococcus pyogenes is:


A. Alpha-hemolytic
B. Oxidase "+"
C. H2S "-"
D. Beta-hemolytic
Ans. D Medical Microbiology by Jawetz., 23rd edition,page 235

74. Mode of transmission of Rickettsia prowazekii is:


A. Droplet nuclei
B. Direct contact with animals
C. Louse bites
D. Wound infection
Ans. C Medical Microbiology by Jawetz., 23rd edition,page 350

75. Non-toxin virulence factor of Escherichia coli is:


A. Opacity associated protein (opa) - promotes adhesion to mucosal surfaces
B. Fimbria - adhesion
C. Fibronectin receptor
D. HSP-60 - possible "mimicry" inducing autoimmune response
Ans. A Medical Microbiology by Jawetz., 23rd edition,page 252

76. Phenotypic characteristic of Gonococcus is:


A. Fastidious
B. Mycolic acid in cell wall
C. Catalase "-"
D. Non-lactose fermenter
Ans. A Medical Microbiology by Jawetz., 23rd edition,page 295

77. Morbidity of Blastomyces dermatitidis is:


A. Urinary tract infections
B. Whooping cough
C. Disseminated granulomas
D. Bronchopneumonia
Ans. D Medical Microbiology by Jawetz., 23rd edition,page 643

78. Staphylococcal food poisoning is the result of


(A) a heat labile enterotoxin.
(B) ingestion of a pre-formed enterotoxin produced by Staphylococcus aureus during
bacterial multiplication in contaminated food.
(C) ingestion and multiplication of coagulase-negative Staphylococcus sp.
(D) multiplication of Staphylococcus aureus in the small intestine.
ANS: B Medical Microbiology by Jawetz., 23rd edition,page 225
79. A 19-year-old soldier develops fever, carditis, and migratory polyarthritis. While
taking her history, the patient remembers that she had a "sore throat" three weeks
previously. A throat culture of the patient yields normal flora only. The laboratory result
MOST CONSISTENT with these findings is that the patient has high titers of antibody
to:
(A) meningococcal lipooligosaccharide.
(B) pneumococcal IgA protease.
(C) pneumococcal polysaccharide.
(D) streptolysin O.
ANS: D Medical Microbiology by Jawetz., 23rd edition,page 234

80. A 65-year-old man with a colonic tumor undergoes a bowel resection. Three days
later, the patient develops a fever, and the drainage from the surgical site is foul
smelling. The skin near the site of the drainage is markedly erythematous. Gram stain of
the drainage reveals Gram-positive cocci and Gram-negative rods. Aerobic culture of
the drainage yields streptococci. The Gram-negative rod seen on Gram stain was most
likely:
(A) Bacteroides fragilis
(B) Clostridium perfringens
(C) Escherichia coli
(D) Pseudomonas aeruginosa
ANS: A Medical Microbiology by Jawetz., 23rd edition,page 306

81. A 50-year-old rancher scrapes his hand with clippers while shearing sheep. Two
days later he notices an ulcer surrounded by vesicles at the site of the wound. Eight
days after the injury, the wound develops a black necrotic center (eschar) surrounded
by edematous fluid. The rancher experiences mild systemic symptoms. The most likely
cause of the infection is:
(A) Bacillus anthracis.
(B) Clostridium tetani.
(C) Neisseria meningitidis.
(D) Streptococcus pyogenes (group A streptococci).
ANS: A Medical Microbiology by Jawetz., 23rd edition,page 202-203

82. A 19-year-old college sophomore dies 24 hours after the development of fever,
petechiae, purpura, and disseminated intravascular coagulation. Culture of
cerebrospinal fluid and blood obtained from the patient on admission to the hospital
MOST LIKELY yielded:
(A) Gram-negative, lactose fermenting rods.
(B) Gram-negative, oxidase positive diplococci.
(C) Gram-positive, catalase negative cocci.
(D) Gram-positive, catalase positive cocci.
ANS: B Medical Microbiology by Jawetz., 23rd edition,page 302

83. The blood culture of a patient with a presumptive diagnosis of endocarditis yields
non-hemolytic colonies of Gram-positive cocci. The isolate is catalase negative, can
grow in 6.5% NaCl, and is resistant to penicillin. The isolate is MOST LIKELY:
(A) Enterococci.
(B) Streptococcus agalactiae (group B streptococci).
(C) Streptococcus bovis.
(D) Streptococcus mutans (Viridans streptoccoci).
ANS: A Medical Microbiology by Jawetz., 23rd edition,page 233

84. A 25-year-old, previously healthy woman develops fever within 24 hours of delivery
of an infant born 6 weeks prematurely. Blood culture of the mother reveals Gram-
positive cocci that are catalase negative, beta-hemolytic on blood agar, and resistant to
bacitracin. The MOST LIKELY cause of the bacteremia is:
(A) Enterococci.
(B) Staphylococcus aureus.
(C) Streptococcus agalactiae (group B streptococci).
(D) Streptococcus pneumoniae.
ANS: C Medical Microbiology by Jawetz., 23rd edition,page 233

85. A 24-year-old college student comes to the emergency room with low blood
pressure. He reports that he had a recent cold, with fever and sore throat. On physical
exam, he has cervical lymphadenopathy, and an enlarged tender spleen. Which virus
infection is the MOST CONSISTENT with his clinical picture?
A) Adenovirus type 12
B) Epstein-Barr virus
C) Herpes simplex virus type 1
D) Rhinovirus
Answer: B Medical Microbiology by Jawetz., 23rd edition,page 447

86. An HIV-infected homosexual man presents with Kaposi’s sarcoma. Which viral
genome is MOST LIKELY to be present in his tumor biopsy?
A. Herpes simplex virus tupe 1
B. Human cytomegalvirus
C. Human herpesvirus type 6
D. Human herpesvirus type 8
Answer: D Medical Microbiology by Jawetz., 23rd edition,page 450

87. Which one of the following viral infections is not effectively prevented by
vaccination?
A. Hepatitis B virus
B. Poliovirus
C. Rhinovirus
D. Rubella virus
Answer: C Medical Microbiology by Jawetz., 23rd edition,page 543

88. Prions cause spongiform encephalopathies including the human disease


A. AIDS dementia complex.
B. Creutzfeldt-Jacob disease.
C. scrapie.
D. subacute sclerosing panencephalopathy (SSPE).
Answer: B Medical Microbiology by Jawetz., 23rd edition,page 584

89. Congenital infection by which one of the following viruses can result in birth defects?
A. Adenovirus
B. Cytomegalovirus
C. Hepatitis B virus
D. Human immunodeficiency virus
Answer: B Medical Microbiology by Jawetz., 23rd edition,page 443

90. Viral carditis of adolescents and adults is frequently caused by


A. Coxsackieviruses.
B. Orthomyxoviruses.
C. Pseudorabiesvirus.
D. Reoviruses.
Answer: A Medical Microbiology by Jawetz., 23rd edition,page 495

91. Several newborns housed in a neonatal intensive care unit have developed
nosocomial pneumonia. Most are wheezing and some have required care such as
mechanical ventilation, increased doses of oxygen, antibiotic therapy, or monitoring for
apnea. A simple, rapid diagnostic test reveals a common viral antigen in nasal wash
specimens from each of the patients. The virus MOST LIKELY to be responsible is
A. Coronavirus
B. Echovirus B2
C. Respiratory syncytial virus
D. Rubeola virus (measles)
Answer: C Medical Microbiology by Jawetz., 23rd edition,page 558-559

92. A baby is born with microphthalmia, cataracts, and a holosystolic/holodiastolic heart


murmer. Infection by what virus is the MOST LIKELY cause?
A. Cytomegalovirus
B. Enterovirus
C. Human immunodeficiency virus
D. Rubella virus
Answer: D Medical Microbiology by Jawetz., 23rd edition,page 567

93. A 7 year-old newly adopted Russian child who has been in the United States 10
days presents with fever, malaise, sore throat, and a grey membrane on the tonsils,
uvula, and soft palate. The child’s immunization history is unknown. The physician
obtains a swab of the child’s throat which provokes bleeding of the membrane. The
swab is sent to the laboratory for routine throat culture. Two days later the laboratory
reports only the presence of normal flora in the specimen. The MOST LIKELY organism
responsible for this child’s illness is:
(A) Bordetella pertussis.
(B) Corynebacterium diphtheriae.
(C) Haemophilus influenzae type b.
(D) Listeria monocytogenes.
Answer: B Medical Microbiology by Jawetz., 23rd edition,page 214

94. Soft cheese imported from Mexico was implicated as the vehicle in an outbreak of
meningitis and bacteremia that occurred among attendees of a large company picnic.
Twenty of the 40 affected individuals were pregnant women. One of the victims had
AIDS. Gram positive, non-spore forming rods were isolated from the cheese that had
been stored in the cold. The MOST LIKELY organism responsible for this outbreak is:
(A) Bacillus cereus
(B) Clostridium difficile
(C) Clostridium perfringens
(D) Listeria monocytogenes
Answer: D Medical Microbiology by Jawetz., 23rd edition,page 217

95. A 25 year old patient presents at a sexually transmitted disease (STD) clinic with a
painful genital ulcer. On examination, the ulcer does not appear to be typical of herpes
simplex virus. Dark field microscopy of exudate from the ulcer is negative for spiral-
shaped organisms. However, a small, gram negative rod is isolated from culture of the
exudate on chocolate agar. The organism MOST LIKELY to be responsible for the
lesion is:
(A) Chlamydia trachomatis.
(B) Haemophilus ducreyi.
(C) Neisseria gonorrhoae.
(D) Treponema pallidum.
Answer: B Medical Microbiology by Jawetz., 23rd edition,page 281

96. Which ONE of the following diseases is transmitted by lice?


(A) Epidemic typhus
(B) Ehrlichiosis
(C) Psitticosis
(D) Scrub typhus
Answer: A Medical Microbiology by Jawetz., 23rd edition,page 350

97. A young army recruit presents at a sexually transmitted disease (STD) clinic clinic
with acute urethritis. A Gram-stained preparation of his urethral exudate reveals
neutrophils with intracellular Gram-negative diplococci. The patient is treated with
ceftriaxone and sent home. He is requested to return in one week so that a urethral
culture can be obtained to test for proof of antibiotic cure. Which ONE of the following
culture media should be used for the follow-up culture?
(A) Blood agar
(B) Bordet Gengou agar
(C) MacConkey agar
(D) Thayer-Martin agar
Answer: D Medical Microbiology by Jawetz., 23rd edition,page 295
98. The growth in the vagina of which ONE of the following bacteria helps maintain a
protective acid pH?
(A) Bacteroides
(B) Gardnerella
(C) Lactobacillus
(D) Mobiluncus
Answer: C Medical Microbiology by Jawetz., 23rd edition,page 307

99. A 25 year-old sexually active woman complains of vaginal itching, odor, and
discharge. The vaginal walls are red and slightly swollen, but the cervix is normal. A
sample of her vaginal discharge is tested with 10% KOH, which produces a strong fishy
odor. Upon microscopic examination of the discharge, you find clue cells but no pear-
shaped flagellates or pseudohyphae. A Gram stain reveals numerous Gram-negative
curved rods. What is your diagnosis?
(A) Bacterial vaginosis
(B) Pelvic inflammatory disease (PID)
(C) Syphilis
(D) Trichomoniasis
Answer: A Medical Microbiology by Jawetz., 23rd edition,page 316

100. A 25 year-old male develops an ulcerative lesion of the hand and several fluctuant
nodules along the lymphatics draining that site. The patient is seen in the dermatology
clinic at the Naval Hospital. Case history reveals that he is a weekend gardener who
cultivates roses as a hobby. What is the MOST LIKELY fungal disease?
(A) Chromoblastomycosis
(B) Mycetoma
(C) Phaeohyphomycosis
(D) Sporotrichosis
Answer: D Medical Microbiology by Jawetz., 23rd edition,page 633
BOARD EXAMINATION QUESTIONS IN MICROBIOLOGY AND PARASITOLOGY

Instruction: Choose the BEST answer. ( Note: The correct answer per item is in bold
and italized letters)

I. GIVEN A CLINICAL CONDITION/ SUSPECTED INFECTIOUS


CONDITION/DISEASE, DETERMINE THE APPROPRIATE SPECIMEN TO BE
COLLECTED / THE LABORATORY PROCEDURE TO BE REQUESTED (OR VICE-
VERSA).

A. RECALL ( 9 ITEMS)

BACTERIA ( 5 ITEMS)

1. Serodiagnosis is most useful in demonstrating previous pyogenic skin infection with:


A. Staphylococcus aureus
B. Streptococcus pyogenes
C. Both A and B
D. Neither A nor B

Reference: Brooks G., et al, 2004. Jawetz, Melnick & Adelberg’s Medical
Microbiology. 23rd
Edition. The McGraw Hills Co., Inc. p. 239

2. Which will differentiate viridans streptococci from pneumococci?


A. CAMP Test
B. Optochin Test
C. Catalase Test
D. Bacitracin Test

Reference: Brooks G., et al, 2004. Jawetz, Melnick & Adelberg’s Medical
Microbiology. 23rd
Edition. The McGraw Hills Co., Inc. p. 240

3. The best means of making the laboratory diagnosis of Legionella is::


A. Gram stain smear and culture of infeted tissue
B. Detection of antigen by immunoassay and culture of urine
C. Direct fluorescence antibody and culture of infected tissue
D. Serology to demonstrate a rise in serum antibody and culture of blood

Reference: Brooks G., et al, 2004. Jawetz, Melnick & Adelberg’s Medical
Microbiology. 23rd
Edition. The McGraw Hills Co., Inc. p. 274

4. Diagnosis of tetanus is usually based on:


A. microscopis and cultural methods
B. serologic detection of circulating antibodies
C. clinical grounds
D. all of the above

Reference: Brooks G., et al, 2004. Jawetz, Melnick & Adelberg’s Medical
Microbiology. 23rd
BOARD EXAMINATION QUESTIONS IN MICROBIOLOGY AND PARASITOLOGY

Reference: Brooks G., et al, 2004. Jawetz, Melnick & Adelberg’s Medical
Microbiology. 23rd
Edition. The McGraw Hills Co., Inc. p. 742

FUNGI (1 ITEM)

6. The most specific approach for diagnosing cryptococcosis is:


A. culture
B. direct KOH examination
C. India ink examination
D. Antibody level determination

Reference: Brooks G., et al, 2004. Jawetz, Melnick & Adelberg’s Medical
Microbiology. 23rd
Edition. The McGraw Hills Co., Inc. p. 647.

PARASITES ( 3 ITEMS)

7. Best method to diagnose malaria is by doing:


A. Direct fecal examination
B. Thick and thin smear of blood
C. Serologic test
D. Manzotti’s test

Reference: Brown & Neva, 1983. Basic Clinical Parasitology. 5th Edition. The
McGraw Appleton-Century
Crofts. p. 90.
Brooks G., et al, 2004. Jawetz, Melnick & Adelberg’s Medical Micro
23rd Ed. The McGraw Hills
Co., Inc. p. 681

8. Finding eggs, scolex and gravid segments in the stool is seen in which of the
following diseases?
A. Endemic hemoptysis
B. Taeniasis
C. Trichinosis
D. Toxoplasmosis

Reference: Brown & Neva, 1983. Basic Clinical Parasitology. 5th Edition. The
McGraw Appleton-Century
Crofts. p. 90.
Brooks G., et al, 2004. Jawetz, Melnick & Adelberg’s Medical
Microbiology. 23rd
Edition. The McGraw Hills Co., Inc. p. 681

9. Extremely high eosinophilic count in the blood and CSF is seen in:
A. Giardiasis
B. Hookworm infection
C. Schistosomiasis
BOARD EXAMINATION QUESTIONS IN MICROBIOLOGY AND PARASITOLOGY

BACTERIA (4 ITEMS)

10. A 41 y/o female consulted her gynecologist due to vaginal itchiness with grayish-
fishy odor vagiinal discharge. The laboratory test most likely to be ordered by her
physician would be:
A. Pap smear
B. Gram stain
C. CBC
D. Urinalysis

Reference: Brooks G., et al, 2004. Jawetz, Melnick & Adelberg’s Medical
Microbiology. 23rd
Edition. The McGraw Hills Co., Inc. p. 698

11. An 18 y/o college male sophomore student consulted at the infirmary because of
pain on urination, with purulent urethral discharge of one day duration. Three days
before, he went out with “barkada” for some “thrilla” in Manila. As a medical
microbiology student, what examination will you request to make an etiologic
diagnosis of this infection?
A. Tzanck’s smear of the urethral exudates
B. Gram’s stain of the urethral smear
C. PCR or LCR of the urine
D. Antibody tests for N. gonorrhea

Reference: Brooks G., et al, 2004. Jawetz, Melnick & Adelberg’s Medical
Microbiology. 23rd
Edition. The McGraw Hills Co., Inc. p. 263

12. A 16- year old female patient developed fever, headache and abdominal pain.
You suspect her to have typhoid fever. The best specimen for the diagnosis should
be:
A. Stool culture on the second week of illness
B. Blood culture during the first week of illness
C. 4-fold increase in antibody titer in acute and convalescent sera
D. serology to demonstrate Salmonella antigen

Reference: Brooks G., et al, 2004. Jawetz, Melnick & Adelberg’s Medical
Microbiology. 23rd
Edition. The McGraw Hills Co., Inc. p. 228

FUNGI (2 ITEMS)

13. A 60 year old male developed painful erythematous swelling of the nail fold. Skin
and nail scrapings reveal pseudohphae and budding cells. This finding confirms that
your patient is infected with:
A. Sporotrix schenckii
B. Cryptococcus neoformans
C. Malassezia furfur
D. Candida albicans

Reference: Brooks G., et al, 2004. Jawetz, Melnick & Adelberg’s Medical
rd
BOARD EXAMINATION QUESTIONS IN MICROBIOLOGY AND PARASITOLOGY

C. hair clippings
D. wound exudates

Reference: Brooks G., et al, 2004. Jawetz, Melnick & Adelberg’s Medical
Microbiology. 23rd
Edition. The McGraw Hills Co., Inc. p. 587.

PARASITES (3 ITEMS)

15. Recurrent hemoptysis of a 39 year old patient from Basilan Island should be
advised to undergo:
A. Sputum examination for fluke ova
B. Chest x-ray examination
C. Sputum microscopy for AFB
D. All of the above

Reference: Brown & Neva, 1983. Basic Clinical Parasitology. 5th Edition. The
McGraw Appleton-Century
Crofts. p. 231.

16. Three weeks after eating “kilawing kuhol”, a freshwater snail, the patient
developed persistent intestinal discomfort. You are suspecting Garrison’s disease.
To confirm your diagnosis, you will request for:
A. Seroligcal testing for IgG antibody
B. Fecalysis for detection of ova
C. Peripheral smear for asexual stages
D. Complete blood count for eosiniphilia

Reference: Brown & Neva, 1983. Basic Clinical Parasitology. 5th Edition. The
McGraw Appleton-Century
Crofts. pp. 219-220.

17. A five year old patient with enlarged abdomen was found tohave palpable mass
on the left side of the his abdomen. Should this be a case of intestinal obstruction
due to ascariasis, what diagnostic procedures are you goin to do to initially support
the etiologic cause of the disease ?
A. Plain X-ray of the abdomen
B. Fecalysis for detection of ova
C. Do surgical intervention to get the bolus
D. None, just wait for the passageof adult worm.

Reference: Brown & Neva, 1983. Basic Clinical Parasitology. 5th Edition. The
McGraw Appleton-Century
Crofts. p. 135
BOARD EXAMINATION QUESTIONS IN MICROBIOLOGY AND PARASITOLOGY

A. RECALL (13 ITEMS)

VIRUSES ( 4 ITEMS)

18. Which of the following viruses commonly causes coryza and nasal congestion?
A. RSV
B. Influenzae
C. Parainfluenza
D. Rhinovirus

Reference: Brooks G., et al, 2004. Jawetz, Melnick & Adelberg’s Medical
Microbiology. 23rd
Edition. The McGraw Hills Co., Inc. p. 489.

19. Pain and swelling of the jaw ina patient with preceeding history of fever, nasal
discharge and muscle pain would lead you to suspect which of the viruses below?
A. Adenovirus
B. Mumps virus
C. EBV
D. CMV

Reference: Brooks G., et al, 2004. Jawetz, Melnick & Adelberg’s Medical
Microbiology. 23rd
Edition. The McGraw Hills Co., Inc. p. 561.

20. A female patient is presented with fever, anorexia, nausea and right upper
quadrant pain followed by dark urine and clay colored stools. She is most likely to be
infected with:
A. rotavirus
B. poliovirus
C. hepatitis A virus
D. hepatitis G virus

Reference: Brooks G., et al, 2004. Jawetz, Melnick & Adelberg’s Medical
Microbiology. 23rd
Edition. The McGraw Hills Co., Inc. p. 481.

21. A 6-year old school age female patient was diagnosed to suffer from subacute
sclerosing panencephalitis. The virus responsible for this condition is:
A. Roseola
B. Rubeola
C. Varicella
D. Rubella

Reference: Brooks G., et al, 2004. Jawetz, Melnick & Adelberg’s Medical
Microbiology. 23rd
Edition. The McGraw Hills Co., Inc. p. 562-563.

BACTERIA ( 5 ITEMS)

22. The most common cause of community acquired adult meningitis is:
BOARD EXAMINATION QUESTIONS IN MICROBIOLOGY AND PARASITOLOGY

Edition. The McGraw Hills Co., Inc. p. 735

23. Woolsorter’s disease is caused by:


A. Bacillus anthracis
B. Streptococcus pneumoniae
C. Listeria monocytogenes
D. Klebsiella pneumoniae

Reference: Brooks G., et al, 2004. Jawetz, Melnick & Adelberg’s Medical
Microbiology. 23rd
Edition. The McGraw Hills Co., Inc. p. 203

24. Etiologic agent of Whooping cough:


A. B. avium
B. B. pertussis
C. B. parapertussis
D. All of the above

Reference: Brooks G., et al, 2004. Jawetz, Melnick & Adelberg’s Medical
Microbiology. 23rd
Edition. The McGraw Hills Co., Inc. p. 249

25. Causative agent of Bubonic plague:


A. Rickettsia ricketssia
B. Yersinia pestis
C. Brucella abortus
D. Rickettsia prowazekii

Reference: Brooks G., et al, 2004. Jawetz, Melnick & Adelberg’s Medical
Microbiology. 23rd
Edition. The McGraw Hills Co., Inc. p. 254

26. Hemolytic uremic syndrome (HUS) may be cause by:


A. Diffusely adherent E. coli
B. Enteroaggregative E. coli
C. Enterohemorrhagic E. coli
D. Enteropathogenic E. coli

Reference: Brooks G., et al, 2004. Jawetz, Melnick & Adelberg’s Medical
Microbiology. 23rd
Edition. The McGraw Hills Co., Inc. p. 223

FUNGI (1 ITEM)

27. The agent of “black-dot” tinea capitis is:


A. Trichophyton schoenleinii
B. Trichophyton tonsurans
C. Trichophyton rubrum
D. Epidermophyton floccosum
BOARD EXAMINATION QUESTIONS IN MICROBIOLOGY AND PARASITOLOGY

A. Brugia malayi
B. Dengue
C. Wuchereria bancrofti
D. Loa loa

Reference: Brown & Neva, 1983. Basic Clinical Parasitology. 5th Edition. The
McGraw Appleton-Century
Crofts. p. 144

29. Hepatosplenomegaly is the hallmark of this disease which is endemic in Leyte,


Samar and Mindoro. This is caused by:
A. Schistosoma japonicum
B. Echinostoma ilocanum
C. Pila luzonica
D. Capillaria philippinensis

Reference: Brown & Neva, 1983. Basic Clinical Parasitology. 5th Edition. The
McGraw Appleton-Century
Crofts. p. 241

30. Pruritus vulvae is a characteristic symptom in :


A. Trichinosis
B.. Trichomoniasis
C. Enterobiasis
D. Scabies

Reference: Brown & Neva, 1983. Basic Clinical Parasitology. 5th Edition. The
McGraw Appleton-Century
Crofts. p 46

B. APPLICATION

VIRUSES (4 ITEMS)

31. A 9-month old infant was brought to the emergency room because of high grade
fever of 39oC of 2 days duration, cough and dyspnea. On physical examination,
there was increased respiratory rate and retractions of the intercostals muscles. The
virus was isolated only from the specimen which was labeled as “ nasal wash”. The
etiologic agent responsible for the condition of the patient is:
A. Respiratory Syncitial virus (RSV)
B. Coronavirus
C. Rhinovirus
D. Influenza virus

Reference: Brooks G., et al, 2004. Jawetz, Melnick & Adelberg’s Medical
Microbiology. 23rd
Edition. The McGraw Hills Co., Inc. p. 558-559.

32. A 6-year old female consulted the pediatrician because of sore throat, fever and
BOARD EXAMINATION QUESTIONS IN MICROBIOLOGY AND PARASITOLOGY

Reference: Brooks G., et al, 2004. Jawetz, Melnick & Adelberg’s Medical
Microbiology. 23rd
Edition. The McGraw Hills Co., Inc. p. 435.

33. An 11-month old infant was brought to the emergency room because of a 3 day
history of fever with mild upper respiratory tract symproms. On the 4 th day,
maculopapular rashes appeared on the trunk and neck The infant was positive for
anti-HHV 6 Igm and cultures were likewise positive for HHV 6. Patient has:
A. Rubeola
B. Rubella
C. Varicella
D. Exanthem subitum

Reference: Brooks G., et al, 2004. Jawetz, Melnick & Adelberg’s Medical
Microbiology. 23rd
Edition. The McGraw Hills Co., Inc. p. 449.

34. A 35-year old male consulted because of fever, anorexia, body malaise and
yellowish discoloration of the skin and eyes. History showed that he underwent
surgery two months prior to consultation and was transfused 6 units of whole blood.
The virus which can cause the symptoms in the patient would be:
A. Hepatitis B virus
B. Hepatitis G virus
C. Hepatitis C virus
D. All of the above

Reference: Brooks G., et al, 2004. Jawetz, Melnick & Adelberg’s Medical
Microbiology. 23rd
Edition. The McGraw Hills Co., Inc. p. 484.

BACTERIA (2 ITEMS)

35. A 46-year old female is diagnosed with subacute bacterial endocarditis. It is


most commonly caused by organisms known as:
A. oral streptococci
B. pyogenic streptococci
C. enterococci
D. pneumococci

Reference: Brooks G., et al, 2004. Jawetz, Melnick & Adelberg’s Medical
Microbiology. 23rd
Edition. The McGraw Hills Co., Inc. p. 237

36. A 35 y/o female developed gastric ulcer. Biopsy on H and E stain revealed
numerous rod shape bacilli. The most likely organism would be:
A. Campylobacter jejuni
B. Helicobacter pylori
C. Shigella dysentery
D. Vibrio cholerae
BOARD EXAMINATION QUESTIONS IN MICROBIOLOGY AND PARASITOLOGY

B. S. aureus
C. B. cereus
D. V. cholerae

Reference: Brooks G., et al, 2004. Jawetz, Melnick & Adelberg’s Medical
Microbiology. 23rd
Edition. The McGraw Hills Co., Inc. p. 238

38. A 24 y/o female had a normal vaginal delivery. Two days after, she noted
yellowish discharge from her baby’s eyes. Gram stain showed gram negative
intracellular diplococci. Most likely organism would be:
A. Candida albicans
B. Neisseria gonorrheae
C. Chlamydia trachomatis
D. Trichomonas vaginalis

Reference: Brooks G., et al, 2004. Jawetz, Melnick & Adelberg’s Medical
Microbiology. 23rd
Edition. The McGraw Hills Co., Inc. pp. 262-263

39. A 22 y/o female who suffered from burn wounds developed greenish pus on the
infected area after three days. The most likely organism you would suspect is:
A. E. coli
B. P. aeruginosa
C. P. vulgaris
D. H. influenzae

Reference: Brooks G., et al, 2004. Jawetz, Melnick & Adelberg’s Medical
Microbiology. 23rd
Edition. The McGraw Hills Co., Inc. p. 232

FUNGI (2 ITEMS)

40. A gardener from flower farm in Tagaytay developed an ulcerative lesions on his
middle finger associated with multiple subcutaneous nodules on her affected arm.
The patient recalled that he was accidentally pricked by rose thorn a month ago.
This infection is most probably caused by:
A. Aspergillus fumigatus
B. Cryptococcus neoformans
C. Sporothrix schenkii
D. Histoplasma capsulatum

Reference: Brooks G., et al, 2004. Jawetz, Melnick & Adelberg’s Medical
Microbiology. 23rd
Edition. The McGraw Hills Co., Inc. p. 634

41. Histological finding of the lungs of an AIDS patient shows small ovoid cells
observed inside the macrophages using Gomori’s methenamine silver stain confirms
the presence of:
A. Cryptococcus neoformans
B. Penicillium marneffei
BOARD EXAMINATION QUESTIONS IN MICROBIOLOGY AND PARASITOLOGY

PARASITES ( 4 ITEMS)

42. A 12 year old girl developed steatorrheic diarrhea after the family vacation in
South Cotabato. In examining her fecal sample, you expect to find :
A. Positive turbidity in peptone broth for possible presence of Vibrio cholerae
B. Cysts of Entamoeba histolytica
C. Large trophozoites of Balantidium coli
D. Binucleated cysts and pear-shaped trophozoite of Giardia lamblia

Reference: Brown & Neva, 1983. Basic Clinical Parasitology. 5th Edition. The
McGraw Appleton-Century
Crofts. pp. 45

43. Subcutaneous nodules associated with muscle pains, eosinophilia and


gastrointestinal discomfort are classic presentation of:
A. Obstructive Lymphatic filariasis
B. Cysticercosis
C. Trichinosis
D. Toxoplasmosis

Reference: Brown & Neva, 1983. Basic Clinical Parasitology. 5th Edition.
The McGraw Appleton-Century
Crofts. pp 110-111

44. Finding larvae in a freshly voided fecal sample from a patient with anemia. The
parasite is most likely:
A. 3rd stage of Ascaris lumbricoides
B. Rhabditiform larvae of Strongyloides stercoralis
C. Immature adult of Trichuris trichiura
D. Adult female of Enterobius vermicularis

Reference: Brown & Neva, 1983. Basic Clinical Parasitology. 5th Edition. The
McGraw Appleton-Century
Crofts. Pp. 119

45. Clinical paroxysms of chills, fever and sweating occur every 72 hours is due to
the invasion of:
A. all ages RBC by merozoites of Plasmodium falciparum
B. reticulocytes by the sporozoites of Plasmodium vivax
C. mature RBCs by merozoites of Plasmodium malariae
D. the liver cells by the primary tissue schizonts of Plasmodium ovale

Reference: Brown & Neva, 1983. Basic Clinical Parasitology. 5th Edition. The
McGraw Appleton-Century
Crofts. p. 14-15

III. GIVEN LABORATORY DATA, IDENTIFY THE INFECTIOUS AGENT/DISEASE.


( OR VICE VERSA).
BOARD EXAMINATION QUESTIONS IN MICROBIOLOGY AND PARASITOLOGY

C. Variola
D. Varicella

Reference: Brooks G., et al, 2004. Jawetz, Melnick & Adelberg’s Medical
Microbiology. 23rd
Edition. The McGraw Hills Co., Inc. p. 460.

47. Detection of heterophil antibodies in serum and atypical lymphocytes in the


peripheral blood smear are characteristics of:
A. Epstein Barr virus
B. Cytomegalovirus
C. Herpes Simplex virus
D. Varicella Zoster virus

Reference: Brooks G., et al, 2004. Jawetz, Melnick & Adelberg’s Medical
Microbiology. 23rd
Edition. The McGraw Hills Co., Inc. p. 671.

BACTERIA ( 5 ITEMS)

48. The ability to clot plasma is a differential feature of:


A. Staphylococcus aureus
B. Staphyloccous epidermidis
C. Staphylococcus saprophyticus
D. All of the above

Reference: Brooks G., et al, 2004. Jawetz, Melnick & Adelberg’s Medical
Microbiology. 23rd
Edition. The McGraw Hills Co., Inc. p. 227

49. Which is a Gram-negative anaerobic bacillus?


A. Neisseria meningitides
B. Bacteroides fragilis
C. Bacillus anthracis
D. Clostridium perfringens

Reference: Brooks G., et al, 2004. Jawetz, Melnick & Adelberg’s Medical
Microbiology. 23rd
Edition. The McGraw Hills Co., Inc. p. 306

50. Which test is used to differentiate E. coli from P. aeruginosa?


A. catalase
B. coagulase
C. oxidase
D. hemolysis

Reference: Brooks G., et al, 2004. Jawetz, Melnick & Adelberg’s Medical
Microbiology. 23rd
Edition. The McGraw Hills Co., Inc. p. 263

51. Hemin and NAD are both essential for the growth of
A. H. parainfluenza
BOARD EXAMINATION QUESTIONS IN MICROBIOLOGY AND PARASITOLOGY

52. Typhidot is positive in infections with:


A. Shigella
B. Salmonella
C. Vibrio
D. Campylobacter

Reference: Brooks G., et al, 2004. Jawetz, Melnick & Adelberg’s Medical
Microbiology. 23rd
Edition. The McGraw Hills Co., Inc. p. 228

FUNGI ( 1 ITEM)

53. Negative India ink staining is used to identify isolates of:


A. Aspergillus fumigatus
B. Candida albicans
C. Cryptococcus neoformans
D. Histoplasma capsulatum

Reference: Brooks G., et al, 2004. Jawetz, Melnick & Adelberg’s Medical
Microbiology. 23rd
Edition. The McGraw Hills Co., Inc. p. 648

PARASITES (3 ITEMS)

54. Finding banana- or sausage-shaped gametocytes in peripheral smear is a


unique characteristic of:
A. Plasmodium malariae
B. Plasmodium vivax
C. Plasmodium falciparum
D. Plasmodium ovale

Reference: Brown & Neva, 1983. Basic Clinical Parasitology. 5th Edition. The
McGraw Appleton-Century
Crofts. p. 81-82

55. Detection of hematophagous trophozoite in the stool confirms the diagnosis of:
A. Enterobius infection
B. Entamoeba histolytica infection
C. Giardia lamblia infection
D. Balantidum coli infection

Reference: Brown & Neva, 1983. Basic Clinical Parasitology. 5th Edition. The
McGraw Appleton-Century
Crofts. p. 28

56. Anchovy sauce aspirate from liver abscess is highly suggestive of:
A. Entamoeba histolytica
B. Bacterial abscess
C. Fasciola hepatica
BOARD EXAMINATION QUESTIONS IN MICROBIOLOGY AND PARASITOLOGY

VIRUSES ( 2 ITEMS)

57. The serum from a 4-year old female with cough, coryza, conjunctivitis and
maculopapular rash agglutinates red blood cells of monkeys in the hemaglutination-
inhibition tests. The agent responsible for this is:
A. Parainfluenza virus
B. Mumps virus
C. Measles virus
D. Respiratory scyncitial virus

Reference: Brooks G., et al, 2004. Jawetz, Melnick & Adelberg’s Medical
Microbiology. 23rd
Edition. The McGraw Hills Co., Inc. p. 563.

58. A 30-year old female cancer patient undergoing immunosuppression therapy


consults because of the appearance of recurrent painful vesicular lesions over her
right thoracic area. Tzank test from the scrapings at the base of the lesion revealed
multinucleated giant cells. Which among the following viruses is confirmed by the
above clinical data?
A. Herpes simplex 1
B. Herpes Zoster
C. Herpes simplex 2
D. Variola

Reference: Brooks G., et al, 2004. Jawetz, Melnick & Adelberg’s Medical
Microbiology. 23rd
Edition. The McGraw Hills Co., Inc. p. 441.

BACTERIA (5 ITEMS)

59. Sputum culture from a patient with pneumonia revealed beta-hemolytic, catalase-
positive cocci. The infecting organism is most probably:
A. Staphylococcus aureus
B. Staphylococcus epidermidis
C. Straptococcus pyogenes
D. Streptococcus pneumoniae

Reference: Brooks G., et al, 2004. Jawetz, Melnick & Adelberg’s Medical
Microbiology. 23rd
Edition. The McGraw Hills Co., Inc. p. 227

60. A Gram positive cocci was isolated from pustular lesions of a 5-year old boy. If
the isolate tested catalase negative, bacitracin sensitive, which agent is most likely
involved?
A. Steptococcus pyogenes
B. Staphylococcus epidermidis
C. Streptococcus agalactiae
D. Staphylococcus saprophyticus

Reference: Brooks G., et al, 2004. Jawetz, Melnick & Adelberg’s Medical
rd
BOARD EXAMINATION QUESTIONS IN MICROBIOLOGY AND PARASITOLOGY

Reference: Brooks G., et al, 2004. Jawetz, Melnick & Adelberg’s Medical
Microbiology. 23rd
Edition. The McGraw Hills Co., Inc. p. 275

62. Modified Thayer-Martin medium is used to isolate:


A. Haemophilus ducreyi
B. Chlamydia trachomatis
C. Neisseria gonorrhoeae
D. Gardnerella vaginalis

Reference: Brooks G., et al, 2004. Jawetz, Melnick & Adelberg’s Medical
Microbiology. 23rd
Edition. The McGraw Hills Co., Inc. p. 247

63. Definitive diagnosis of Leptospirosis consists of:


A. isolating the organism in culture plus a compatible clinical illness
B. microscopic agglutination titer of 1:100 plus a compatible clinical illness
C. positive slide agglutination test plus a compatible clinical illness
D. All of the above

Reference: Brooks G., et al, 2004. Jawetz, Melnick & Adelberg’s Medical
Microbiology. 23rd
Edition. The McGraw Hills Co., Inc. p. 297

64. Which of the following statements is TRUE regarding the typical Leptospira
organism?
A. It is a tightly coiled, thin, flexible organism
B. It grows best under aerobic condition in serum-containing semi-solid
medium(Fletcher’s medium)
C. It can be viewed under the electron or darkfield microscope
D. All of the above
E. Any two of the above

Reference: Brooks G., et al, 2004. Jawetz, Melnick & Adelberg’s Medical
Microbiology. 23rd
Edition. The McGraw Hills Co., Inc. pp. 295-297

FUNGI (2 ITEMS)

65. Histopathologic examination of the endotracheal aspirate with history of chronic


PTB shows conidial heads and hyaline septate hyphae with dichotomous
branchings confirms the concomi -tant disease with:
A. aspergillosis
B. dermatophytosis
C. histoplasmosis
D. zygomycosis

Reference: Brooks G., et al, 2004. Jawetz, Melnick & Adelberg’s Medical
Microbiology. 23rd
BOARD EXAMINATION QUESTIONS IN MICROBIOLOGY AND PARASITOLOGY

C. Penicillium spp.
D. Microsporum spp.

Reference: Brooks G., et al, 2004. Jawetz, Melnick & Adelberg’s Medical
Microbiology. 23rd
Edition. The McGraw Hills Co., Inc. p. 650

PARASITES ( 3 ITEMS)

67. Diarrheic stool of the patient with suspected Entamoeba histolytica infection
must be examined within one hour after voiding because:
A. cyst undergo excystation
B. trophozoites disintegrates
C. trophozoites encyst once they are excreted
D. none of the above

Reference: Brown & Neva, 1983. Basic Clinical Parasitology. 5th Edition. The
McGraw Appleton-Century
Crofts. Pp. 27-28

68. Which of the following techniques is employed most successfully for recovering
pinworm eggs?
A. Sugar fecal floatation technique
B. Direct fecal smear
C. Anal swabbing with cellophane
D. Fecal sedimentation

Reference: Brown & Neva, 1983. Basic Clinical Parasitology. 5th Edition. The
McGraw Appleton-Century
Crofts. Pp. 316

69. The Knott’s concentration technique is used to recover:


A. Adult W. bancrofti in the blood
B. Trypomastigotes of T. gambienze in blood smear
C. Microfilariae of B. malayi in lymphatic fluid and blood
D. Embryonated eggs of A. lumbricoides from feces

Reference: Brown & Neva, 1983. Basic Clinical Parasitology. 5th Edition. The
McGraw Appleton-Century
Crofts. Pp. 323

IV. GIVEN AN ENDEMIC DISEASE, IDENTIFY THE STAGE IN THE LIFE CYCLE OF
THE ETIOLOGIC AGENT THAT IS MOST VULNERABLE TO INTERVENTION.
BOARD EXAMINATION QUESTIONS IN MICROBIOLOGY AND PARASITOLOGY

C. Metagonimus yokogawai
D. Capillaria philippinensis

Reference: Brown & Neva, 1983. Basic Clinical Parasitology. 5th Edition. The
McGraw Appleton-Century
Crofts. p. 166

71. Eating improperly cooked freshwater fish is the main cause of transmission of :
A. Clonorchis sinensis
B. Diphyllobothrium latum
C. Capillaria philippinensis
D. All of the above

Reference: Brown & Neva, 1983. Basic Clinical Parasitology. 5th Edition. The
McGraw Appleton-Century
Crofts. pp. 217, 175-176, 166

72. Patient with anemia due to hookworm infection should be advised to :


A. use Iron tablets to correct the iron deficiency
B. take Antihelminthic drug to get rid of the worm
C. wear shoes and slippers regularly
D. All of the above

Reference: Brown & Neva, 1983. Basic Clinical Parasitology. 5th Edition. The
McGraw Appleton-Century
Crofts. p. 125- 126

73. Enormous number of housefly is a public health concern mainly because they:
A. may carry Salmonella and may cause food-borne epidemic of
salmonellosis
B. function a s intermediate host of trypanosome
C. can cause a poisonous sting
D. served as mechanical vector for transmission of Hepatitis B

Reference: Brown & Neva, 1983. Basic Clinical Parasitology. 5th Edition. The
McGraw Appleton-Century
Crofts. p. 279

B. APPLICATION

PARASITES ( 3 ITEMS)

74. A 10-year program for sanitary waste disposal in far-flung barangays in the
Philippines would eradicate:
A. Capillariasis
B. Echinostomiasis
C. Ascariasis
D. Enterobiasis

Reference: Brown & Neva, 1983. Basic Clinical Parasitology. 5th Edition. The
McGraw Appleton-Century
BOARD EXAMINATION QUESTIONS IN MICROBIOLOGY AND PARASITOLOGY

Reference: Brown & Neva, 1983. Basic Clinical Parasitology. 5th Edition. The
McGraw Appleton-Century
Crofts. Pp. 126

76. A 29-year old male patient fom Bulacan consulted because of recurrent
abdominal discomfort and passage of “ribbon-like structure” from his anus. He is
fond of eating roasted pork and beef . If this is pork tapeworm. The best treatment
would be:
A. to give Niclosamide immediately
B. to relay treatment for a month and advise patient regarding the disease
C. surgical intervention as soon as possible
D. All of the above

Reference: Brown & Neva, 1983. Basic Clinical Parasitology. 5th Edition. The
McGraw Appleton-Century
Crofts. p. 185

V. IDENTIFY APPROPRIATE MANAGEMENT AND CONTROL MEASURES FOR


GIVEN INFECTIOUS AGENTS/ DISEASE (OR VICE VERSA)

A. RECALL

VIRUSES ( 4 ITEMS)

77. Decreased use of contaminated needles and syringes by injection drug users
and use of safety devices by health care workers reduces transmission of:
A. Hepatitis B
B. Hepatitis C
C. Hepatitis D
D. All of the above

Reference: Brooks G., et al, 2004. Jawetz, Melnick & Adelberg’s Medical
Microbiology. 23rd
Edition. The McGraw Hills Co., Inc. p. 483.

78. Which of the following preventive measures for mumps is effective in school
setting?
A. Isolation of the patient until parotid swelling has resolved.
B. Vaccination as part of the MMR vaccine
C. Mumps immunoglobulin injection
D. Live virus mumps vaccine for patientson immunosuppressive therapy

Reference: Brooks G., et al, 2004. Jawetz, Melnick & Adelberg’s Medical
Microbiology. 23rd
Edition. The McGraw Hills Co., Inc. p. 565.

79. Proper handwashing practices and adequate disposal of enteric excretions are
important preventive measures for the transmission of:
A. rotavirus
B. astroviruses
BOARD EXAMINATION QUESTIONS IN MICROBIOLOGY AND PARASITOLOGY

80. Protection from mosquito bites and vector control are primary preventive
measures for:
A. Unconventional viruses
B. flaviviruses
C. poxviruses
D. coxsackieviruses

Reference: Brooks G., et al, 2004. Jawetz, Melnick & Adelberg’s Medical
Microbiology. 23rd
Edition. The McGraw Hills Co., Inc. p. 572.

BACTERIA (5 ITEMS)

81. The 23-valent vaccine is used to prevent diseases caused by:


A. Neisseria gonorrhoeae
B. Group B streptococci
C. Streptococcus pneumoniae
D. Hemophilus influenzae type B

Reference: Brooks G., et al, 2004. Jawetz, Melnick & Adelberg’s Medical
Microbiology. 23rd
Edition. The McGraw Hills Co., Inc. p. 242

82. Best available preventive measures for gas gangrene include:


A. antibiotic therapy
B. antitoxin administration
C. Both A and B
D. Neither A nor B

Reference: Brooks G., et al, 2004. Jawetz, Melnick & Adelberg’s Medical
Microbiology. 23rd
Edition. The McGraw Hills Co., Inc. p. 209

83. Drugs of choice of Lyme disease:


A. Doxycycline
B. Amoxycillin
C. Penicillin
D. All of the above

Reference: Brooks G., et al, 2004. Jawetz, Melnick & Adelberg’s Medical
Microbiology. 23rd
Edition. The McGraw Hills Co., Inc. p. 178

84. Antimicrobial drugs in the treatment of diphtheria serve to:


A. neutralize the toxin
B. eliminate the organism
C. confer toxin resistance to the organism
D. prevent further toxin absorption by tissues

Reference: Brooks G., et al, 2004. Jawetz, Melnick & Adelberg’s Medical
rd
BOARD EXAMINATION QUESTIONS IN MICROBIOLOGY AND PARASITOLOGY

Reference: Brooks G., et al, 2004. Jawetz, Melnick & Adelberg’s Medical
Microbiology. 23rd
Edition. The McGraw Hills Co., Inc. pp. 242 & 302

PARASITES (3 ITEMS)

86. Taenia solium and Trichinella spiralis can be prevented by eating properly
cooked:
A. fish
B. pork
C. beef
D. vegetables

Reference: Brown & Neva, 1983. Basic Clinical Parasitology. 5th Edition. The
McGraw Appleton-Century
Crofts. Pp. 185, 111

87. Recommendation for the control of hookworm in endemic areas include the
construction of sanitary facilities and :
A. wearing of footwear
B. reduction of the stray cats population
C. through cooking of meats and fish
D. use of insecticides to control mosquitoes and flies.

Reference: Brown & Neva, 1983. Basic Clinical Parasitology. 5th Edition. The
McGraw Appleton-Century
Crofts. p. 126

88. Loeffler’s syndrome is best treated with:


A. Metronidazole
B. Mebendazole
C. Niclosamide
D. Antibiotic

Reference: Brown & Neva, 1983. Basic Clinical Parasitology. 5th Edition. The
McGraw Appleton-Century
Crofts. p. 134-135

B. APPLICATION

VIRUSES (4 ITEMS)

89. A 35-year old term pregnant female has a history of on and off appearance of
vesicular lesions in her genitalia. History showed that her husband also has had the
same lesions in his genitals. What is the best mode of management from the
choices below?
A. Tell the patient to ignore the lesions since it will just disappear in time.
B. Advise the mother that she has to deliver her baby by caesarian section.
C. Deliver the baby vaginally
D. Give the patient a vaccine to prevent transmission of the infection to the infant
BOARD EXAMINATION QUESTIONS IN MICROBIOLOGY AND PARASITOLOGY

90. A 16-year old girl was bitten in the hands by their pet dog as she was trying to
remove unfinished food. The vaccination status of the dog for rabies is
questionable. The best thing to do is:
A. cleanse the wound thoroughly with soap and water.
B. give Penicillin and observe the dog for 10 days
C. inject rabies immune globulin and rabies vaccine
D. all of the above

Reference: Brooks G., et al, 2004. Jawetz, Melnick & Adelberg’s Medical
Microbiology. 23rd
Edition. The McGraw Hills Co., Inc. p. 519.

91. A 31-year old , pregnant patient in 32nd week AOG consulted because of
exposure to her niece who was diagnosed to have german measles. Patient does
not have a history of having had german measles nor a history of receiving rubella
vaccine. The best action would be to:
A. suggest termination of pregnancy
B. administer live attenuated rubella vaccine
C. give her rubella immune serum
D. follow her antibody liter serologically

Reference: Brooks G., et al, 2004. Jawetz, Melnick & Adelberg’s Medical
Microbiology. 23rd
Edition. The McGraw Hills Co., Inc. p. 567.

92. A 2-year old female consulted at the emergency room because of 3-day
remittent fever associated with blood tinged vomitus. Complete blood count showed
markedly decreased platelets and an increased hematocrit. Diagnosis is dengue
hemorrhagic fever. Preventive measure/s consist of:
A. insecticidal fogging in the neighborhood
B. vaccination be given to all children
C. removal of containers with water which can serve as vessels to egg deposition of
the vector
D. all of the above

Reference: Brooks G., et al, 2004. Jawetz, Melnick & Adelberg’s Medical
Microbiology. 23rd
Edition. The McGraw Hills Co., Inc. p. 527.

BACTERIA (5 ITEMS)

93. In 1988, a convention of Philippine War Veterans was held in a big Convention
Center . To check on the safety to prevent occurrence of Legionella infection
among elderly participants, you should advise to:
A. Minimize production of aerosols from the air-conditioning equipments
B. hyperchlorinate the water of the colling towers
C. elevate water temperatures above 70 oC
D. All of the above

Reference: Brooks G., et al, 2004. Jawetz, Melnick & Adelberg’s Medical
rd
BOARD EXAMINATION QUESTIONS IN MICROBIOLOGY AND PARASITOLOGY

D. All of the above

Reference: Brooks G., et al, 2004. Jawetz, Melnick & Adelberg’s Medical
Microbiology. 23rd
Edition. The McGraw Hills Co., Inc. p. 294

95. A 30 y/o male developed diarrhea, vomiting and mild abnormal pain after eating
one day old cooked rice for lunch. You suspect food poisoning with B. cereus, hence
you would advice:
A. supportive measures such as fluid replacement for diarrhea
B. immediate antibiotic treatment
C. hospitalization
D. All of the above

Reference: Brooks G., et al, 2004. Jawetz, Melnick & Adelberg’s Medical
Microbiology. 23rd
Edition. The McGraw Hills Co., Inc. p. 184

96. A 38 y/o female went to the emergency room due to dehydration with profuse
diarrhea with 12 episodes in 4 four hours and vomiting. The stools were
characterized as rice watery stools. You suspect infection with V. cholerae, hence
you would first:
A. give supportive measures and send her home
B. hook her on IV fluids and advise admission
C. order stool exam and give antibiotics
D. request CBC, urinalysis and stool examination

Reference: Brooks G., et al, 2004. Jawetz, Melnick & Adelberg’s Medical
Microbiology. 23rd
Edition. The McGraw Hills Co., Inc. p. 28-239

97. Which is/are true of the differences between Chlamydia and viruses?
A. Chlamydia reproduce by budding while viruses reproduce by fission
B. Chlamydia contains either RNA or DNA while viruses contain both RNA and DNA
C. Chlamydia are susceptible to antibiotics while viruses are not susceptible
D. All of the above

Reference: Brooks G., et al, 2004. Jawetz, Melnick & Adelberg’s Medical
Microbiology. 23rd
Edition. The McGraw Hills Co., Inc. p. 311-312

FUNGI (1 ITEM)

98. A swimmer developed small papular itchy skin lesion on his back. Few weeks
later, the the lesions disappeared but replaced by whitish discoloration of the skin.
Which of the statements is NOT correct regarding this skin infection?:
A. Sharing of towel and soap after swimming prevent the transmission to other
swimmers.
B. Personal hygiene is important in the prevention of pityriasis versicolor
BOARD EXAMINATION QUESTIONS IN MICROBIOLOGY AND PARASITOLOGY

99. Humans become infected when they swallow the eggs and develop cysts in
various organs to cause hydatidosis. This tapeworm is could be best prevented by:
A. reducing larval E. multilocularis in sheep and hogs
B. reducing adult E. granulosus infection in dogs
C. proper and sanitary disposal of waste
D. surgical intervention

Reference: Brown & Neva, 1983. Basic Clinical Parasitology. 5th Edition. The
McGraw Appleton-Century
Crofts. p. 197

100. Dyspnea in Loeffler’s syndrome caused by larval invasion in massive ascariasis


in the lungs could be best treated with:
A. salbutamol nebulization
B. giving mebendazole to treat the infection
C. both A and B
D. neither A nor B

Reference: Brown & Neva, 1983. Basic Clinical Parasitology. 5th Edition. The
McGraw Appleton-Century
Crofts. Pp. 134-135
DEPARTMENT OF MICROBIOLOGY

MULTIPLE CHOICE: Choose the best answer.

1. Alkaline pH in urine seen in Proteus is due to its ability to split urea into:
A. NH3 & C02 B. Cl & Ca C. Ca & Mg D. Na & Cl
Answer: A (pp.208)

2. It is responsible for Escherichia coli causing meningitis in infants


A. H antigen B. LPS C. K antigen D. O antigen
Answer: C (pp.208)

3. “Swarming phenomenon” is associated with


A. Proteus B. Morganella C. E.coli D. Salmonella
Answer: (p205)

4. The presence of blood & mucus in diarrhea is suggestive of


A. Salmonellosis B. Cholera C. Shigellosis D. all
Answer: C (p256**)

5. It has the ability to produce phenylalanine deaminase


A. Citrobacter B. E. coli C. Proteus D. Klebsiella
Answer: C (p205)

6. Vi antigen for Salmonella typhi


A. penetrates epithelial lining of the intestinal tract
B. antigenic structure associated with invasiveness
C. interfere with antibody binding to bacteria
D. confers a certain degree of immunity
Answer: B (p 209)

7. Escherichia coli 0157:H7 is associated with


A. Shiga toxin B. verotoxin C. exotoxin D. an antitoxin
Answer: B (p208)

8. Enterobactreriaceae which is encapsulated


A. Klebsiella B. Shigella C. Enterobacter D. Citribacter
Answer: A (p204)

9. The best method for diagnosis of Typhoid fever


A. blood culture B. stool culture C. WIDAL test D. Typhi dot
Answer: A (p210)

10. Salmonella food poisoning is due to


A. S. typhimurium b. S. typhi c. S. paratyphi d. S. enteritidis
Answer: A (p210)

11. Initial recognition event for the activation of the alternative pathway
A. C3b B.C1 C.C4 D. Factor D
Answer: A (p 108)

12. True regarding viral envelope


A. lipid-containing membrane
B. confers stability to the virus
C. acquired during the late synthetic stage of viral replication
D. all of the above
Answer: A (p 324)

13. The characteristic morphologic feature of influenza responsible for antigenic


changes of the virus
A. envelope B. segmented genome C. capsid D. nucleic acid
Answer: B (p 320)

14. The largest & most complex virus


A. CMV B. Poxviruses C. Togavirus D. Rhabdovirus
Answer: B (p318)

15. Single-stranded RNA virus/es


A. Reovirus B. Parvovirus C. Influenza virus D. all of the above
Answer: C (p320)

16. Replicate in the cytoplasm of the host cell


A. Adenovirus B. Orthomyxovirus C. Poxviruses D. HIV
Answer: C (p332)

17. The exudative lung parenchymal lesion with lympadenopathy in primary tuberculosis
infection is called
A. tuberculosis B. granuloma C. ghon complex D. fibrotic lesion
Answer: C (p278)

18. Acquire its envelope from the endoplasmic reticulum


A. Poxviruses B. Coronaviruses C. Herpes viruses D. HIV
Answer: B (p487)

19. Physical separation of the viral capsid from the viral genome
A. penetration B. adsorption C. uncoating D. budding
Answer: C ( p330)

20. Requires appropriate pH & ionic concentration


A. adsorption B. penetration C. assembly D. uncoating
Answer: A (p330)

21. Virus losses its infectivity during


A. eclipse period B. assembly C. synthetic period D. release
Answer: A (p332)

22. Nucleic acid when purified can carry-out its replication


A. pseudovirion B. infectious nucleic acid C. viroids D. DI particles
Answer: B (p330)

23. Which of the following statement/s is correct regarding neonatal herpes


A. infections are almost always asymptomatic
B. most common route of infection is thru contact with herpetic lesion during vaginal
delivery
C. causes latent infection
D. usually cause by HSV 1
Answer: B (p376)

24. The ffg are correct statements regarding the epidemiology of HSV
A. the highest incidence of HSV-1 infection occurs among children 6 mos to 3 yrs of
age
B. The virus spread is spread by respiratory route
C. virus shedding in the stool last for years
D. pregnant women usually develop disseminated disease after 1st infection
Answer: A (p377)

25. Smallpox have been totally eradicated due to immunization. The success of
smallpox vaccination is due to
A. there’s only one stable serotype of smallpox
B. their is an animal serotype of the virus
C. carrier state can develop
D. no Antibodies develop against the virus
Answer: A (p395)

26. Which has been called the kissing disease?


A)Laryngitis B) Mononucleosis C)Herpes/cold sores D.)Tonsillitis
Answer: B (p386)

27. Poxviruses acquire its envelope from


A. cytoplasmic membrane C. nuclear membrane
B. inclusion bodies D. endoplasmic reticulum
Answer: B ( p395)

28. An 18 year old college student developed fever followed by jaundice. Acute viral
hepatitis was entertained. Results of the hepatitis profile are as follows: anti-HAV
IgM (-), anti-HbsAg (-), anti-HBc IgM (+), anti-HBc IgG negative. He most probably
has:
A. Hepatitis A B. Hepatitis B
C. Heatitis C D. Hepatitis E
Answer: B ( p 410)

29. A household of 5 members had cough,coryza and fever with muscle and joint pains
almost at the same time. A diagnosis of influenza was given by their family
physician. Which of the following can cause similar illness?
A. coronavirus B. mumps C. measles D. chicken pox
Answer: A ( p 490)

30. A 21 y/o male with history of sexual contact 4 days ago resents with painful urination
and purulent penile discharge. Which of the following is most likely?
A. primary syphilis B. genital herpes
C. gonorrhea D.lymphogranuloma venerum
Answer: C (p259)

31. A 1 and half y/o boy developed fever, coryza and cough followed 4 days later by
maculopapular rashes starting from the hairline and becoming generalized. Fever
persisted. The most likely etiology is a/an
A. RNA virus B DNA virus c. gram positive coccus D. gram negative bacillus
Answer: A ( p 470)

32. A neonate is born with the following features: microcephaly, jaundice, poor suck and
chorioretinitis. A congenital infection is entertained. The most common congenital
infection is due to:
A. Chlamydia B.HIV C. CMV D. HSV
Answer: C ( p 382)

33. A 2 month old girl was brought to you because of milk curd-like lesions in the mouth
noted for a week. The whitish lesions were adherent to the oral mucosa with pinpoint
bleeding when the mother tried to remove them. This is caused by
A. herpes simplex 1 B. Coxsackie A C. C. albicans D. C. neoformans
Answer: C ( p 550)

34. A 23 y/o office worker found out that his sexual partner had HIV infection. The
screening procedure of choice for HIV infection is
A. PCR B. HIV Elisa C. western blot D. northern blot
Answer: B ( p 525)

35. A 6 month old baby has acute watery diarrhea. Which of the following is the most
common cause of diarrhea in this age group?
A. ETEC B. rotavirus C. EPEC D. V. cholerae
Answer: B ( p434)

36. A health worker who did not receive prior mumps vaccine developed parotid swelling
and fever. The following belongs to the same virus family as mumps virus
A. german measles B. RSV C. Influenza D. Japanese B virus
Answer: B ( p 471)

37. A 25 yr old saleslady developed high fever, chills and cough. On PE she had
crackling rales on the right lung base. The diagnosis is pneumonia. The most
common bacterial cause of this condition in this age group is:
A. S. aureus B. N. meningitidis C. S. pneumoniae D. Hemophilus influenzae
Answer: C ( p 213)

38. Infection with this organism is the most common risk factor for cervical CA
A. HIV B. N. gonorrhea C. HPV D. HSV 2
Answer: C ( p512)

39. Most common cause of UTI in women is characterized by


A. Urease production B. fruity odor on culture
C. greenish metallic sheen D. swarming phenomenon
Answer: C ( p 219)

40. Which of the following laboratory tests can be used to monitor response to treatment
in syphilis?
A. FTA-ABS B. VDRL C. Darkfield microscopy D. ELISA
Answer: B ( p 287)

41. An encapsulated fungi that causes meningitis is


A. H. capsulatum B. C. neoformans C. C albicans D. C. immitis
Answer: B ( p 552)

42. The following statements are true regarding N. meningitidis


A. it is the only specie that is oxidase positive
B. oxidizes maltose and glucose
C. oxidizes glucose only
D. it is the only gram negative specie
Answer: B (p256)

43. Most common sequelae of streptocci following a pharyngeal infection


A. Rheumatic fever B. Acute glomerulonephritis
C. pyoderma D. impetigo
Answer: B ( p.209)
44. The following statements are true about S. aureus
A. it is catalase negative C. prefers anaerobic condition
B. can secrete preformed toxins D. canoot grow in 7.5% NACL
Answer: B (p.199)

45. Toxic shock syndrome secondary to Staphylococcus tend to commonly cause


infection among
A. tampon users B. asthamtics C. IV drug users D. infants
Answer: A (p 201)

46. A 14 year old male has high fever for a week, tonsillitis, hepatosplenomegaly and
cervical lymphadenopathy. Peripheral smear shows 10% atypical lymphocytes. The
most likely diagnosis is:
A. Streptococcal sore throat B. leukemia
C. infectious mononucleosis D. diphtheria
Answer: C (p 386)

47. Streptococcal sore throat is definitively diagnosed by


A. ASO titer B. throat swab culture
C. rapid antigen testing from throat swab D. Gram stain
Answer: B (p 210)

48. Which of the following, when positive, indicates an acute Epstein-Barr virus
infection?
A. anti-EBNA B. anti-VCA IgG C. anti-VCA IgM D. anti-EBV DNA
Answer: C (p 388)

49. The hallmark of Dengue Hemorrhagic Fever is


A. thrombocytopenia B. shock C. plasma loss D. positive tourniquet test
Answer: C (p 452)

50. Dengue virus has how many serotypes?


A. one B. two C. three D. four
Answer: D ( p 452)

51. The most common vector of Dengue virus is ( p 452)


A. Anopheles minimus B. Mansonia sp.
C. Aedes aegypti D. Aedes albopictus

52. A 1 year old girl has high fever for four days, coryza, hacking cough and poor
appetite. On PE, she has congested pharynx and Koplik’s spots. The causative
agent of this condition
A. is a DNA virus B. has three serotypes
C. belongs to the paramyxovirus family D. has double-stranded RNA
Answer: C ( P482)

53. Which of the following diseases is currently vaccine-preventable?


A. cytomegalic inclusion disease B. German measles
C. Dengue fever D. Hepatitis C
Answer: B ( p 485)

54. A 20 year old male developed fever, vomiting and poor appetite for four days
followed by jaundice. A diagnosis of viral hepatitis is entertained. The results
of the hepatitis profile are as follows: positive anti-HAV IgM, negative anti-
HBsAg, positive HBsAg, negative anti-HBc IgM. Which is correct?
A. The patient has acute Hepatitis A and is most likely a Hepatitis B carrier
B. He has acute Hepatitis B
C. He has acute Hepatitis C
D. He does not have acute viral hepatitis
Answer: A ( p 411)

55. The protective antibody following immunization against Hepatitis B is


A. anti-HBc IgM B. anti-HBc IgG C. anti-HBsAg D. anti-HCV
Answer: C ( p 415)

56. Which of the following agents of viral hepatitis is fecal-oral in transmission


and causes more severe disease in pregnant women?
A. Hepatitis A B. Hepatitis B C. Hepatitis C D. Hepatitis E
Answer: D ( p 408)

57. Bovine spongiform encephalopathy (mad cow disease) is caused by an agent which
has the following characteristic
A. double-stranded DNA B. single-stranded RNA
C. double-stranded RNA D. does not contain nucleic acid
Answer: D ( p 500)

58. Human influenza virus which undergoes antigenic shift and can cause pandemics
A. influenza A B. influenza B C. influenza C D. highly-pathogenic avian flu
Answer: A (p 463)

59. Fetal hydrops is a prominent complication of maternal infection with


A. rubella B. parvovirus B19 C. measles D. cytomegalovirus
Answer: B ( p 357)

60. An 8 month old girl developed high fever for three days but was otherwise playful.
On the fourth day, fever was gone but she had maculopapular rashes on the face
which rapidly spread all over the body. This common disease is caused by
A. measles virus B. human herpesvirus 6 C. parvovirus B19 D. rubella virus
Answer: B ( p389)

61. A 22 year old sex care worker wanted to be tested for human immunodeficiency
virus (HIV). What test would you initially recommend?
A. Western blot B. HIV Elisa C. CD4:CD8 ratio D. VDRL
Answer: B ( p 526)

62. HIV infection causes the ratio of CD4 to CD8 T cell ratio to fall. CD4 T cells are also
called
A. T cytotoxic cells B. cytolytic T cells C. T helper cells D. suppressor T
cells
Answer: C (p 521)

63. The major function of CD8 T lymphocytes is


A. to kill cells infected by intracellular pathogens like viruses
B. antibody-dependent cellular cytotoxicity
C. opsonization
D. phagocytosis
Answer: A (p 116)

64. A one and a half year old boy has recurrent bacterial pneumonia and middle ear
infection. An antibody deficiency is entertained. Antibodies are produced in large
quantities by
A. B lymphocytes B. T lymphocytes C. plasma cells D. mast cells
Answer: C (p 125)
65. Immunoglobulins aid phagocytosis because they can bind to phagocytes via
A. Fab receptors B. Fc receptors C. CD4 receptors D. C3b receptors
Answer: B ( p 125)

66. The following virus/viruses exhibit/s latency in nerve ganglia


A. smallpox virus B. chickenpox virus C. cytomegalovirus D. adenovirus
Answer: B (p 378)

67. Congenital viral infections are often asymptomatic. The most common cause of
congenital viral infection is
A. rubella B. cytomegalovirus C. varicella D. measles
Answer: B ( p 384)

68. A virus which infects B lymphocytes by attaching to the latter’s complement


Receptors
A. cytomegalovirus B. Epstein-Barr virus
C. Japanese B encephalitis virus D. HIV
Answer: B ( p 386)

69. Measles can be currently prevented by giving the following as part of routine
Immunization
A. live, attenuated vaccine B. killed virus vaccine
C. subunit vaccine D. gamma globulin
Answer: A ( p484)

70. A 1 year old infant had exposure to a caregiver with cavitary tuberculosis. Chest
x-ray was negative. Mantoux test done showed induration of 12 mm after 2 days.
The Mantoux tuberculin test result is an example of
A. Type I hypersensitivity B. Type II hypersensitivity
C. delayed-type hypersensitivity D. Arthus reaction
Answer: C ( p 130)

71. Mycobacterium tuberculosis is most commonly transmitted by


A. droplets B. fomites C. droplet nuclei D. vector
Answer: C ( p281)

72. The following are characteristics of M. tuberculosis


A. have minimal lipid content in their cell wall
B. cannot be grown on artificial media
C. retain carbolfuchsin dye after exposure to acid-alcohol
D. appear as round chains on Gram stain
Answer: C ( p25)

73. Rifampin, utilized as part of treatment for tuberculosis, acts by


A. inhibiting bacterial cell wall synthesis
B. inhibiting protein synthesis
C. binding to bacterial DNA gyrase
D. inhibiting folate synthesis
Answer: B ( p280)

74. Sexually transmitted virus which is the most common predisposing factor to cancer
of the uterine cervix in women
A. polyoma virus B. Papilloma virus
C. Herpes simplex type 2 D. Herpes simplex type 1
Answer: B ( p512)
75. Subacute sclerosing panencephalitis is a late complication of infection with this virus
A. Varicella B. Rubeola C. West Nile virus D. poliovirus
Answer: B ( p483)

Reference:
Jawetz, Melnick & Adelberg’s Medical Microbiology 22nd Edition, LANGE
** Jawetz, Melnick & Adelberg’s Medical Microbiology 23rd Edition, LANGE
MIRCROBIOLOGY AND PARASITOLOGY

MULTIPLE CHOICE QUESTIONS: Choose the letter of the most appropriate answer for each question.

Answer # questions page

B 1 Many of this type of bacteria do not possess superoxide dismutase 66


and catalase
A. obligate aerobes
B. obligate anaerobes
C. facultative anaerobes
D. microaerophiles

A 2 It is the period when the bacterium is assessing the nutrients present 53


in medium; the bacterium does not divide but there is an intense metabolic
activity. The phase of bacterial growth is
A. Lag
B. Log
C. Stationary
D. Decline

A 3 Which the best to sterilize scissors and other sharps? 60


A. Glutaraldehyde
B. Formaldehyde
C. Iodophor
D. Phenol

A 4 These are groups of genes that are clustered together in the DNA of some 98
bacterial species and which may enable them to invade a host
A. Pathogenicity island
B. Episomes
C. Transposons
D. Replicons

D 5 Which is true about the bacterial chromosome? 99


A. Contains histones that stabilize the DNA
B. Composed of a single pair of homologous chromosomes
C. Replicates in a very different manner from that of eukaryotic chromosomes
D. Contain single copy of each gene

B 6 A non-encapsulated strain of Hemophilus influenzae acquires the gene for 106


capsule production from a DNA extract of another encapsulated strain.
The event that has occurred is
A. Transformation
B. Conjugation
C. Transduction
D. Transcription

D 7 Which of the following statements concerning the bacterial structure is correct? 28


A. Flagella mediate the interaction of bacteria with mucosa epithelium.
B. Flagella are non-antigenic in humans because they closely resemble
human flagella in chemical composition.
C. Polysaccharide capsules are non-antigenic.
D. Both gram-negative rods and cocci have lipopolysaccharide in their cell wall.

C 8 What is the substance primarily responsible for the structural integrity of 22


the bacterial cell wall and is susceptible to lysozyme?
A. Cytoplasmic membrane
B. Teichoic acid
C. Peptidoglycan
D. Lipopolysaccharide

D 9 A hospitalized patient with dysuria and suprapubic pain is treated with 164
ciprofloxacin. What is the mechanism of action of this antibiotic?
A. It inhibits DNA-dependent RNA polymerase
B. It inhibits protein synthesis by binding to the 30S ribosomal subunit
C. It inhibits protein subunits by binding to the 50S ribosomal subunit
D. It inhibits topoisomerase II (DNA gyrase)
2

C 10 Which bacterial structure may carry gene for antibiotic resistance? 98


A. Mesosome
B. Periplasm
C. Plasmid
D. Outer membrane

A 11 What is the predominant indigenous flora of the colon? 199


A. Anaerobic, gram-negative, non-spore-forming bacteria
B. Anaerobic, gram-positive, non-spore-forming bacteria
C. Aerobic, gram-negative, non-spore-forming bacteria
D. Aerobic, gram-positive, spore-forming bacteria

C 12 Which of the following is the true regarding the bacterial cell? 35


A. He motility of bacteria is due to flagella, axial filaments or cilia
B. The ability to colonize cell surfaces is dependent on toxic production
C. The ability to survive in soil for a long period of time is dependent
on the production of endospores
D. It has the same ribosomes as that of a eukaryotic cells

D 13 Which of the following is the counterpart of mitochondria in a bacterial cell? 17


A. Cell wall
B. Endospores
C. Ribosomes
D. Cell membrane

A 14 Which of the following agents possess either DNA or RNA? 2


A. Viruses
B. Prions
C. Bacteria
D. Fungi

B 15 Most pathogenic microorganism grow best at the temperature range of about __. 65
A. 15-20ºC
B. 30-37ºC
C. 50-60ºC
D. 80-100ºC

C 16 A 22-year-old woman came in due to meningococcemia. Few hours after 155-156


the admission, the patient experienced disseminated intravascular coagulation
(DIC). The patient shock was due to what component of the causative agent?
A. Cell wall
B. Capsule
C. Lipopolysaccharide
D. Endospores

C 17 Which one of the following antimicrobial agents acts on microorganisms by 189


inhibiting nucleic acid synthesis?
A. Streptomycin
B. Cefalexin
C. Ciprofloxacin
D. Erythromycin

D 18 A newborn was diagnosed with neonatal meningitis. The causative agent 199
was found out to be isolated in the vagina of the mother. Which
one of the following microorganisms can be a part of the normal vaginal
flora of the mother and can cause neonatal meningitis?
A. Mycoplasma pneumoniae
B. Staphylococcus aureus
C. Escherichia coli
D. Streptococcus agalactiae

C 19 Which of the following is the “C3 activation unit” in the classical complement 136
pathway?
A. C1q
B. C3
C. C4b, C2a
D. C5, C6, C7, C8 and C9
3

C 20 The classic pathway of the complement system is activated when__ 136


A. The Fab portion of IgM binds to the C1s of the complement pathway
B. The Fc portion of IgE binds to the C1q of the complement pathway
C. The hypervariable regions of the heavy and light chains binds to the
membrane of the bacterial cell
D. The mannose binding lectin stimulates mannose associated serine protease

D 21 Antigen-presenting cells that activate helper T cell must express which one of 139
the following on their surfaces?
A. IgE
B. Gamma interferon
C. Class I MHC antigens
D. Class II MHC antigens

D 22 Increased susceptibility with Hepatitis virus and Candida albicans would 138
be expected with a deficiency of which of the following cells?
A. PMN
B. Eosinophils
C. B cells
D. T cells

B 23 In Type I hypersensitivity, the predominant antibody that is elicited is: 140


A. IgA
B. IgE
C. IgG
D. IgM

D 24 Contact dermatitis is an example of: 141


A. IgE mediated
B. Complement mediated
C. IgG mediated
D. Cell mediated

A 25 This immunoglobulin is important in preventing diarrhea in the newborn 129


A. IgA
B. IgM
C. IgE
D. IgD

C 26 Main component in secondary response. 135


A. IgA
B. IgM
C. IgG
D. IgD

C 27 At birth, the main source of Hematopoetic Stem Cell is the 126


A. Liver
B. Spleen
C. Bone Marrow
D. Thymus

C 28 Which method will you see to detect antigens directly from tissue? 142
A. ELISA
B. Ouchterlony technic
C. Immunofluorescence
D. Complement fixation

B 29 Which is true about humoral immunity and antibodies? 128


A. Antibodies found on the surface of B cells are IgA
B. IgG has two bindings sites for an antigen
C. IgM is the main protection of the newborn up to 6 (six) months
D. The Fab region of IgE binds to mast cell receptor

A 30 The following is true of T cells:


A. They are responsible for long term memory
B. T helper2 cells stimulate B cells to become plasma cells
C. All “T cells” bear CD4
D. Some T cells can serves as antigen presenting cells
4

C 31 Which of the following complement cascade does not involve C3b? 136
A. C5 convertase of the alternative pathway
B. C5 convertase of the classic pathway
C. C3 convertase of the alternative pathway
D. C3 convertase of the classic pathway

D 32 Which of the following cells is a part of the innate immune response? 123
A. B-lymphocytes
B. T-helper lymphocytes
C. T-cytotoxic lymphocytes
D. Macrophages

A 33 Which of the following is LEAST associated with Enterohemorrahgic E. coli? 253


A. fermentation of sorbitol
B. strain 0157:H7
C. undercooked hamburgers
D. hemolytic-uremic syndrome

B 34 Which of the following is true about cholera and its causative agent? 271
A. A secretory diarrhea due to increases in cGMP in the intestinal cells
B. The stool is described as ‘rice water-like’
C. The causative agent is non-motile, nonfermentative
D. Grows poorly in medium with alkaline pH

B 35 The appearance of moistpale papules or condylomas in the anogenital 332


area is characteristic of
A. Primary syphilis
B. Secondary syphilis
C. Tertiary syphilis
D. T. pallidum

D 36 A 4-year old Asian child develops an infection with Chlamydia trachomatis. 360
How does infection with this organism cause blindness?
A. Retinal detachment
B. Cataract formation
C. Hemorrhage of the anterior chamber
D. Scarring of the cornea

D 37 Which is associated with atherosclerosis? 364


A. K. pneumoniae
B. C. trachomatis
C. S. pyogenes
D. C. pneumoniae

D 38 Which of the following is LEAST effective against M. pneumoniae infection? 345


A. Erythromycin
B. Tetracycline
C. Kanamycin
D. Vancomycin

A 39 The only disease that can be diagnosed by demonstration of spirochetes 335


in peripheral blood smear
A. Relapsing fever
B. Leptopirosis
C. Pinta
D. Lyme disease

A 40 Which of the following is LEAST associated with Helicopter pylori? 275


A. It requires at least 2% NaCl for growth (halophilic)
B. It requires increased carbon dioxide tension and decreased oxygen tension
C. It is sensitive to the acidity of the gastric juice
D. Most commonly diagnosed by the demonstration of urease production

A 41 In which type of diarrhea is the use of antibiotics not recommended? 259


A. Salmonellosis
B. Campylobacter infection
C. Cholera
D. Shigellosis
5

C 42 Which causes urinary tract infection characterized by alkaline urine and 254
formation of urinary calculi?
A. Shigella dysenteriae
B. Escherichia coli
C. Proteus mirabilis
D. Enterobacter cloacae

C 43 Which is LEAST acceptable as specimen for the diagnosis of anaerobic infection 309
A. Transtracheal aspirate
B. Suprapubic aspirate
C. Vaginal swab
D. Sample of tissue from infected site

B 44 Which of the following microorganisms takes up the color of Crystal violet upon 208
properly performed Gram staining?
A. Neisseria gonorrheae
B. Clostridium perfringens
C. Escherichia coli
D. Pseudomonas sp.

D 45 Which of the following is a Toxoid vaccine? 215


A. Pnuemococcal
B. Measles
C. Varicella
D. Diphtheria

A 46 The most infectious stage in pertussis is: 283


A. Catarrhal stage
B. Prodomal stage
C. Paroxysmal stage
D. Convalescent stage

A 47 Regarded as a TB marker in the diagnosis of tuberculous effusion: 407


A. Gamma interferon
B. AFB smear
C. Caseation necrosis
D. PPD positivity

B 48 Presence of immunity to C. diphtheriae is indicated by: 215


A. Positive elek test
B. Negative elek test
C. Positive schick test
D. Negatice schick test

B 49 Which of the following is MOST likely acquired from caves? 642


A. Blastomycosis
B. Histoplasmosis
C. Coccidioidomycosis
D. Paracoccidioidomycosis

C 50 Asteroid bodies are seen tissue infected with 634


A. P. verrucosa
B. P. boydii
C. S. schenckii
D. L. loboi

C 51 The agents of chromomycosis are seen in infected tissues as 635


A. Intracellular yeast
B. Granules
C. Sclerotic bodies
D. Cigar-shaped bodies

C 52 Which of the following is NOT dimorphic? 636


A. P. boydii
B. H. capsulatum
C. S. schenckii
D. C. immitis
6

B 53 The agent of blastomycosis are seen in infected tissues as 642


A. Intracellular yeast
B. Single budding yeast
C. Multiple budding yeast
D. Cigar-shaped bodies

D 54 The physician closely monitored the patient’s serum creatinine and 653
ion levels. The patient is taking certain anti-fungal drug. What possible
antifungal agent may affect the renal function of this patient?
A. Ketoconazole
B. Terbinafine
C. Griseofulvin
D. Amphotericin B

B 55 The primary site of echovirus multiplication in the human host is 487


A. The muscular system
B. The alimentary tract
C. The anterior horn cells
D. The respiratory system

D 56 Which of the following properties of polioviruses is not shared by rhinovirus? 489


A. Icosahedral symmetry
B. Resistance to lipid solvents
C. Naked virus
D. Stability at acid pH (pH3.0)

D 57 Segmented, double-stranded RNA genome that is a common cause of 505-507


viral gastroenteritis?
A. Astrovirus
B. Norwalk virus
C. Calicivirus
D. Rotavirus

B 58 Diagnostic of Congenital rubella is the demonstration of rubella antibodies like: 568


A. Maternal IgM
B. Neonatal IgM
C. Neonatal IgA
D. Maternal IgA

D 59 A viral cause of nephropathy in immunocompromised patient is 598-599


A. Human papillomavirus, all types
B. Human papillomavirus, low-risk types
C. Hepatitis C
D. Polyomavirus BK

D 60 Which of the following individuals may be at increased risk of acquiring and 616-617
HIV infection?
A. A pregnant mother with a seafarers husband
B. A secretary at an AIDS institute
C. A doctor with a colleague that is HIV positive
D. A male celebrity who has multiple sexual partner including a prostitute

D 61 Type of papillomavirus that is commonly associated with cervical carcinoma 601


A. Type 1
B. Type 6
C. Type 9
D. Type 18

C 62 Progressive Multifocal leukuencepalopathy is associated with a virus that 597-599


is described as __.
A. ssDNA, icosahedral, naked
B. dsDNA, helical, enveloped
C. dsDNA, icosahedral, naked
D. ssRNA, segmented, naked
7

C 63 Which of the following virus that can enter a skin through abrasions? 599
A. Adenovirus
B. Polyomavirus
C. Papillomavirus
D. Rhinovirus

D 64 A 6-year old child had recently had fifth disease. Her 43-year old mother 414-416
subsequently developed a rash and arthalgia. Which one of the following
best describes the causative agent?
A. It has the same size as poxvirus
B. It has a helical symmetry
C. Inactivated by ether
D. A single-stranded DNA genome

D 65 HIV is a retrovirus. A retrovirus is a 605


A. Cause tumors in mouse
B. Contain a DNA genome
C. Cause rapidly progressive neurological disease
D. Multiplies in the nucleus

C 66 Most common cause of post transfusion hepatitis 466


A. HAV
B. HBV
C. HCV
D. HDV

B 67 Site of latent infections is at the sacral ganglia: 433-434


A. HSV 1
B. HSV 2
C. EBV
D. CMV

C 68 Which of the following viruses can establish a latent infection? 371


A. Poxviruses
B. Rubella viruses
C. Herpesviruses
D. Coronaviruses

D 69 A 23-year-old medical student experienced fever, sore throat, and 429


lymphadenopathy. These was accompanied by lymphocytosis and atypical
cells in the blood smear. True statement regarding the causative agent
A. the major target cell of the virus is the T lymphocyte
B. the agent multiplies in the cytoplasm
C. it can be transmitted thru direct contact with the lesion
D. it possess a double-stranded DNA genome

B 70 The following hepatitis B marker is indicative of active viral replication 468


A. HBsAg
B. HBeAg
C. Anti- HBs
D. Anti- HBe

B 71 The habitat is the large intestine. 104


A. Entamoeba gingivalis
B. Entamoeba histolytica
C. Giardia lamblia
D. Naegleria fowleri

C 72 The infective stage of Entamoeba histolytica to man has 103-104


A. Pseudopodia
B. Ingested red blood cells
C. Bull’s eye karyosome
D. Pointed chromatoidal bodies
8

B 73 The most commonly involved area in Amoebiasis is the 108-109


A. Brain
B. Cecum
C. Liver
D. Recto-sigmoid region

A 74 There is no increase in the number of nuclei of 215


A. Balantidium coli
B. Entamoeba coli
C. Entamoeba
D. Giardia lamblia

A 75 Tissue invasion is relatively slow and tends to stimulate granuloma formation 142
A. Acanthamoeba castellanii
B. Balantidium coli
C. Entamoeba histolytica
D. Naegleria fowleri

B 76 Mosquito vector of Malaria in the Philippines is 164


A. Aedes aegypti
B. Anopheles flavirostris
C. Culex quinquefasciatus
D. Mansonia annulata

A 77 There is secondary exo-erythrocytic phase in 177


A. Benign tertian malaria
B. Malignant tertian malaria
C. Quartan malaria
D. Sub-tertian malaria

C 78 The duration of the Erythrocytic-Schizogonic Cycle is 72 hours. 184-191


A. A. Benign tertian malaria
B. Malignant tertian malaria
C. Quartan malaria
D. Sub-tertian malaria

A 79 The gametocytes of the organism are described as banana or 188-190


crescent-shaped.
A. Plasmodium falciparum
B. Plasmodium malariae
C. Plasmodium ovale
D. Plasmodium vivax

C 80 Produces rectal prolapse in heavy infection especially among children 243


A. Capillaria philippinensis
B. Trichinella spiralis
C. Trchuris trichiura
D. Ascaris lumbricoides

B 81 Bachman intradermal test & Xenodiagnosis are use for the diagnosis of 239
infection cause by
A. Capillaria philippinensis
B. Trichinella spiralis
C. Trchuris trichiura
D. Ascaris lumbricoides

A 82 The most common cause of Greeping Eruption or Cutaneous Larva Migran 281
A. Ancytostoma braziliense
B. Enterobius vermicularis
C. Strongyloides stercoralis
D. toxocara canis

D 83 The most common cause of Visceral Larva Migran 325


A. Ancylostoma braziliense
B. Enterobius vermicularis
C. Strngyloides stercolaris
D. Toxocara canis
9

D 84 Produces autoinfection to man 256


A. Ancylostoma duodenale
B. Ascaris lumbricoides
C. Necator americanus
D. Strongyloides stercolaris

B 85 What parasite produces Iron deficiency anemia to man? 284


A. Ascaris lumbricoides
B. Necator americanus
C. Strongyloides stercolaris
D. Trichuris trichiura

B 86 Associated with Noctrunal Pruritus Ani. 304


A. Ascaris lumbricoides
B. Enterobius vermicularis
C. Necator americanus
D. Trichuris trichiura

B 87 Graham Scotch tape technique is used for the diagnosis of the infection 305
caused by
A. A. Ascaris lumbricoides
B. Enterobius vermicularis
C. Necator americanus
D. Trichuris trichiura

A 88 What parasite sucks and ingests blood? 284


A. Ancylostoma duodenale
B. Ascaris lumbricoides
C. Enterobius vermicularis
D. Trichuris trichiura

A 89 What parasite shows Tramway sign in the X-ray? 317


A. Ascaris lumbricoides
B. Enterobius vermicularis
C. Necator americanus
D. Trichuris trichiura

D 90 The infective stage of the parasite is found in crabs or crayfishes 465


A. Clonorchis sinensis
B. Echinostoma ilocanum
C. Fasciola hepatica
D. Paragonimus westermani

A 91 What parasite is associated with neoplasm of the biliary duct or cancer 477
of the liver?
A. Clonorchis sinensis
B. Fasciola hepatica
C. Opisthorchis felineus
D. Paragonimus westermani

D 92 What parasite produces infection that stimulates tuberculosis? 467-468


A. Clonorchis sinensis
B. Fasciola hepatica
C. Opisthorchis felineus
D. Paragonimus westermani

A 93 What is the infective stage of Schistosoma to man? 417


A. Cercariae
B. Embryonated egg
C. Metacerciae
D. Miracidium

D 94 Molluscan host of Schistosoma in the Philippines is 416


A. Australorbis mystax
B. Biomphalaria alexandrina
C. Coxiella burnetii
D. Oncomelania quadrasi
10

C 95 Laboratory procedure that makes use of the egg in the test is 770
A. CFT
B. CHR
C. COPT
D. Casoni test

D 96 The infective of Diphyllobothrium latum to man to produce adult 497


infection is the
A. Cysticercoid larva
B. Cysticercus larva
C. Procercoid larva
D. Sparganum larva

C 97 Produces Ocular cysticercosis to man 518


A. Dipylidium caninum
B. Taenia saginata
C. Taenia solium
D. Hymenolepis nana

D 98 Lateral uterine branches in gravid proglottid are counted for diagnosis of 522
infection cause by
A. Diphyllobothrium latum
B. Dipylidium caninum
C. Hymenolepsis nana
D. Teania saginata

B 99 Taenia saginata is associated with 521


A. Cat
B. Cattle
C. Dog
D. Pig

B 100 Ultrasound and MRI usually help in the diagnosis of the infection cause by 532
A. Dipylidium caninum
B. Echinococcus granulosus
C. Hymenolepis diminuta
D. Hymenolepis nan

REFERENCE
rd
Jawetz 23 edition
th
Clinical Parasitology 9 edition
1

Board Questions in Microbiology and Parasitology


Micro-Para
Subject: Microbiology (60)
Topic: Immunology
Instruction: Choose the best answer.

1. Carlo has leukemia; unfortunately he was exposed to HBsAg positive


blood. The best immediate management which can be given to him is to
give:
A. recombinant Hep B vaccine
B. passive immunization
C. anti-viral drugs
D. vitamins
Answer: B
Reference: Medical Microbiology by Jawetz, 23rd ed. p.119
MPL: 1

2. Mario, a 45-year-old executive is a sickly guy. He always complains of


coughs and colds. He used to be a smoker but he had stopped since he
was in his early thirty’s. His doctor said that he is often attacked by
organisms which attack/ attach to the mucous membranes of his
respiratory tract. These organism then do which of the following actions
against its host:
A. breakdown of IgA by producing proteases
B. evasion of phagocytosis by means of their capsule
C. excretion of hyaluronidase
D. neutralization of lysozymes
Answer: A
Reference: Medical Microbiology by Jawetz, 23rd ed. p. 121
MPL: 0.25

3. A child who is breast fed is not usually prone to diarrhea, this is probably
because of:
A. the presence of hydrolytic enzymes in the saliva which kills most
bacteria
B. the acidity of the stomach
C. presence of proteolytic enzymes and active macrophages in the in
the small intestines
D. all of the above
Answer: D
Reference: Medical Microbiology by Jawetz, 23rd ed. p.122
MPL: 1

4. Luna was diagnosed to have a starting skin cancer, which somehow


spontaneously resolved. Aside from a possible miracle the doctor said it
could be due to the Luna’s natural killer cells. Which is NOT a
characteristic of natural killer (NK) cells)?
A. morphologically related to T cells
B. they can lyse malignant cells
C. they resemble large granular lymphocyte
D. all of the above
Answer: D
Reference: Medical Microbiology by Jawetz, 23rd ed. p. 125
MPL: 1

5. Martin suffers from runny nose, itchy eyes and frequent sneezing
especially during the months of March to May and November to January.
He probably has:
A. Type I hypersensitivity
B. Type II hypersensitivity
C. Type III hypersensitivity
D. Type IV hypersensitivity
2

Answer: A
Reference: Medical Microbiology by Jawetz, 23rd ed. p. 140
MPL: 1

6. Vivian was rushed to the hospital for shortness of breath. She gave a
history of taking penicillin for her sore throat two days prior to admission.
She gave no history of allergy to penicillin. Laboratory test done on her
blood showed the presence of antibodies which hemolyses her own RBC.
She is probably suffering from:
A. Type I hypersensitivity
B. Type II hypersensitivity
C. Type III hypersensitivity
D. Type IV hypersensitivity

Answer: B
Reference: Medical Microbiology by Jawetz, 23rd ed. p.141
MPL: 1

7. Athena was born December 2004. She has an older brother Bitoy who got
sick of chicken pox when Athena was one month old. Ikay, another older
sister of Athena got sick of chicken pox while she did not. This is
because of:
A. Maternal IgA
B. Athena’s IgA
C. Maternal IgG
D. Athena’s IgG

Answer: C
Reference: Medical Microbiology by Jawetz, 23rd ed. p. 128
MPL: 0.5

8. Isabel becomes a different person during the months of December. In


these months she is grouchy because of hay fever. Her doctor explained
that she is allergic to pollen and the major culprit is the so-called IgE,
which is a homocytotropic immunoglobulin which binds to all of the
following cells bellow EXCEPT:
A. eosinophils
B. mast cells
C. basophils
D. neutrophils

Answer: D
Reference: Medical Microbiology by Jawetz, 23rd ed. p 130
MPL: 0.25

9. Betty has colds almost every three months. But she is not bothered
because she noticed that even without treatment she gets better The self-
limiting nature of her colds which is often of viral etiology is partly
attributed to:
A. Gamma interferon
B. Alpha interferon
C. Beta interferon
D. All choices are correct except A

Answer: D
Reference: Medical Microbiology by Jawetz, 23rd ed. p. 125
MPL: 0.25

10. Mike was sick of pneumonia, for which he almost died. Before his
discharge from the hospital he asked the doctor to explain to him his
illness and how he was able to recover. The doctor talked of the germs
3

having slimly materials around them. That it was necessary for some
substance to coat the germs so that the scavenger’s cells in the body can
eat them up. The doctor was referring to opsonization. Which of the
following substances is involved in opsonization?
A. C3a
B. C3b
C. C2a
D. C2b
Answer: B
Reference: Medical Microbiology by Jawetz, 23rd ed. p. 137
MPL: 0.25

11. A 7-year-old girl was brought to the derma section of UPHR medical
center for
swollen, weeping earlobes with some signs of crusting. One earlobe has
an almost embedded tiny earring. The doctor said the girl was probably
allergic to the nickel component of the fancy earrings. This condition is
also termed as:
A. Contact hypersensitivity
B. Tuberculin hypersensitivity
C. Atopic hypersensitivity
D. Immune complex hypersensitivity

Answer: A
Reference: Medical Microbiology by Jawetz, 23rd ed. p. 141
MPL: 0.25

12. Nonong was an OFW from Puerto Rico. He came home because he was
diagnosed to have syphilis. He consulted a local doctor for a second
opinion.
The doctor scrapped off the base of the ulcer found in his buccal
mucosa. He
explained to Jojo that he will add to the specimen a substance which he
called
antibody to which will be attached a certain dye which will emit light
under a
“ dark microscope “.. This immunologic diagnostic test is probably:
A. Immunoblotting
B. Immunofluorescence
C. ELISA
D. Radioimmunoassay

Answer: B
Reference: Medical Microbiology by Jawetz, 23rd ed. p. 142
MPL: 1

13. Jake had typhoid fever. When seen by the doctor he has already on the third
weeks and had started to feel better. If you can enter Jake’s body which of
the following item below would show that he had achieved an adaptive
immune response against typhoid bacilli?
A. physical barriers
B. chemical barriers
C. clonal expansion of effector cells
D. phagocytosis

Answer: C
Reference: Medical Microbiology by Jawetz, 23rd ed. p. 143
MPL: 1
4

14. Doy was diagnosed to have an infectious disease of unknown etiology.


Examination of his blood showed that the complement levels are normal. In
infections, one defense against the agent is the activation of the complement.
What then among the item below is the best trigger for the of complement
activation ?
A. IgG
B. Mannose-containing bacterial glycolipids
C. Microbial surfaces
D. IgM-antigen immune complexes

Answer: D
Reference: Medical Microbiology by Jawetz, 23rd ed. p. 147
MPL: 0.25

15. A 13-year-old boy from Ilocos tested negative in the lepromin test. He
presents with multiple , erythematous, anesthetic lesions all over the body.
The skin test shows which type of hypersensitivity:
A. Type I Hypersensitivity
B. Type II hypersensitivity
C. Type III hypersensitivity
D. Type IV hypersenitivity
Answer: C
Reference: Medical Microbiology by Jawetz, 23rd ed. p. 141
MPL: 1

16. Roy was found to have antibodies to AIDS by the ELISA method. To confirm
the diagnosis, antigens of the virus must be detected in his blood. Which of
the following methods will identify the HIV antigen mix in a complex mixture
of proteins :
A. Western blotting
B. Southern blotting
C. Eastern blotting
D. Northern blotting
Answer: A
Reference: Medical Microbiology by Jawetz, 23rd ed. p. 143
MPL: 1

Subject: Microbiology (Virology)


Instruction: Choose the correct answer.
17. The human vaccine for this viral infection is made of killed virus grown in
human diploid fibroblasts or chicken fibroblasts:
A. Rubella
B. Rabies
C. Jap B encephalitis
D. Measles
Answer: B
Reference: Medical Microbiology by Jawetz, 23rd ed. p. 410
MPL: 0.25

18. Luisa is an absent-minded medical intern. She was asked by the resident to
get some influenza vaccines from the refrigerator in the call room and to bring
them to the OPD for immunization of the old nurses in the section. Luisa
followed the instructions. Unfortunately she did not find the resident so she
placed the vaccine on the table with a short note. After 2-4 hours, the
resident came and found the vaccines. Influenza is a killed viral vaccine, all
of the following are its disadvantages EXCEPT:
A. high stability at room temperature
B. shorter duration of immunity
C. poor CMI produced
D. no IgA produced
5

Answer: A
Reference: Medical Microbiology by Jawetz, 23rd ed. p. 410
MPL: 1

19. Allan was bitten by his dog in the lower leg. The wound is not deep but it
bled. In the decision to give postexposure prophylaxis the following are
considered except which of the following :
A. manner of attack
B. nature of biting animal
C. severeity of the bite
D. none of the choices
Answer: D
Reference: Medical Microbiology by Jawetz, 23rd ed. p. 580
MPL: 1

20. Laura was in her 2nd month of pregnancy when she tested positive for Rubella
antibodies. Inspite of this she decided to continue with the pregnancy. She
delivered the baby 4 weeks earlier than the expected date. A t a glance the
baby seemed normal. She asked her doctor what could be the possible effect
if her baby got congenital rubella. Which of the following could explain any
abnormality that may be later seen in Laura’s baby ?
A. infected cells in the baby have reduced growth rate
B. hypoplastic organs results in abnormalities in the embryonic cells
which have
reduced growth rate
C. structural anomalies due to hypoplastic organs
D. all of the above
Answer: D
Reference: Medical Microbiology by Jawetz, 23rd ed. p. 567
MPL: 0.25

21. A baby born with congenital rubella was found to be deaf. Defects in rubella
are grouped into broad categories, i.e temporary, developmental, permanent.
Deafness is under :
A. temporary
B. developmental
C. permanent
D. none of the choices

Answer: C
Reference: Medical Microbiology by Jawetz, 23rd ed. p. 568
MPL: 0.33

22. Ana is getting married and thus was advised to get her Rubella vaccine
shot. Which of the following is/are true of the rubella vaccine ?
A. it is live attenuated vaccine
B. it is given to prevent congenital rubella infection
C. vaccinated children pose no threat to mothers who are susceptible and
pregnant
D. all of the above

Answer: D
Reference: Medical Microbiology by Jawetz, 23rd ed. p. 567
MPL: 1

23. Jay a 9-month old baby is rushed to the ER for fever and persistent
cough. On examination, rales are heard on his left chest and an infiltrate
6

in the left lung is seen on the chest x-ray film. The diagnosis given is
pneumonia. Which of the following viruses is the most likely cause?
A. Rhinovirus
B. Respiratory syncytial virus
C. Adenovirus
D. Coronavirus

Answer: B
Reference: Medical Microbiology by Jawetz, 23rd ed. p. 402
MPL: 1

24 In this viral infection, humans are dead-end hosts, thus virus shedding
does
not occur. A. measles
B. Rabies
C. influenza
D. chicken pox

Answer: B
Reference: Medical Microbiology by Jawetz, 23rd ed. p. 198
MPL: 1

25. A ten-year-old girl complained of difficulty and pain in swallowing. It was


accompanied by runny nose and fever. Upon examination, the intern in
the OPD noted slightly swollen tonsils and erythematous pharynx. No
exudates were noted on the tonsillar surface. The diagnosis given was
upper respiratory infection . The most likely etiologic agent for this case
is:
A. adenovirus
B. Respiratory syncytial virus
C. Rhinovirus
D. Coronavirus
Answer: A
Reference: Medical Microbiology by Jawetz, 23rd ed. p. 402
MPL: 0.25

26. Ramon, a 4 year old boy was seen by a pediatrician for complains of low-
grade fever, cold, and rash on the face which showed a typical “
slapped cheek ” appearance. Complains of flu-like symptoms such as
malaise and myalgias were also noted. The pediatrician gave a diagnosis
of erythema infectiosum of the fifth disease which is caused by which of
the following viruses:
A. Hanta virus
B. B19 parvovirus
C. Herpes simplex
D. Coxsackie viruses
Answer: B
Reference: Medical Microbiology by Jawetz, 23rd ed. p. 414
MPL: 0.25

27. Last summer, a community hospital in Laguna, recorded 25 cases of


pharyngoconjunctivitis among the grade five students from a private
school in the nearby municipality. It was learned that all of them attended
7

a summer camp, which lasted for 2 weeks. Which of the following


adenovirus serotypes are most likely involved?
A. types 3 and 7
B. types 1 and 70
C. types 40 and 45
D. types 8 and 10
Answer: A
Reference: Medical Microbiology by Jawetz, 23rd ed. p. 425
MPL: 0.25

28. A 2 ½-year-old boy was noticed by his mother to have less appetite for
solid food, but when given ice cream and soft drinks he was quite happy.
This was unusual because he likes to eat and play with solid foods. The
mother examined him and found that he has some swollen nodules in his
neck. Suspecting something in the mouth, she examined the mouth and
found vesicular lesions in the buccal mucosa. This is probably caused by
:
A. Candida albicans
B. Streptococcus pyogenes
C. Measles virus
D. Herpes simplex virus
Answer: D
Reference: Medical Microbiology by Jawetz, 23rd ed. p. 435
MPL: 0.5

29. Yoyong, a 15-year-old boy acquired chicken pox. When vesicular lesions
became evident, he was “isolated” by his mother from the rest of the
family by confining him to his room. After a week , his pocks subsided
with some lesions already developing scabs. It was at this time that two
other siblings started showing the same signs and symptoms he had.
Which of the following item below best describes the disease/etiologic
agent of involved in Yoyong’s case?
A. the virus infects only humans
B. the disease is highly contagious
C. both are correct
D. neither one is correct
Answer: C
Reference: Medical Microbiology by Jawetz, 23rd ed. p.438
MPL: 0.25

30. Idang is due two deliver in two weeks time. Unfortunately she had
premature labor and deliverered a stillborn baby boy. The baby was
autopsied and the one finding was the presence of cells which look like
“owls eye” i.e. cells with large intranuclear inclusion bodies. The most
probable diagnosis is:
A. Congenital measles
B. Congenital rubella
C. Congenital chicken pox
D. congenital cytomegalovirus infection
Answer: D
Reference: Medical Microbiology by Jawetz, 23rd ed. p. 443
MPL: 0.25

31. JR a macho dancer from the red district did not report for work for almost
three days. RJ his friend became worried and went to his place. He
found JR with fever, headache, and feels weak and tired. They went to
the clinic and was found to have enlarged lymph nodes and spleen and
slightly icteric conjunctiva. Blood exam showed large, atypical
lymphocytes. The most probable diagnosis is: A. Cytomegalovirus
infectious mononucleosis
8

B. Infectious mononucleosis
C. Herpes simplex infection
D. Mumps

Answer: B
Reference: Medical Microbiology by Jawetz, 23rd ed. p. 447
MPL: 0.33

32. A 28-year-old woman has recurrent genital herpes. Which of the following
about genital herpes infection is true ?
A. cannot be transmitted in the absence of apparent lesions
B. reactivation of latent virus during pregnancy poses no threat to
the
newborn
C. can be caused by either herpes simplex type 1 or type 2
D. recurrent episodes are more severe than the primary infection

Answer: C
Reference: Medical Microbiology by Jawetz, 23rd ed. p. 435
MPL: 0.25

33. MR consulted a dermatologist for the small, pink, wart-like lesions on her
face, arms, back and buttocks. The diagnosis given was moluscum
contagiosum infection. Which of the following about this is NOT true?
A. virus has a wide range of host
B. transmission is by direct and indirect contact
C. it is considered a sexually transmitted disease
D. the virus has a DNA genome

Answer: A
Reference: Medical Microbiology by Jawetz, 23rd ed. p. 463
MPL: 0.25

34.Motong is a leukemia patient. He was admitted for post transfusion


hepatitis. Test showed the etiologic agent to be flavivirus. This virus is:
A. Hepatitis A
B. Hepatitis B
C. Hepatitis C
D. Hepatitis E

Answer: C
Reference: Medical Microbiology by Jawetz, 23rd ed. p.468
MPL: 1

35. Ferdie was a diagnosed carrier of Hepatitis B. In one of his follow-ups in


the clinic, anti-HBe antibodies were detected in his serum. This means
that:
A. he is now developing resistance
B. his HB virus titer is now low
C. it has no meaning
D. a mistake was committed because HBs Ag and anti-HBe do not
exist
together in the blood

Answer: B
Reference: Medical Microbiology by Jawetz, 23rd ed. p. 468
MPL: 0.25
9

36. Lola, a middle-aged woman complained of acute onset fever, and nausea.
Her eyes were “yellow” and she was surprised to see her urine to be dark
colored. She consulted a private clinic and the laboratory test done on her
blood was positive for HAV IgM antibody. What will you tell Lola if you
are the physician?
A. That she acquired the disease through sexual contact
B. that probably it was transmitted to her when she had blood
transfusion
C. to be very careful, because she can transmit the infection to
other members of her family by person-to-person spread
D. that she might develop liver carcinoma in the future
Answer: C
Reference: Medical Microbiology by Jawetz, 23rd ed. p. 483
MPL: 1

37 A young executive from Makati was hospitalized for myopericarditis with


mild congestive heart failure that increases over several weeks. The
attending physician gave a diagnosis of Coxsackie B5 infection. Other
than his conditions what other syndrome/s is/ are associated with this
virus?
A. herpangina
B. aseptic meningitis
C. acute hemorrhagic conjunctivitis
D. all of the above
Answer: D
Reference: Medical Microbiology by Jawetz, 23rd ed. p. 495
MPL: 0.25

38. After getting his first salary, Bitoy went to the seaside restaurant and
feasted on seafoods, particularly raw oyster with his favorite beer. After 24
hours, he became ill with sudden onset of vomiting, diarrhea and
headache. He consulted a private clinic and the physician, after thorough
examination, ruled out a bacterial etiology. To him the most probable viral
etiologic agent is:
A. Astrovirus
B. Norwalk virus
C. Rotavirus
D. Hepatitis A virus
Answer: B
Reference: Medical Microbiology by Jawetz, 23rd ed. p. 512
MPL: 0.25

39. A 45-year-old woman from a ranch in Mindanao, complained of fever,


malaise and sore throat, followed by nausea, vomiting and then sudden
stupor. She was diagnosed to be suffering from eastern equine
encephalitis. The control of this disease in humans could be accomplished
by eradication/ control of:
A. ticks
B. birds
C. mosquitoes
D. sandflies
Answer: B
Reference: Medical Microbiology by Jawetz, 23rd ed. p. 522
MPL: 0.25

Subject: Microbiology (Mycology)


40. DJ went to Boracay last summer. He had a good tan except on some
portion of his neck down to his upper back which showed minimal scaling .
After a week, the tan portion of his skin started to peel. To his surprise
10

the “ untanned” portion of his skin seemed to have become bigger. His
skin started to return to its normal color but the “ untanned” portion
seemed to stand out this time looking more whitish that his normal light
brown skin. The most likely diagnosis is :
A. leprosy
B. superficial mycoses
C. avitaminosis
D. none of the choices
Answer: B
Reference: Medical Microbiology by Jawetz, 23rd ed. p. 628
MPL: 0.25

41. DJ consulted a dermatologist for the whitish discoloration which showed


after his Boracay escapade. The doctor made a KOH mount of the skin
scrapings and found, short, stout, angular hyphal elements. The skin
lesions also fluoresce under the Wood’s lamp. The most likely etiologic
agent is:
A. Tinea corporis
B. Pityriasis versicolor
C. White piedra
D. candidiasis
Answer: B
Reference: Medical Microbiology by Jawetz, 23rd ed. p. 628
MPL: 1

42. Melanie is a cook in a small restaurant responsible for the barbecue stand.
Later, because of arthritis she was reassigned to the counter as the one
taking the orders. She noticed that her left hand still showed the blackish
discoloration caused by handling charcoal. She had not noticed it before
because after cooking she does not wash her hand often because of fear
of “ pasma”. This time she used alcohol to clean the discoloration, but to
her surprised it had no effect on the “ dirt.” After trying to remove it to no
avail for a week, she decided to consult a doctor. The diagnosis given
was tinea nigra. The etiologic agent of this is :
A. Hortaea werneckii
B. Cladosporium spp.
C. Trichophyton spp.
D. Exophiala hortai

Answer: A
Reference: Medical Microbiology by Jawetz, 23rd ed. p. 628
MPL: 0.25

43. A group of “ street oldies” were taken to the DSWD. They were given
bath, fresh clothes and something to eat. A medical team was also
present who did some quickie check up on them. One man became
the center of attraction of the doctors and interns because of the
presence of soft, yellowish nodules on the hair of his axilla and beard.
The interns thought of them as the eggs and nits of lice. The were
however proven wrong after they had squeezed some of it. The most
likely diagnosis for this is :
A. Black piedra
B. Trichophyton hair infection
C. White piedra
D. Excess soap suds

Answer: C
Reference: Medical Microbiology by Jawetz, 23rd ed. p. 628
MPL: 0.33
11

44. Vic plays basketball almost everyday after school for the last 6 weeks,
until the vesicular lesions in his both feet particularly in the toe webs
became ulcerative and painful. Also, he gets embarrassed in the shower
room because his feet smell. The obvious diagnosis is athlete’s foot.
Three of the following dermatophytes below are common etiologic agents
of athlete’s foot EXCEPT :
A. T.mentagrophytes
B. T. rubrum
C. E. flocossum
D. M. canis

Answer: D
Reference: Medical Microbiology by Jawetz, 23rd ed. pp.631-632
MPL: 0.25

45. RC, a 40-year-old farmer consulted a dermatologist for the multiple


sinuses in his left arm that seem to follow a line. According to him, it
started as small wounds caused by rose’s thorns, when he was pruning
the ground roses. The discharge from the sinuses showed oval budding
cells that are variable in shape, but often fusiform. The discharge was
cultured in SAB. Growth in SAB yielded septate hyphae with small conidia
clustered at the tip of conidiophores, appearing like daisies with stem.
The most likely identification of this organism is: A. Trichophyton
mentagrophytes
B. Phialophora species
C. Sporothrix schenckii
D. Cladosporium carrionii

Answer: C
Reference: Medical Microbiology by Jawetz, 23rd ed. p. 632
MPL: 0.25

46. A farmer from Laguna consulted the district hospital for his “ big foot ” with
draining sinuses. He has been with it for almost three years now. He
was forced to see a doctor because he could no longer do his job in the
farm. The doctor noted that black granules are coming out of the sinuses
. The most probable diagnosis is:
A. sporothrichosis
B. mycetoma
C. elephantiasis
D. chromomycosis

Answer: B
Reference: Medical Microbiology by Jawetz, 23rd ed. p.636
MPL: 0.5

47.The black granules coming out of the sinuses in the “ big foot” of a farmer
are hard and showed intertwined septate hyphae. Which of the following is
the most likely etiologic agent?
A. Madurella grisea
B. Actinomyces
C. Nocardia spp.
D. Trichophyton rubrum
12

Answer: A
Reference: Medical Microbiology by Jawetz, 23rd ed. p. 630
MPL: 0.5

48. Jojo a Filipino migrant in California came home for a two-month vacation.
He thought he can relax, but on his second week in the Philippines he had
no choice but to seek medical help for small draining sinus on his left
chest at the level of the axilla. Other symptoms were cough and fever .
Examination of the discharge using 10% KOH showed sac like structure
with spore like structures inside. The most likely diagnosis is:
A. Histoplasmosis
B. Coccidioidomycosis
C. Blastomycosis
D. Tuberculosis

Answer: B
Reference: Medical Microbiology by Jawetz, 23rd ed. p. 638
MPL: 0.33

Subject: Microbiology (Bacteriology)


49. An elderly woman with pneumonia developed bloody diarrhea on the 7th
hospital day, accompanied by abdominal cramps and fever. The doctors
got worried because her fever caused by pneumonia had lysed as early as
her 3rd hospital day. She was immediately placed in the ICU and all
antibiotics were stopped. She has been on ampicillin for 5 days prior to
admission and on admission was started with clindamycin and augmentin
Her condition is probably caused by:
A. Enterohemorrhagic Escherichia coli
B. Shigella dysenteriae
C. Clostridium difficile
D. Vibrio parahemolyticus
Answer: C
Reference: Medical Microbiology by Jawetz, 23rd ed. p. 210
MPL: 0.33
50. Severino had an accident that resulted in the amputation of his right leg, two
inches above the knee. About five days post-op, signs of wound infection
was evident. Local wound care was given and the exudate was sent to the
lab. The doctors were willing to wait for the lab results before changing
antibiotics. After two days the culture came with no organism isolated from
the specimen as reported. They suspected anaerobic infection. Anaerobic
infection is usually suspected when:
A. there is presence of foul smell
B. the site of infection is near a mucosal surface
C. there is gas in tissues
D. all of the above
Answer: D
Reference: Medical Microbiology by Jawetz, 23rd ed. p. 309
MPL: 1

51. George was rushed to the hospital for fever and pain in the right lower
abdomen. Rectal exam was compatible with the diagnosis of
13

appendicitis. He was again rushed to the operating table because of


unstable vital signs. On opening up, he was found to have a ruptured
appendicitis with abscess formation around it. The exudates later yielded
Bacteroides fragilis. Which of the following factors promote abscess
formation by B. fragilis?
A. capsule
B. lipopolysaccharide
C. pili
D. superoxide dismutase

Answer: A
Reference: Medical Microbiology by Jawetz, 23rd ed. p. 309
MPL: 0.33

52. GC, a 20-year-old girl working in the call center, consulted a gynecologist
because of whitish-gray vaginal discharge with a bad odor of about a
week duration. Previous to it, she had several sexual relationships with
her boyfriend of two months. With her condition, she felt guilty and
decided to lead a clean life as soon as the she gets well. The doctor got
some fluid from the vaginal canal, placed some on the slide and some she
tested for pH which was 5.5 against the normal value of 4.5. Wet mount
showed many epithelial cells which appear granular. No PMN was noted.
Which of the following is the most likely diagnosis?
A, Trichomonas vaginalis
B. Yeast vaginitis
C. Gonorrhea
D. Bacterial vaginosis

Answer: D
Reference: Medical Microbiology by Jawetz, 23rd ed. p. 316
MPL: 1

53. A 67-year-old man was taken to the ER for difficulty of breathing. He has
been feeling weak since 4 days ago. He has a chronic cigarette cough
since he was in his 40s, but did not bother to see a doctor for his
complaints until his son noticed that he looked worst. On physical
examination inspiratory and expiratory wheezes and rales were noted
over the right lower lung . His chest x-ray further revealed patchy lower
right lung infiltrates. The differential diagnosis for this patient is:
A. Streptococcus pneumoniae
B. Legionella pneumophilia
C. Mycoplasma pneumoniae
D. all of the above

Answer: D
Reference: Medical Microbiology by Jawetz, 23rd ed. p. 318
MPL: 0.25

54. Joe was diagnosed with lung cancer and in his bronchial washing acid-fast
bacilli were seen mixed with the cancer cells. Joe had been given the full
short course of the TB regimen before and so the doctor suspected a drug
resistant strain. The bronchial washing was cultured and it grew orange
colored colonies when exposed to light. The most probable etiologic agent
is:
14

A. Mycobacterium tuberculosis
B. Mycobacterium smegmatis
C. Mycobacterium kansasii
D. Mycobacterium avium complex

Answer: C
Reference: Medical Microbiology by Jawetz, 23rd ed. p. 327
MPL: 0.5

55. You are giving lectures to TB patients about the characteristics of


Mycobacterium tuberculosis. This is to convince them that they should
cover their mouths when coughing and to avoid spitting in the environment.
Which of the following characteristics should NOT be emphasized to them?
A. The organism is grown in complex media
B. The organism is resistant to drying and survive for long
periods in dried sputum
C. The organism is resistant to acid and alkali
D. The organism are found in the droplet nuclei discharge by
the infected person.

Answer: A
Reference: Medical Microbiology by Jawetz, 23rd ed. p. 321
MPL: 0.25

56. A visiting media team conducted the “ search for the child with primary
complex ” in an elementary school in Tanay. After a thorough physical
examination, the children in two grade one sections were skin tested using
tuberculin at a dose of 5TU. Which of the following statements on PPD is
correct ?
A. The tuberculin skin test becomes positive within 4-6 weeks
after primary infection.
B. Immunization with BCG will result in a positive PPD which
may last for 3-7 years.
C. Persons who had been PPD positives before but are healthy
may fail to give a positive PPD test when tested again.
D. All are correct

Answer: D
Reference: Medical Microbiology by Jawetz, 23rd ed. p. 323
MPL: 0.25

57. About fifty percent of the grade one students from Barrio Tubibi was
found to have primary tuberculosis. Which of the following features of
tuberculosis is most correct?
A. characterized by acute exudative lesions which rapidly spreads to
the lymphatics and regional lymph nodes
B. all organism are killed when by the immune response that
develop during primary infection
C. It is often accompanied by severe pulmonary edema
D. Caseous materials streaked with blood often are seen in
expectorated sputum

Answer: A
Reference: Medical Microbiology by Jawetz, 23rd ed. p. 322
MPL: 1
15

58. Paul, a 33-year-old lawyer from Ilocos was seen in the OPD with
maculopapular rash over his trunk but not in his mouth or on his palms.
He is a bachelor and travels around the Philippines a lot. The attending
doctor’s impression was secondary syphilis. He ordered RPR test to be
done which turns out to be reactive. Paul was advised to send his serum
to a diagnostic test for TPHA or FTA-ABS test, which later turned out to be
positive. Which of the following diseases can be ruled out ?
A. atypical measles
B. Secondary syphilis
C. Coxsackie virus infection
D. German measles

Answer: B
Reference: Medical Microbiology by Jawetz, 23rd ed. pp.332-333
MPL: 1

59. You Ming, a 14-year-old exchange student from China joined the annual
boy’s scout camping. They stayed in the province for two weeks. On
their second week, You Ming together with some boys went to the forest
and came back with many insect bites. You were so afraid because they
even extracted a fat-bellied tick in his right leg. At the end of the two
weeks, he noticed the tick bite to be reddish but flat. It grew bigger
every day, with clearing of the central area. With the expansion of the
lesion was fever, chills, myalgias and headache. At this point his guardian
brought him to the hospital. The most likely diagnosis is:
A. leptospirosis
B. lyme disease
C. Scrub typhus
D. Rat bite fever

Answer: B
Reference: Medical Microbiology by Jawetz, 23rd ed. p. 336
MPL: 0.5

60. An MMDA traffic enforcer was admitted to the hospital for sudden onset of
high fever (39 C), headache and pain in his calf muscles. On admission his
blood tests showed, increased PMN, abnormal liver and renal function test.
On physical examination he also had subconjunctival hemorrhage. He also
had a history of wading in the floodwaters of Tondo, about 10 days prior to
the illness. The diagnosis of the attending medical intern was leptospirosis.
Which of the following would be most likely to confirm the diagnosis?
A. Test acute and convalescent sera using the RPR
B. Culture the blood on Thayer Martin agar
C. Test acute and convalescent serum for antileptospiral antibodies
D. Do darkfield examination of serum and urine for leptospires

Answer: C
Reference: Medical Microbiology by Jawetz, 23rd ed. p. 340
MPL: 0.5

Subject: Parasitology (40)


Choose the correct answer.
61. A 31-year old female from Southern Mindanao is brought in for chronic
diarrhea of two months duration. Stool examination reveals larvae with
prominent genital primodium. What is your probable diagnosis?
a. Capillariasis
b. Hookworm infection
16

c. Strongyloidiasis
d. Ascariasis

Answer: C
Ref. Clinical Parasitology by Beaver and Jung 9th edition, pp. 260-261
MPL: 0.5

62. A 10-year-old girl, with perianal pruritus, was brought by her mother to her
pediatrician. What is your most probable diagnosis?
a. Ascariasis
b. Trichuriasis
c. Enterobiasis
d. Hookworm infection

Ans: C
Ref. Clinical Parasitology by Beaver and Jung 9th edition, pp. 304-305
MPL: 0.75

63. A 10-year old girl with perianal pruritus was brought by her mother to her
pediatrician. Based from your probable diagnosis, what diagnostic
procedure will you request to support your diagnosis?
a. Stool concentration technique
b. Direct fecal smear
c. Harada-mori stool culture
d. Cellulose tape swab

Ans: D
Ref. Clinical Parasitology by Beaver and Jung 9th edition, pp. 304-306
MPL: 0.75

64. A 15-year old boy from Davao del Norte is noted to have pallor and
malnutrition. Stool examination reveals an ovum with thin colorless cell
wall. What is your diagnosis?
a. Ascariasis
b. Trichuriasis
c. Enterobiasis
d. Hookworm infection

Ans: D
Ref. Clinical Parasitology by Beaver and Jung 9th edition, p.285
MPL: 0.5

65. A 15-year old boy from Davao del Norte is noted to have pallor and
malnutrition. Stool examination reveals an ovum with thin colorless cell
wall. Probable diagnosis is hookworm infection. Name the portal of entry
of this worm.
a. oral ingestion
b. inhalation
c. skin penetration
d. mosquito bite

Ans: C
Ref. Clinical Parasitology by Beaver and Jung 9th edition, pp. 280-285
MPL: 0.5
17

66. Ova with flat bi-polar plugs were seen in the stool specimen of a 16-year
old female with chronic diarrhea from Compostela Valley province. What is
your diagnosis?
a. Capillariasis
b. Hookworm infection
c. Strongyloidiasis
d. Ascariasis

Ans: A
Ref. Clinical Parasitology by Beaver and Jung 9th edition, p. 246
MPL: 0.5

67. Ova with flat bi-polar plugs were seen in the stool specimen of a 16-year
old female with chronic diarrhea from Compostela Valley province. If your
diagnosis is correct, how did this patient acquire the infection?
a. Eating of salad
b. Eating raw infected snail
c. Eating raw infected fresh water fish
d. Eating raw or improperly cooked meat

Ans: C
Ref. Clinical Parasitology by Beaver and Jung 9th edition, pp. 246-247
MPL: 0.5

68. A 24-year old abaca farmer from Sorsogon with fever associated with
signs and symptoms of inflammation of the lymph glands was brought to
the Out Patient Department of the Philippine General Hospital. What
parasitic diagnostic procedure will you request in the laboratory?
a. Stool examination
b. Sputum examination
c. Thick blood film
d. Urine examination

Ans: C
Ref. Clinical Parasitology by Beaver and Jung 9th edition, pp.752-753
MPL: 0.75

69. A 24-year old abaca farmer from Sorsogon with fever associated with
signs and symptoms of inflammation of the lymph glands was brought to
the Out Patient Department of the Philippine General Hospital.
Microscopic finding in the blood taken at 10:00 pm revealed presence of
larvae. What is your diagnosis?
a. Malaria
b. Toxoplasmosis
c. Filariasis
d. Angiostrongiloidiasis

Ans: C
Ref. Clinical Parasitology by Beaver and Jung 9th edition, pp.749-750
MPL: 0.25

70. A 24-year old abaca farmer from Sorsogon with fever associated with
signs and symptoms of inflammation of the lymph glands was brought to
the Out Patient Department of the Philippine General Hospital.
Microscopic finding in the blood taken at 10:00 pm revealed presence of
microfilariae. What drug can you prescribe to this patient?
a. Mebendazole
b. Metronidazole
c. Di-ethyl carbamazine
d. Praziquantel

Ans: C
18

Ref. Clinical Parasitology by Beaver and Jung 9th edition, p.362


MPL: 0.75

71. A 40-year old male overseas worker complaining of muscle pains seek
medical attention upon his arrival to the Philippines. Biochemical tests
showed elevated creatinine, phosphokinase, lactate dehydrogenase and
myokinase levels. Results of complete blood count showed high blood
eosinophilia. What is your most probable diagnosis?
a. Taeniasis
b. Capillariasis
c. Trichinosis
d. Filariasis

Ans: C
Ref. Clinical Parasitology by Beaver and Jung 9th edition, pp.238-239
MPL: 0.5

72. A 40-year old male overseas worker complaining of muscle pains seek
medical attention upon his arrival to the Philippines. Biochemical tests
showed elevated creatinine, phosphokinase, lactate dehyrogenase and
myokinase levels. Results of complete blood count showed high blood
eosinophilia. What other laboratory test would you recommend to confirm
your probable diagnosis?
a. Bentonite flocculation test
b. Montenegro test
c. Sabin Feldman test
d. Knott’s blood concentration test

Ans: A
Ref. Clinical Parasitology by Beaver and Jung 9th edition, p. 768
MPL: 0.25

73. A 40-year old male overseas worker complaining of muscle pains seek
medical attention upon his arrival to the Philippines. Biochemical tests
showed elevated creatinine, phosphokinase, lactate dehyrogenase and
myokinase levels. Results of complete blood count showed high blood
eosinophilia. Serological test may confirm your diagnosis, but since there
is no available serological test, what would you request to confirm your
diagnosis?
a. Skin test
b. Muscle biopsy
c. Rectal biopsy
d. Skin scrapping

Ans: B
Ref. Clinical Parasitology by Beaver and Jung 9th edition, p.754
MPL: 0.5

74. A flat whitish worm, measuring 1.5 c.m. long was submitted to the
Diagnostic Parasitology Laboratory for identification. It was found in the
underwear of a 7-year old girl complaining of abdominal discomfort and
occasional itchiness of the perianal area. What parasitic infection would
you consider?
a. Enterobiasis
b. Taeniasis
c. Strongyloidiasis
d. Heterophydiasis

Ans: B
19

Ref. Clinical Parasitology by Beaver and Jung 9th edition, p.521


MPL: 0.5

75. A flat whitish worm measuring 1.5cm long was submitted to the Diagnostic
Parasitology Laboratory for identification. It was found in the underwear of
a 7-year old girl complaining of abdominal discomfort and occasional
itchiness of the perianal area. Laboratory results identified the worm as
segment of a Taenia saginata. How did this patient acquire the infection?
a. Eating of infected pork salad
b. Eating raw infected snail
c. Eating raw infected fresh water fish
d. Eating raw infected beef

Ans: D
Ref. Clinical Parasitology by Beaver and Jung 9th edition, p.521
MPL: 0.75

76. A flat whitish worm measuring 1.5 cm long was submitted to the
Diagnostic Parasitology Laboratory for identification. It was found in the
underwear of a 7-year old girl complaining of abdominal discomfort and
occasional itchiness of the perianal area. Laboratory results identified the
worm as segment of a Taenia saginata. What is the drug of choice for this
parasitic infection?
a. mebendazole
b. albendazole
c. praziquantel
d. pipperazine

Ans: C
Ref. Clinical Parasitology by Beaver and Jung 9th edition, p.522
MPL: 0.5

77. Adult roundworm, measuring 27 cm. in length, was seen in the colon of a
10-year old boy who died of pneumonia. Manifestations such as lung
infiltration, asthmatic attacks, and edema of the lips were documented
before the patient died. What is the most probable parasitic infection can
you identify in this case?
a. Trichuriasis.
b. Ascariasis
c. Capillariasis
d. Hookworm infection

Ans: B
Ref. Clinical Parasitology by Beaver and Jung 9th edition, pp. 226-228
MPL: 0.75

78. A fisherman from Davao Oriental was rushed to a nearby hospital with
chest pain, persistent cough, and hemoptysis as his main complains.
Patient interview revealed excessive drinking of native alcohol together
20

with raw mountain crabs as their “pulutan”. What is the Diagnostic work-up
you should include for this patient?
a. chest x-ray
b. sputum examination
c. stool examination
d. all of the above

Ans: B
Ref. Clinical Parasitology by Beaver and Jung 9th edition, pp.466-468
MPL: 0.75

79. A fisherman from Davao Oriental was rushed to a nearby hospital with
chest pain, persistent cough, and hemoptysis as his main complains.
Patient interview revealed excessive drinking of native alcohol together
with raw mountain crabs as their “pulutan”. Chest x-ray demonstrated
patchy, cloudy infiltration of the lungs with nodular shadows and calcified
spots. What parasitic infection can you consider?
a. Pulmonary tuberculosis
b. Paragonimiasis
c. Ascariasis
d. Strongyloidiasis

Ans: B
Ref. Clinical Parasitology by Beaver and Jung 9th edition, pp. 464-468
MPL: 0.75

80. A fisherman from Davao Oriental was rushed to a nearby hospital with
chest pain, persistent cough, and hemoptysis as his main complains.
Patient interview revealed excessive drinking of native alcohol together
with raw mountain crabs as their “pulutan”. Chest x-ray demonstrated
patchy, cloudy infiltration of the lungs with nodular shadows and calcified
spots. Paragonimiasis is the parasitic infection you are considering? What
social factor strongly contributes to your diagnosis?
a. drinking habits
b. occupation
c. eating habits
d. gender

Ans: C
Ref. Clinical Parasitology by Beaver and Jung 9th edition, p.468
MPL: 0.75

81. A 51-year old farmer from Surigao, Mindanao was brought to a hospital
with abdominal pain and frequent loose stools. Further examination
revealed hepatomegaly with tenderness in the upper right quadrant of the
abdomen. Laboratory results showed increase of eosinophilia. Direct fecal
smear showed no ova or parasite seen, while rectal biopsy demonstrated
eggs with miracidia. Based from clinical and laboratory results, what
parasitic disease can you suggest?
21

a. schistosomiasis
b. paragonimiasis
c. leishmaniasis
d. heterophydiasis

Ans: A
Ref. Clinical Parasitology by Beaver and Jung 9th edition, pp.416-425
MPL: 0.5

82. A 51-year old farmer from Surigao, Mindanao was brought to a hospital
with abdominal pain and frequent loose stools. Further examination
revealed hepatomegaly with tenderness in the upper right quadrant of the
abdomen. Laboratory results showed increase of eosinophilia. Direct fecal
smear showed no ova or parasite seen, while rectal biopsy demonstrated
eggs with miracidia. What factor should you consider to make your
specific diagnosis?
a. clinical signs and symptoms
b. recovery of characteristic eggs
c. increase of eosinophilia
d. evidence of exposure to endemic area

Ans: B
Ref. Clinical Parasitology by Beaver and Jung 9th edition, pp. 416-440
MPL: 0.75

83. A 51- year old farmer from Surigao, Mindanao was brought to a hospital
with abdominal pain and frequent loose stools. Further examination
revealed hepatomegaly with tenderness in the upper right quadrant of the
abdomen. Laboratory results showed increase of eosinophilia. Direct fecal
smear showed no ova or parasite seen, while rectal biopsy demonstrated
eggs with miracidia. What is/are the possible reason for the false negative
result of stool examination?
a. early infection
b. all male infection
c. all female infection
d. all of the above

Ans: D
Ref. Clinical Parasitology by Beaver and Jung 9th edition, pp. 416-425
MPL: 0.75

84. A 51-year old farmer from Surigao, Mindanao was brought to a hospital
with abdominal pain and frequent loose stools. Further examination
revealed hepatomegaly with tenderness in the upper right quadrant of the
abdomen. Laboratory results showed increase of eosinophilia. Direct fecal
smear showed no ova or parasite seen, while rectal biopsy demonstrated
eggs with miracidia, which lead you to diagnose schistosomiasis. This
parasitic infection is also known as:
a. Bilharziasis
b. Katayama fever
c. Snail fever
d. All of the above

Ans: D
Ref. Clinical Parasitology by Beaver and Jung 9th edition, pp.434-440
MPL: 0.75
22

85. A 62-year old farmer from Sorsogon was diagnosed for schistosomiasis
japonicum. Its main pathology and disease manifestation are due to host
granulomatous reaction. The underlying mechanism for its granuloma
formation is due to:
a. acute inflammatory reaction
b. chronic vascular obstruction
c. immediate allergic reaction
d. delayed hypersensitivity

Ans: D
Ref. Clinical Parasitology by Beaver and Jung 9th edition, pp.416-425
MPL: 0.75

86. A 51-year old farmer from Surigao, Mindanao was brought to a hospital
with abdominal pain and frequent loose stools. Further examination
revealed hepatomegaly with tenderness in the upper right quadrant of the
abdomen. Laboratory results showed increase of eosinophilia. Direct fecal
smear showed no ova or parasite seen, while rectal biopsy demonstrated
eggs with miracidia, which lead you to diagnose schistosomiasis. What is
the drug of choice for this parasitic infection?
a. mebendazole
b. albendazole
c. praziquantel
d. pipperazine

Ans: C
Ref. Clinical Parasitology by Beaver and Jung 9th edition, p.424
MPL: 0.75

87. A 51-year old farmer from Surigao, Mindanao was brought to a hospital
with abdominal pain and frequent loose stools. Further examination
revealed hepatomegaly with tenderness in the upper right quadrant of the
abdomen. Laboratory results showed increase of eosinophilia. Direct fecal
smear showed no ova or parasite seen, while rectal biopsy demonstrated
eggs with miracidia, which lead you to diagnose schistosomiasis. Serious
complications of this parasitic disease include:
a. renal failure
b. intestinal lesions
c. bone metastases
d. cerebral involvement

Ans: D
Ref. Clinical Parasitology by Beaver and Jung 9th edition, pp. 421-423
MPL: 0.5

88. Sexually active 26-year old woman consulted a VD control clinic for
vaginal itching and purulent discharge. Your tentative diagnosis should
include the following:
a. gonorrhea
b. trichomoniasis
c. candidiasis
d. all of the above

Ans: D
23

Ref. Clinical Parasitology by Beaver and Jung 9th edition, pp. 50


MPL: 0.25

89. Sexually active 26-year old woman consulted a VD control clinic for
vaginal itching and purulent discharge. Trichomoniasis is your tentative
diagnosis. What parasitologic examination should you include in your
work-up?
a. stool culture
b. wet mount of vaginal fluid
c. pH test of blood sample
d. ova and parasite fecal smear

Ans: B
Ref. Clinical Parasitology by Beaver and Jung 9th edition, p.51
MPL: 0.5

90. Sexually active 26-year old woman consulted a VD control clinic for
vaginal itching and purulent discharge. Trichomoniasis is your tentative
diagnosis, which is best treated with:
a. albendazole
b. mebendazole
c. metronidazole
d. praziquantel

Ans: C
Ref. Clinical Parasitology by Beaver and Jung 9th edition, p. 51
MPL: 0.75

91. Sexually active 26-year old woman consulted a VD control clinic for
vaginal itching and purulent discharge. Trichomoniasis is your tentative
diagnosis, which is contracted almost exclusively through sexual
intercourse. Control and prevention require
a. prompt treatment of the patient
b. prompt treatment of sexual partner
c. treatment of both
d. require no treatment, since it is self-limiting

Ans: C
Ref. Clinical Parasitology by Beaver and Jung 9th edition, p. 51
MPL: 0.75

92. A 30-year old woman, native of Bulacan, traveled to Palawan for a


vacation. After 2 weeks she consulted her doctor with chills and fever as
her main complains. What parasitologic test would you request to
establish your diagnosis?
a. sputum examination
b. stool examination
c. thick blood smear
d. thick and thin blood smear

Ans: D
Ref. Clinical Parasitology by Beaver and Jung 9th edition, pp. 180-181
MPL: 0.5

93. A 30-year old woman, native of Bulacan, traveled to Palawan for a


vacation. After 2 weeks she consulted her doctor with chills and fever as
her main complaints. Microscopic examination of thick and thin blood
smear showed infected RBC with double chromatin dot. What is your most
probable diagnosis?
24

a. Not malaria
b. Plasmodium falciparum
c. Plasmodium vivax
d. Plasmodium malariae

Ans: B
Ref. Clinical Parasitology by Beaver and Jung 9th edition, pp.185-188
MPL: 0.75

94. A 6-month old baby was brought in for a fever of one-day duration. She is
found to have a temperature of 38.5°C and with hepatomegaly (3 cm) and
splenomegaly (3 cm). Microscopic examination revealed infected RBC
with presence of band form stages. What is your most probable
diagnosis?
a. Not malaria
b. Plasmodium falciparum
c. Plasmodium vivax
d. Plasmodium malariae

Ans: D
Ref. Clinical Parasitology by Beaver and Jung 9th edition, pp. 184-185
MPL: 0.75

95. A 49-year-old man from Manila received 4 units of packed red blood cells
(PRBCs) on January 15 while undergoing hip replacement surgery. He
was again hospitalized on February 1 with fever, hypotension, and renal
failure. Peripheral blood smears confirmed malarial infection. The patient
has never traveled outside Manila for 20 years. How did this patient
acquire the infection?
a. bite of mosquito
b. blood transfusion
c. relapse
d. infected syringe

Ans: B
Ref. Clinical Parasitology by Beaver and Jung 9th edition, pp.174-180
MPL: 0.75

96. A company plans to send a large number of employees to work in


Afghanistan, for variable periods of time. Medical examination by their
employees’ health service accidentally found this banana shape organism
while doing routine CBC. Will there be a malaria risk in Afghanistan?
a. No risk
b. Risk in all areas
c. Risk in areas where mosquito vectors are present
d. Risk in all areas, during rainy season only

Ans: C
Ref. Clinical Parasitology by Beaver and Jung 9th edition, pp. 188-193
MPL: 0.75

97. A 6-month old baby was brought in for a fever of one-day duration. She is
found to have a temperature of 38.5°C and with hepatomegaly (3 cm) and
splenomegaly (3 cm). Microscopic examination revealed infected RBC
with presence of ring form stage only. What is your most probable
diagnosis?
a. Not malaria
b. Plasmodium falciparum
c. Plasmodium vivax
25

d. Plasmodium ovale

Ans: B
Reference: Clinical Parasitology by Beaver and Jung 9th edition, pp. 185-188
MPL: 0.75

98. A 40-year old man, resident of the Manila, presents to an emergency


room with a 5-day history of fever, chills, nausea, vomiting, and myalgias.
He returned 2 weeks ago from a 2-year job contract from Zambia.
Microscopic examination revealed oval shaped infected RBC. What is
your most probable diagnosis?
a. Plasmodium falciparum
b. Plasmodium vivax
c. Plasmodium ovale
d. Plasmodium malariae

Ans: C
Reference: Clinical Parasitology by Beaver and Jung 9th edition, pp. 182-184
MPL: 0.75

99. An overseas worker with febrile illness that began in December 2003,
approximately 3 months and 14 months, respectively, after leaving Africa,
consulted the hospital. During the course of his illness, the patient
experienced fluctuating temperatures and lost 13 pounds of body weight.
Light-microscopic examinations or cultures of bone marrow and liver-
biopsy specimens noted group of intracellular small rounded organisms.
Laboratory results identify it as amastigote stage. What is the most likely
parasitic infection present in this patient?
a. Malaria
b. Leishmaniasis
c. Toxoplasmosis
d. Trypanosomiasis

Ans: B
Reference: Clinical Parasitology by Beaver and Jung 9th edition, pp.71-74
MPL: 0.5

100. An overseas worker coming home from Uganda was rushed to a nearby
hospital for seizure attack. His relatives informed the medical officer on
duty that the patient is experiencing progressive confusion and personality
changes. Laboratory examination of the CSF revealed presence of
polymorphic with undulating membrane. What is the most likely parasitic
infection present in this patient?
a. Malaria
b. Leishmaniasis
c. Toxoplasmosis
d. Trypanosomiasis

Ans: D
Reference: Clinical Parasitology by Beaver and Jung 9th edition, pp. 77-87
MPL: 0.5
MICROBIOLOGY & PARASITOLOGY

1. The most common cause of meningitis in the newborn and sepsis in immunocompromised adults is:
A. Nocardia spp. C. Legionella pneumophilia
B. Actinomyces spp. D. Listeria monocytogenes

Ans. D
Nocardia causes mycetoma, cavitary nodules & brain abscesses. Actinomyces causes orocervicofacial
abscesses showing sulfur granules. Legionella causes pneumonia.

Reference: Zinsser p. 484

2. This organism produces a blue-green pigment and therefore produces a characteristic blue-green pus.
A. Staph aureus C. Aspergillus fumigatus
B. Pseudomonas aeruginosa D. Strep. pyogenes

Ans. B
Pseudomonas aeruginosa produce pyocyanin pigment w/c is blue green in color and also has antibiotic-
like characteristics against some bacteria

Reference: Zinsser p. 577

3. Each of the following statements concerning the surface structures of bacteria is correct EXCEPT:
A. Pili mediate the interaction of bacteria with mucosal epithelium
B. Polysaccharides capsules retard phagocytosis
C. Both gram negative rods and cocci have polysaccharides (endotoxin in their cell wall)
D. Bacterial flagella are not antigenic in humans

Ans. D
Bacterial flagella are made of proteins and are useful in distinguishing serovars or variation of gram
negative bacteria such as E. coli H. antigens Sal. H .antigens.

Reference: Zinsser. Chapter 3

4. The most common site of asymptomatic gonococcal infection in women is the:


A. Vagina C. Urethra
B. Myometrium D. Endocervix

Ans. D
The columnar epithelial cells here are the ones that are being infected by N. gonorrhea.(A). Vaginal walls
composed of stratified squamous epithelial cells which are not colonized. (B) and (C) are not affected.

Reference: Zinsser p. 456

5. The following etiologic agents are known to cause diarrhea EXCEPT:


A. Clostridium perfringens C. V. cholera
B. Rotavirus D. Proteus vulgaris

Ans. D
Protues vulgaris produce urinary tract infection. C. clostridium perfringens diarrhea thru it’s invasiveness
so blood and mucus is (+) V. cholera-produced diarrhea by virtue of enterotoxin. Rotavirus most common
cause of viral gastroenteritis in small children.

Reference: Zinsser p. 552

6. A 20 year old male developed a carbuncle with surrounding cellulitis. Gram stain revealed gram positive
cocci, catalase and coagulase positive. Which of the following antibiotics is initially appropriate for this
patient?
A. Vancomycin C. Cefriaxone
B. Cloxacillin D. Penicillin

1
Ans. B
90% of S. aureus strains are resistant to penicillin because pf production of plasmid-derived β-lactamses.
Such organisms can be treated with β-lactamase-resistant penicillin such as cloxacillin or dicloxacillin
oxacillin. Oxacillin and Nafcillin have unpredictable absorption by oral route, thus is not recommended

Reference: Zinsser p. 412

7. Which of the following features help distinguish group A streptococcus from other streptococcal species?
A. Bacitracin susceptibility C. Optochin sensitivity
B. Bile solubility D. Growth inhibition in 6.5 % NaCl

Ans. A
Group A streptococcus is a β-hemolytic streptococcus and can usually be distinguished from the other β-
hemolytics by its bacitracin susceptibility or by precipitation with specific anti sera (against its
"Lancefrield antigen"). Bile solubility test and optochin test distinguishes the L-hemolytic streptococci, S.
pneumoniae is bile soluble and optochin sensitive. Among the hemolytic streptococci, S. viridans growth is
inhibited by 6.5% NaCl while Enterococcus species grows in 6.5% NaCl.

Reference: Zinsser p. 419

8. Which of the following Clostridia cause gangrene?


A. Clostridium tetani C. Clostridium perfringens
B. Clostridium botulinum D. Clostridium difficile

Ans. C
C. tetani causes lockjaw. C. botilinum causes botulism. C. difficile causes antibiotic associated
pseudomembranous colitis. C. perfringens cause gasgangrene and Food poisoning.

Reference: Zinsser p. 639

9. This gram positive rod is usually associated with bacterial vaginosis producing malodorous vaginal
discharge and "clue cells"
A. Erysipelothrix rhusopathiae C. Gardenella vaginalis
B. Lactobacillus acidophilus D. Propionibacterium acnes

Ans. C
E. rhusopathiae causes erysipeloid skin infection that resembles erysipelas occurring among meat and fish
handlers. L. acidphilus is normal flora of genital tract and is the main source of lactic acid. P. acnes
produces lipase which contributes to the genesis of acne.

Reference: Zinsser p. 604-605

10. Which of the following organisms is MOST likely to involve invasion of the intestinal mucosa.
A. Vibrio cholera C. Enterotoxigenic E. coli
B. Shigella sonnei D. Clostridium botulinum

Ans. B
Shigella sonnei is the only invasive, presence of blood and mucus in the stool. Clostridium botilinum , V.
cholera and Enterotoxigenic E. coli causes diarrhea by producing enterotoxin

Reference: Zinsser p. 561

11. Food poisoning that produces gastrointestinal symptoms approximately 1-2 hours after eating is most likely
to be due to:
A. Salmonella enteritidis C. Clostridium perfringens
B. Campylobacter jejuni D. Staph. aureus

Ans. D
2
Staph aureus – the incubation period is 1-2 hrs. after ingestion of contaminated food. Sal. enteritidis – 6
hrs. – 2 days. Campylobcacter – 8 hrs. – l day. C. perfringens – 8 hrs. – 12 hrs. Bacillus aureus – 8 hrs. –
12 hrs. V. cholera – 2 – 3 days. V. parahaemolyticus – 8 hrs. – 2 days

Reference: Zinsser p. 412

12. A 30 year old pregnant woman had a normal delivery but the fetus was stillborn. CSF examination showed
positive rods. The pathogen is most likely:
A. Listeria monocytogenes C. Group B Streptococcus
B. Gardenerella vaginalis D. Staphylococcus aureus

Ans. A
Listeria monocytogenes is a gram positive rod and can infect pregnant women and poses the threat of
stillbirth or serious damage to the fetus and also cause neurological disease. Gardenella vaginalis gives
rise to frothy, fishy odor vaginal discharge, it can also cause premature births and low-birth-weight
infants.

Reference: Zinsser 484

13. The following statements are true with Shigellosis EXCEPT:


A. Antimicrobial therapy is effective in shortening the course of diarrhea
B. Fecal-oral transmission is the only mode of transmission
C. The etiologic agent is a gram negative bacilli, under the family enterobacteriacea
D. There is an available vaccine that is being used

Ans. D
Until now they can't produce vaccine because of the antigenical complexity of the organism, its antigen has
similarity with Enteroenvasive E. coli.

Reference: Zinsser p. 559

14. A 25 year old man came to the STD clinic complaining of urethral discharge which is thick, yellow and
purulent. The following may be done to identify the pathogen EXCEPT:
A. darkfield microscopy C. Gram stain
B. culture on Thayer Martin medium D. ELISA

Ans. A
Treponema is a spirochete which can be demonstrated by D.M. and it is seen as hard-based chancre not
discharge. B-D are all being used to Neisseria gonorrhea which is the probable agent of the patient The
discharge is characteristic of which can be validated by B-D. A is for spirochetes foremost of which is
Treponema.

Reference: Zinsser 665

15. The following are the virulence factors of E. coli EXCEPT:


A. Endotoxin production C. presence of fimbriae for attachment to host cells
B. Iron binding sidephore D. production of H2S

Ans. D
E. coli does not produce H2S

Reference: Zinsser p. 546

16. Which of these organisms is a major pathogen for nosocomial infections because of its ubiquitous present
in the hospital environment?
A. Staph. Aureus C. Serratia marcescens
B. Staph. Epidermidis D. Pseudomonas aeroginosa

Ans. D

3
Pseudomonas aeruginosa, S. aureus - yellow gold pigment when cultured at room temperature. S,
epidermidis - white pigment when cultured at room temperature. Serratia marcescens - red pigment

Reference: Zinsser p. 577

17. A 3 year old girl was brought by the mother to her pediatrician because of paroxysmal coughing of 2 weeks
duration. If the probable etiologic agent is Bordetella pertusis, the fastest way to definitely identify this is:
A. Gram stain of bronchial secretions C. Throat culture on Bordet-Gengou medium
B. Blood culture D. ELISA

Ans. C
To definitely identify is to isolate the etiologic agent in the culture medium

Reference: Zinsser p. 473

18. An individual experiences severe diarrhea after eating sushi (raw fish) in a West Coast restaurant. The
most probable etiologic agent is:
A. Yersenia enterocolitica C. Shigella sonnei
B. Salmonella enteritidis D. Vibrio parahaemolyticus

Ans. D
Vibrio parahaemolyticus - is a salt loving organism and found in fishes. Salmonella enteritidis - found in
chicken, pig. Yersenia enterocolitica- reserviors rodents

Reference: Zinsser p. 571

19. When symptoms to typhoid fever first become apparent, Salmonella typhi is most frequently isolated from:
A. Feces C. Blood
B. Urine D. Bone marrow

Ans. C
Blood - on the first week of infection, Feces - second week. Urine - third week.

Reference: Zinsser p. 563

20. The area in the body where Salmonella typhi resides if one is a chronic carrier
A. small intestine C. blood stream
B. kidney D. gall bladder

Ans. D
Gall bladder - where organisms are found and are excreted in the stool and urine of the chronic carrier of
Salmonella.

Reference: Zinsser p. 562

21. Legionella pneumophila has the following characteristics EXCEPT:


A. facultative intracellular parasites C. are encapsulated relatively slender rods
B. primarily causes respiratory tract infections D. particularly requires L-cysteine

Ans. C
Legionella is unencapsulated intracelular and may cause pneumonia

Reference: Zinsser p. 696

22. A 3 year old boy was diagnose to have meningitis CSF culture on Chocolate agar showed growth of small
gram negative rods. The most probable etiologic agent is:
A. N. meningitides C. Group B Streptococcus
B. S. aureus D. H. influenzae type b

Ans. D

4
H. influenzae type b - is the most commonly isolated etiologic gram negative rods causing meningitis. N.
meningtidis - gram negative plo. Group B Strep - gram positive cocci.

Reference: Zinsser p. 462

23. Among the gram negative rods producing gastroenteritis, the most infectious is:
A. V. cholera C. Sal. Typhimurium
B. E. coli (0157) D. Shig. Dysenteriae type 1

Ans. D
Shigella - only 200 cells is enough to cause diarrhea while A-C you need to 10 3 - 10 5 cells to produce
infection.

Reference: Tortora et al

24. The resting site for leptospirosis in the natural host is the:
A. lumen of the nephrotic tubules C. endothelium
B. liver hepatocytes D. eyes

Ans. A
Leptospirosis usually infect the kidneys. Renal involvement is an interstitial nephritis with glomerular
swelling and hyperplasia. Infections with leptospirosis are incurred directly by contact with urine of
carriers or indirectly by contact with streams, swamps or wet soils contaminated with urine carriers.
These organisms are shed into urine from the nephron tubules. Urine sheeding may persist 2-3 months in a
large number of cases.

Reference: Zinsser p. 673

25. Diagnosis of leptospirosis is established in the laboratory by the following methods EXCEPT:
A. demonstration of the organisms by direct darkfield microscopy
B. serological tests
C. culture
D. gram-stain of infected tissue

Ans. D
Leptospirosis is readily cultures from the blood or CSF during the acute phase of treatment. Direct dark
field examination may be of value for the examination of specimens in which there is a high concentration
of leptospires (e.g. blood, peritoneal fluid) liver suspension from hamsters or guinea pigs infected with
clinical material. Diagnosis by microscopic examination should be confirmed by cultural or serological
tests.

Reference: Zinsser p. 674

26. Pneumococci primarily cause disease by:


A. toxin production C. eluding phagocytosis and favoring invasiveness
B. enzyme production D. producing delayed immunologic reaction

Ans. C
Pneumococci does not produce toxins and enzymes. Capsules are virulence factors which interfere with
phagocytosis and favoring invasiveness.

Reference: Zinsser p. 435

27. The formation of pseudomembrane in diphtheria is due to:


A. inhibitions by toxin of protein synthesis by ADP ribosyslation of EF-2
B. spores which spread subcutaneously
C. invasion of mononuclear phagocytic cells
D. neutralization of exotoxin by antibody of blocking the interaction of fragment B with receptors

5
Ans. A
Diphtheria toxin inhibits protein synthesis of eukaryotic cells thereby causing cell death. Pseudomembrane
formation is due to accumulation of necrotic tissue.

Reference: Zinsser p. 488

28. Lyme disease is an endemic inflammatory disorder with this distinctive skin disorder.
A. Chronic dermatitis C. erythema chronicum migrans
B. erythema marginatum D. ecthyma gangrenosum

Ans. C
Erythema chronicum migrans is a tick-borne associated syndrome.

Reference: Lippincott Illustrated Review in Microbiology p. 225

29. Serology in the diagnosis of Syphilis using a non-treponemal tests is the following EXCEPT:
A. Microhemagglutination assay C. Fluorescent Antibody Tests
B. Venereal Disease Research Laboratory D. Reiter Protein Complement Fixation

Ans. B
Two different types of test are used. Non-treponemal tests detects Wasserman or reagenic antibodies; a
few examples of the nontreponemal tests include VDRL, automated regin, Kahn, plasmacrit, Hinton and
Kline tests. Treponemal tests detect antibodies specific for treponemal antigens; they include FTA=ABS, T.
pallidium immunobolization and hemagglutination tests.

Reference: Zinsser p. 663-664

30. The following statements are true of Chlamydia EXCEPT:


A. obligate intracellular parasite
B. Genome is composed of DNA
C. Causes surface infection
D. Reticulate body is the progenitor of elementary bodies

Ans. A
The elementary body is extracellular and metabolically inert. It has DNA genome and does not invade
tissues.

Reference: Zinsser p. 774-719

31. The following statements are true about Chlamydia trachomatis EXCEPT
A. It has a reservoir in domestic fowl
B. it is the most common cause of non-gonococcal urethritis
C. it can be detected by direct immunofluorescence in specimens
D. it has a reservoir in fowl

Ans. A
All other options are correct. C. trachomatis is strictly a human pathogens, whereas other chlamydia
species are parasites of birds. It is the best documented cause of post gonococcal urethritis.

Reference: Zinsser p. 724

32. Mycoplasmas are very pleomorphic and cannot be classified as to shape because they lack the following:
A. Peptidoglycan C. nucleus
B. lipid bilayer D. lipopolysaccharide

Ans. A

6
Lacking cell walls, mycoplasmas are enclosed instead by a membrane composed of a lipid bilayer. They
are therefore plastic and very pleomorphic.

Reference: Zinsser p. 733

33. The following statements are true of Mycoplasma:


A. they have limited biosynthetic capabilities C. they can be grown in cell-free media
B. contain sterols in their cell membranes D. all of the above

Ans. D
Mycoplasmas have sterols in their lipid bilayer. They cannot synthesize the sterol ring and they require an
external source of cholesterol from serum or similar medium supplement. They can be grown in cell-free
media.

Reference: Zinsser p. 732

34. A definitive identification of M. tuberculosis can be obtained by doing the following procedure?
A. Ziehl-Neelsen stain C. Biopsy of Tuberculin test
B. AFB Culture D. Nucleic acid amplification

Ans. B
Cultures can detect small numbers of organism in the original sample. The presence of Acid-fast
organism on Ziehl Neelsen cannot distinguish on morphologic groups M. tuberculosis from the other
mycobacteria that are saprophytic. Nucleic acid amplification utilizes enzymes that rapidly IGS ribosomal
RNA which can be used for patients with positive smears and while culture results are pending.

Reference: Lippincott Illustrated Review for Microbiology p. 251

35. The absence of the mammillary layer of Ascaris fertilized embryonated eggs means that these eggs are:
A. non-viable C. potentially still infective
B. not going to mature D. laid by immature females
Ans. C
The three layers of the Ascaris fertilized eggs are the outer coarsely mammillated albuminoid layer, the
thick transparent middle layer derived from glycogen and the non-permeable innermost lipoidal vitelline
membrane that is not found in unfertilized eggs (Craig & Faust, p. 337). The loss of the outermost layer
does not make the egg less viable.

Reference: Craig & Faust p. 337

36. Brugia malayi microfilaria can be distinguished from that of Wuchereria bancrofti by the:
A. periodicity of the appearance of this stage in the blood
B. obstructive symptoms that this causes in the infected hosts
C. morphologic characteristic of the stage in peripheral blood
D. species of the vector that ingests this stage during its blood meal
7
Ans. C
Brugia malayi microfilaria is distinguishable from that of W. bancrofti by means of its specific
morphologic features. W. bancrofti microfilaria has a graceful body curvature with its somatic nuclei
arranged in rows, its cephalic structure is proportionate (width=length) with no terminal nuclei. B. malayi
microfilaria has its body thrown into kinks, its cephalic structure is disproportionate (2length=width)
and has two terminal nuclei. Both are sheathed.

Reference: Brown 6th ed. p. 154 & 161

37. Rifamficin a broad antimicrobial agent is effective treatment for tuberculosis by:
A. selective inhibition of the biosynthesis of the arabinogalactan and lipo-arabinomannan
B. mediating drug permeability and efflux
C. Inhibiting peptidoglycan synthesis by blocking action of D-alanise synthesis
D. binding to RNA polymerase thereby interfering with mRNA synthesis

Ans. D
Rifamficin is a broad spectrum antimicrobial agent that acts the interferring with the synthesis of mRNA by
binding to RNA polymerase. Selective inhibitions by ethamentol of the biosynthesis of arabinogalactan and
lipoarabinomannan, key structural components of the mycobacterial cell wall. Cyclosine inhibits synthesis
of peptidoglycan by blocking the action of the D-alanine synthetase.

Reference: Tortora p. 561

38. The atypical mycobacteria:


A. are acid-fast bacilli
B. cause chronic granulamatous infection
C. May or may not cross-react with tuberculoprotein
D. All of the above

Ans. D
The atypical mycobacteria have the same morphology on M. tuberculosis on AFB stains. They can be
cultured as photochromogenes yellow to orange pigments where expressed to light, darkness
(scotochromogenes) or do not produce pigments when exposed to light. They are frequently from soil or
water. The atypical mycobacterium that is strongly cross reactive with standard PPD. The delayed
hypersensitivity that developes strongly cross-react with standard PPD.

Reference: Zinsser p. 510

39. In primary disease, M. tuberculosis survives and grows within these host cells
A. macrophages C. neutrophils
B. basophils D. eosinophils

Ans. A
M. tuberculosis may continue to survive and remain dormant in activated macrophages where it is
protected from phagocytosis

Reference: Lippincott Illustrated Review of Microbiology p. 247

40. Mycobacteria species are usually differentiated by the following properties EXCEPT:
A. rate of growth C. pigmentation
B. colonial morphology D. acid-fast staining
Ans. D
All mycobacteria regardless of species stain positive in Acid-fast smear and have identified morphology

Reference: Lippincott Illustrated Review of Microbiology p. 245


41. Members of the genus Rickettsia are morphologically & biochemically similar to
A. gram-positive bacteria C. clostridia
B. gram negative bacteria D. viruses

Ans. B
8
Rickettsia are short-rod shaped or cocci bacillary organisms measuring 0.8 to 2.0 um long

Reference: Lippincott Illustrated Review of Microbiology p. 259

42. The hallmark of viral infection of the cell is the production of


A. inclusion bodies C. cytopathic effect
B. multinucleated giant cells D. granule formation

Ans. C
The hallmark of viral infection of the cell is the cytopathic effect. This change begin with a rounding
and darkening of the cell and culminates in either lysis or giant cell formation. Infected cells frequently
contain inclusion bodies which are discrete areas containing viral particles.

Reference: Zinsser p. 943

43. The complete infectious forms of Hepatitis B is:


A. HBeAg C. HBsAg
B. Dane particles D. HBcAg

Ans. B
The Dane particle is the complete form of Hepatitis B whereas the antigens are subunit forms of the
surface and core of the virus.

Reference: Zinsser p. 1040

44. Which of the following virus/es contain/s Hemagglutinin on its surface?


A. Hepatitis B C. Influenza virus
B. Hepatitis C D. all of the above

Ans. C
The antigen hemagglutinin is characteristic of Influenza virus.

Reference: Zinsser p. 995

45. In their multiplication, they produce DNA which is integrated into the cell genome.
A. Hepadnavirus C. Picornavirus
B. Reovirus D. Herpesvirus

Ans. A
Hepadnavirus transforms non-permissive cells by integration of DNA transcripts into host cell genome.
They carry an RNA-dependent DNA polymerase (reverse transcriptase).

Reference: Zinsser p. 810

46. The most common intrauterine viral infection is caused by:


A. Cytomegalovirus C. Adenovirus
B. Rubella D. Herpes simplex

Ans. A
Cytomegalovirus accounts for the biggest # of intrauterine infection followed by rubella.

Reference: Zinsser p. 962

47. The following statements is not true of Influenza:


A. Viruses are classified as types A, B and C
B. Recurrence of infections is due to lack of protective antibody
C. Viruses contains H and N glycoprotein responsible for antigenic shift
D. Isolation can be done by xenodiagnosis
9
Ans. D
Virus isolation is by inoculation of the allantoic and amniotic sacs of 10-day old embryonated chicken eggs
or inoculation of several types of tissue. Xenadiagnosis which is by use of insect vectors has no place
here. A, B, C are true.

Reference: Zinsser p. 995

48. Transmission of EBV is more commonly occur through:


A. salivary contact C. sexual contact
B. aerosol inhalation D. blood transfusion

Ans. A
EBV is excreted in saliva and epithelial cells of the oropharynx. It cause mononucleosis

Reference: Zinsser p. 963

49. A persistent, latent infection with intermittent reactivation is characteristic of:


A. Orthomyxoviruses C. papovaviruses
B. Paramyxoviruses D. herpesvaviruses

Ans. D
Only the herpesviruses can lie dormant is neural and non-neural tissue.

Reference: Zinsser p. 840

50. EBV is unique among the herpesviruses in its ability to:


A. transform & immortalize B-lymphocytes
B. resist lipid solvents acid pH and detergent solutions
C. enter latent state and reactivated at a later time
D. all of the above

Ans. A
EBV has the ability to transform B-lymphocytes while other herpesviruses do not.

Reference: Zinsser p. 964

51. Common characteristics of Herpesviridae are the following EXCEPT:


A. enveloped DNA viruses
B. linear, double-stranded DNA genome
C. Herpesviruses replicate in the cytoplasm
D. envelope contain antigenic, species-specific glycoproteins

Ans. C
Herpesviruses being DNA viruses, replicate in the nucleus before their release into the cytoplasm.

Reference: Zinsser p. 958

52. Among the hermaphroditic species of trematodes which has molluscs for its second intermediate host?
A. Fasciola hepatica C. Clonorchis sinensis
B. Echinostoma ilocanum D. Paragonimus westermani

Ans. B
10
F. hepatica has a fresh water plant for its 2nd IH. Mulloscs are the 2nd IH of E. ilocanum and a fish is
the 2nd IH for C. sinensis. P. westermani has a fresh water crab for its 2nd IH

Reference: Brown 6th p. 235

53. Among the DNA viruses which is non-enveloped?


A. Herpesvirus C. Papillomavirus
B. Hepatitis B virus D. Poxvirus

Ans. C
Herpes virus - DNA envelope. Hepatitis B virus - DNA enveloped. Papillomavirus - DNA non- enveloped
Poxvirus - DNA enveloped

Reference: Zinsser p. 975

54. These viruses are most commonly transmitted through sexual contact EXCEPT for:
A. hepatitis C. Cytomegalovirus
B. Varicella D. Herpes Simplex virus Type 2

Ans. B
Direct contact with infected secretions. Varicella or chickenpox is acquired via respirations transmission
by infected aerobes

Reference: Zinsser p. 955

55. The typical infected cell in Herpes simplex infection is the:


A. Negri bodies C. Atypical lymphocytes
B. Multinucleated giant cell D. intranuclear inclusion bodies in giant cells

Ans. B
Negri bodies found in Rabies. Atypical lymphocytes found in EBV infection. Intranuclear inclusion
bodies in giant cells found in CMV infection.

Reference: Zinsser p. 958

56. Among the DNA viruses which requires a DNA-dependent RNA polymerase?
A. Herpes virus C. Papillomavirus
B. Hepatitis B virus D. Poxvirus

Ans. D
Poxvirus requires this enzyme because the virus replicates in the cytoplasm and does not have access to
the cellular RNA polymerase which is located in the nucleus.

Reference: Zinsser p. 805

57. Smallpox was eradicated by the global use of vaccine which contains:
A. killed virus C. preformed antibodies
B. attenuated live virus D. preformed antibodies and live attenuated virus

Ans. B
Smallpox disease was eradicated by use of a live attenuated vaccine. Live vaccine are used because
duration of immunity is longer with greater effectiveness of protection, both IgA and IgG are elicited
when administered by the natural route of infection which is intradermally where the virus replicates.
Cell mediated immunity is produced.

Reference: Zinsser p. 949


58. The most important laboratory test for the detection of early Hepatitis B virus infection is:
A. immunoassay for HBsAg C. immunoassay for Anti HBc
B. immunoassay for HBeAg D. immunoassay for Anti HBs

Ans. A
11
HBsAg appears during the incubation period and is detectable in most patients during the prodrome
and acute phase; falls to undetectable levels during convalescence. HBeAg arises during the incubation
period and is present during the prodome and early acute disease. Its presence is an important indicator
of transmissibility. Anti HBc is detectable during prodrome and acute disease. Anti HBs is detectable
during late convalescence.

Reference: Zinsser p. 1043

59. A common upper respiratory tract disease produced by adenovirus is:


A. Rhinitis C. pharyngoconjunctivitis
B. atypical pneumonia D. parotitis

Ans. C
Adenoviruses cause a variety of upper and lower respiratory tract disease. Pharyngoconjunctivitis is
common.

Reference: Zinsser p. 970

60. Skin and plantar warts are caused primarily by:


A. HPV-1 C. HPV-16
B. HPV-6 D. HPV-18

Ans. A
HPV-1 through HPV-4 causes skin and plantar warts. HPV-6 and HPV-11 causes genital warts. HPV- 16
and HPV-18 are associated by carcinoma of the cervix.

Reference: Zinsser p. 772

61. It's most important pathogenic role is its interference with Vitamin B12 absorption:
A. Echinococcus granulosus C. Diphyllobothrium latum
B. Taenia saginata D. Taenia solium

Ans. C
Infection by D. latum causes little damage in the small intestines. In some individuals megaloblastic
anemia occurs as a result of Vit B12 deficiency caused by preferential uptake of the vitamin by the
worm. Echinococosis forms one large fluid filled cyst that may rupture and cause anaphylaxis. Little
damage results from the presence of the adult T. saginata.

Reference: Brown 6th ed. p. 190

62. The rash in measles is due to:


A. virus infecting the epidermis C. cell mediated response
B. vascular permeability D. cytotoxic response

Ans. D
The rash is primarily by cytotoxic T-cells attacking the measles virus-infected vascular endothelial cells.

Reference: Zinsser p. 1011

63. In a pregnant woman exposed to rubella virus, the presence of IgM antibody indicates:
A. Immunity C. recent infection
B. protection of the fetus D. present infection

Ans. C
12
In a pregnant woman, the presence of IgM antibody in a single acute-phase serum sample indicate recent
infection. Presence of IgG antibody with a 1:8 or greater increase in titer indicates immunity.

Reference: Zinsser p. 1012

64. Retroviruses characteristically have a reverse transcriptase within the virion, this is responsible for the:
A. replication of the viral RNA in cytoplasm
B. integration of the DNA provirus into host cellular DNA
C. Dissemination of the single-stranded positive RNA
D. Attachment of the virus to CD4

Ans. B
This unique enzyme produces DNA from RNA, a reverse of the normal replication process and integrates
the provirus DNA into the host cell genome.

Reference: Zinsser p. 779

65. The virus that is increasingly being recognized as cause of nonbacterial infant diarrhea
A. Retrovirus C. Orbivirus
B. Rotavirus D. Calicivirus

Ans. B
Rotavirus gastroenteritis is one of the most common childhood illness and a leading cause of infant
deaths in developing countries.

Reference: Zinsser p. 993

66. Norwalk gastroenteritis virus of humans is what virus?


A. Enterovirus C. Picornavirus
B. Hepatitis virus D. Calicivirus

Ans. D
Norwalk virus is a Calcivirus that is associated with diarrhea. Enteroviruses, hepatitis virus do not
cause diarrhea.

Reference: Zinsser p. 774

67. In the laboratory this protein from Influenza virus agglutinates red blood cells whereas in vivo, it binds to
the surface receptor of the host cell to initiate infection.
A. Hemagglutinin C. Hemolysin
B. Neuramidase D. capsid antigen

Ans. A
The Influenza virus has an envelope covered with 2 different spikes Hemagglutinatinin and
neuramidase. Hemagglutination initiates infection in the host and in the lab causes agglutination of
RBC's. Neuramidase cleaves neuramic acid (sialic acid) to release progeny virus from the infected cell.
Hemolysin is from bacterial cells and causes hemolysis of RBC.

Reference: Zinsser p. 778

68. In the prevention of poliomyeletis, the use of oral vaccine is preferred because:
A. induces intestinal IgA C. can produce lifelong immunity
B. induces humoral IgG D. can produce immunity to unimmunized adults

Ans. A
13
The oral vaccine or Sabin vaccine which is a live attenuated vaccine is preferred over the Salk vaccine or
killed vaccine because (1) it interrupts fecal-oral transmission by inducing secretory IgA in the GIT;
killed vaccine does not. Both kinds of vaccine induce humoral IgG. Infection provides lifelong type
specific immunity.

Reference: Zinsser p. 981

69. Hand-foot-and-mouth disease characterized by versicular rash on the hands and feet and ulcerations in the
mouth, mainly in children is caused by:
A. Echovirus C. Coxsackie virus Group A
B. Parainfluenza virus D. Coxsackie virus Group B

Ans. C
Echovirus causes aseptic meningitis. Parainfluenza causes croup. Coxsackie B causes pleurodynia.
Coxsackie A causes herpangina and hand-foot-and-mouth disease.

Reference: Zinsser p. 774

70. All Hepatitis viruses carry genes for the encoding of its protein coat EXCEPT:
A. Hepatitis A C. Hepatitis C
B. Hepatitis B D. Hepatitis D

Ans. D
Hepatitis D is a defective virus because it does not have the genes for its protein coat. It can only
replicate in cells also infected with HBV because HDV uses the surface antigen of HBV (HBsAg) as its
coat.

Reference: Zinsser p. 1043

71. The following are Arboviruses:


A. Togaviridae C. Bunyaviridae
B. Reoviridae D. Filoviridae

Ans. D
Arboviruses include Togaviridae, Flaviviridae, Bunyaviridae and Reoviridae - Filoviridae include
Marburg virus and Ebola virus which have monkeys as their natural reservoir.

Reference: Zinsser p. 1019

72. If on Sabouraud agar, the culture of a suspected dermatophytosis infection showed a lot of microconidia,
coiled hyphae, the probable genera of the etiologic agent is:
A. Microsporum C. Epidermophyton
B. Trichophyton D. Pityrosporum

Ans. B
Microsporum has lot of Microconidia. Epidermophyton is macroconidia in group. Pityrosporum is a
superficial infection not a dermatophyte

Reference: Zinsser p. 1127

73. The most common fungal opportunistic infection is caused by:


A. Aspergillus fumigatus C. Candida albicans
B. Cryptococcus neoformans D. Mucor

Ans. C
14
Candida albicans - it is endogenous, a lot of risk factors like pregnancy, diabetic person, prolonged used of
antibiotic, used of birth control pills

Reference: Zinsser p. 1139

74. Cryptococcosis can be diagnosed in the lab by using the following test:
A. latex agglutination test C. gram staining
B. India ink D. both A & B

Ans. D
Crypotococcosis antibodies are develop can be detected using latex agglutination test. India ink -
demonstration of the yeast with a big capsule. Gram staining - you can't do gram staining with C.
neoformans

Reference: Zinsser p. 1147

75. The most common mode of infection of systemic fungi is thru:


A. ingestion C. inhalation
B. traumatic implantation D. all of the above

Ans. C
Inhalation of infectious particles (spores) which are suspended in the air.

Reference: Zinsser p. 1091

76. A 45 year old post kidney transplant patient develop fever and pneumonia with marked neutropenia.
Bronchial washing show dichotomously branching filamentous fungi. The fungi involved is:
A. sprorothrix C. candida
B. cryptococcus D. aspergillus

Ans. D
In immunocompromised patient, inhalation of spores of Aspergillus can lead to pneumonia and are
seen as dichotomous branching hyphae (filamentous).

Reference: Zinsser p. 1127

77. In a 7 year old malnourished boy with multiple nematode infections, the most severe complication will
come from:
A. Ascaris lumbricoides C. Necator americanus
B. Trichuris trichiura D. Enterobius vermicularis

Ans. A
Ascaris has a tendency to migrate to other openings causing more severe disease.

Reference: Brown 6th ed. p. 141-142, Roberts and Janovy p. 422

78. Hyperinfective syndrome occurs in:


A. Ancylostoma caninum C. Toxocara cati
B. Strongyloides stercoralis D. Balantidium coli

Ans. B
This condition occurs in immunocompromised persons wherein the filariform larvae become spread
systemically. A. caninum causes cutaneous larva migrans, T. cati, visceral larva migrans while B. coli,
dysentery like diarrhea.

Reference: Brown 6th ed. p. 126


79. The description of an adult of Trichuris trichiura is likened to a/an:
A. noodle C. earthworm
B. thread D. whip

Ans. D
15
T. trichiura has a whiplike anterior and a buggy posterior. The earthworm likeness is more for A.
lumbricoides.

Reference: Brown 6th ed. p. 120

80. Loeffler’s syndrome can be seen in the following parasitic infections EXCEPT:
A. Ascariasis C. Necatoriasis
B. Strongyloidiasis D. Trichuriasis

Ans. D
Loeffler’s syndrome is eosinophilic lung. As the parasitic larvae (rhaditiform for Ascaris, filariform for
Strongyloides and Necator) travels to the pulmonary-circulatory system, the body reacts with an
anaphylactic type of hypersensitivity that produces the eosinophilic lung condition. Trichuris does not
have a pulmonary circulatory pathway.

Reference: Brown 6th ed. p. 141, 126, 130


Roberts & Janovy p. 386, 422

81. All cestodes require an intermediate host EXCEPT:


A. Diphyllobothrium latum C. Hymenolepis nana
B. Taenia saginata D. Echinococcus granulosus

Ans. C
Man gets H. nana infection by ingesting the eggs directly. All other cestodes require an intermediate
host

Reference: Brown 6th ed. p. 192

82. Cysticercosis due to Taenia solium is acquired through ingestion of:


A. cysticercus larva C. procercoid larva
B. cysticercoid larva D. egg

Ans. D
A more dangerous sequence can occurs when a person ingests the egg of Taenia solium in contaminated
food or water rather than eating raw or undercooked pork containing the larva cystercus. Cysticercoid
larva pertains to D. caninum and H. nana. Procercoid larva is for D. latum. The ingested egg hatch in
the small intestines and the oncopheres burrow through the wall into a blood vessel, where they can
disseminate to many organs. This condition is called cysticercosis.

Reference: Brown 6th ed. p. 207

83. The infective stage to man of Necator americanus is:


A. rhabditiform larva C. embryonated egg
B. filariform larvae D. Microfilaria

Ans. B
Filariform larvae of hookworms penetrate the skin

Reference: Roberts & Janovy p. 410

84. This cestode possess a quadrate scolex with 4 circular suckers and no rostellum or hooklets:
A. Diphyllobothrium latum C. Taenia saginata
B. Hymenolepis nana D. Echinococcus granulosus

Ans. C
16
D. latum has an elongated scolex. H. nana has a globular scolex with 4 suckers, a short retractile
rostellum armed with a single row of hooklets. E. granulosus has 4 suckers with a rostellum and a
double row of hooklets.

Reference: Brown 6th p. 194, Roberts & Janovy p. 333

85. The laboratory diagnosis of the disease produced by this helminth is based on finding eggs in the sputum?
A. Fasciola hepatica C. Clonorchis sinensis
B. Echinostoma ilocanum D. Paragonimus westermani

Ans. D
F. hepatica, E. ilocanum, C. sinensis diagnosis is based on finding eggs in feces. P. westermani adults
live in the lungs so diagnosis can be based on finding eggs in sputum and likewise in the stool.

Reference: Brown 6th ed. p. 243, Roberts & Janovy p. 272

86. All trematodes produce disease in man by ingestion of infected intermediate host EXCEPT:
A. Fasciolopsis buski C. Schistosoma japonicum
B. Opisthorchis felineus D. Heterophyes heterophyes

Ans. C
Schistosoma spp. Produce disease by penetration of the skin by the cercaria in snail-infested water.

Reference: Brown 6th ed. p. 247, Roberts & Janovy 242

87. The single most important epidemiological factor in schistosomiasis is:


A. human population eating local fresh water vegetation
B. availability of suitable species snail host
C. human waste in water containing the intermediate host
D. presence of mammalian host

Ans. C
Hygienic waste disposal is sufficient to eliminate schistosomiasis as a disease of humans

Reference: Roberts & Janovy p. 243

88. Amoebic abscess of the liver is mostly found in the


A. left anteromedial lobe C. inferior lobe
B. right posterosuperior lobe D. middle lobe

Ans. B
Spread of amoeba from the large intestines is by the portal vein and deposit on the right posterosuperior
lobe

Reference: Chatterjee p. 25

89. A condition in the newborn presenting with convulsions, chorioretinitis, microcephaly with a
radiographic finding of the skull showing brain calcification is indicative of congenital:
A. syphilis C. toxoplasmosis
B. rubella D. cytomegalovirus infection

Ans. C
This is the condition seen in the newborn especially when exposed during the 3rd trimester of gestation.

Reference: Craig & Faust, p 233

90. A previously healthy young man who had been swimming in a lake about a week ago develops fever and
signs of meningoencephalitis. What is the most probable cause?
A. Acanthamoeba C. Mycobacterium
B. Naegleria D. Hartmanella

17
Ans. B
Naegleria meningoencephalitis affects healthy young persons usually manifesting in the acute phase
while Acanthamoeba or Hartmanella affects the immunocompromised. TB meningitis is spread usually
from a respiratory focus.

Reference: Brown 6th ed. p. 35; Roberts & Janovy p. 11

91. Xenodiagnosis is a technique that:


A. quatifies antigenic levels in the host
B. utilizes insect vectors for diagnosis
C. correlates clinical manifestation of the disease
D. subjects a clinical specimen to a series of procedures

Ans. B
Insect vectors are used to bite a person suspected with the particular parasite. The parasites, if present
will develop further in these insects.

Reference: Brown 6th ed. p. 69, Roberts & Janovy p. 68

92. This type of cell increases in number in intestinal nematodes.


A. eosinophil C. monocyte
B. macrophage D. mast cell

Ans. A
The host responds to intestinal nematodes with a hypersensitivity type I reaction (IgE mediated) that
elicits the increase in eosinophils.

Reference: Brown p. 5, Stanley p. 46, Roitt, Brostoff & Male p. 16.8

93. When freshly passed in stools, Ascaris lumbricoides fertilized eggs are described as:
A. Planoconvex with thin shelled with embryo
B. peanut shaped, with flat bipolar plugs and an uncleaved embryo
C. round, thick shelled , bile stained with uncleaved embryo
D. barrel-shaped with protruding bipolar mucous plugs and an unsegmented embryo

Ans. C
Ascaris lumbricoides has a thick shell due to three shell layers. A, B, & D are the description for the eggs
of E. vermicularis, C. philippinensis and T. trichiura

Reference: Roberts and Janovy, p. 419

94. The earliest antibody response to infection is:


A. IgA C. IgM
B. IgG D. IgE

Ans. C
In the primary immune response, the IgM is the major antibody which is also the first to respond in an
infection. This antibody is a pentamer (with 5 Ig structures) which make it a very good immunoglobulin in
binding complement. In prenatal infections, this antibody increases in number.

Reference: Stanley p. 29

95. Little Joey was frequently sick of various infectious diseases. He was found to have low CD4+
lymphocytes titers. Which of the following immune response processes would still be functional in Joey’s
body?
A. T cell help for B lymphocytes
B. Recognition of peptide-MHC complex
18
C. Phagocytic function for foreign molecules
D. T-dependent antigen recognition

Ans. C
This is an innate natural cellular immune ability of the host to respond to foreign molecules which is
independent of the Th series.

Reference: Stanley p. 92; 124

96. The host immune response in viral infections is mostly by:


A. Th cells C. Tc cells
B. B cells D. NK cells

Ans. C
Viral infections produce endogenous peptides which are presented on the surface of cells in conjunction
with MHC Class 2. These are recognize primarily by Tc cells. B cells are mostly for exogenous peptides
that are antibody mediated mainly stimulated by Th cells while the NK cells’ major activity is to directly
destroy aberrant cells such as in cancer.

Reference: Roitt, Brostoff & Male p. 15.3-15.4

97. A killed virus is the major component of the vaccine against this pathogenic organism.
A. measles C. mumps
B. rubella D. rabies

Ans. D
Measles, rubella, mumps vaccines contain living but attenuated virus (also for chickenpox, Sabin polio)
Rabies vaccine (also for Hep A, Salk polio, influenza) contain killed or inactivated viruses. Hep B
vaccine contains antigenic fragments of the virus usually using recombinant technology (yeast-derived).
Subunit vaccines are also used as in pertussis. Tetanus and diphtheria utilize toxoids which are
inactivated toxins or antitoxins, antibodies against these. H. influenzae type b vaccine is conjugated to a
protein which can be a diphtheria toxoid.

Reference: Tortora p. 502

98. Endotoxins can be differentiated from exotoxins by the following except:


A. Bacterial origin C. chemical composition
B. clinical manifestation D. location in microbial cell

Ans. A
Toxins are mostly produced by bacteria: endotoxins generally from Gram negatives while exotoxins are
from Gram positive organisms; and from some algae, diatoms, dinoflagellates, fungi . Bacterial toxins
can be differentiated by B,C and D. Endotoxins from whatever gram negative bacterial source will
induce similar clinical manifestations which is usually systemic while exotoxin are specific (e.i.
neurotoxin of C. tetani, botox from C. botilinum, diphtheria toxin, enterotoxins, etc.). Exotoxins are
produced inside the cell and released by viable bacteria while endotoxins are in the outer membrane
surrounding the peptidoglycan of the cell wall of gram negative organisms and released when the cells
disintegrate. Exotoxins are usually proteins while endotoxins are lipids (Lipid A in LPS)

Reference: Tortora p. 442-443

99. Which of the following statements is true of direct antigen recognition in innate immunity?
A. NK cells interact with virally infected cells.
B. Phagocytes bind to C3b attached to antigens.
C. NK cells bind to receptors attached to opsonins on antigens.
D. NK cells bind to antibody attached to target cells.
19
Ans. A
For direct antigen recognition to occur, there should be no receptor in between the reacting cells which is
in A. All the other choices have intervening receptors, C3b, opsonins and antibody.

Reference: Stanley p. 17

100. The reservoir host of Capillaria philippinensis that are responsible for the spread of endemicity of the
disease are birds that eat
A. freshwater fishes C. worms
B. rodents D. monkeys

Ans. A
C. philippinesis cystic stages are found in freshwater fishes, hence, migratory birds which are fish eater
spread the endemicity of the parasite

Reference: Brown 6th ed. P. 149 Roberts & Janovy p. 388

Stanley J (2002) Essentials of Immunology & Serology NY: Delmar Thomson Learning
Roberts, L. & Janovy, J (2000) Foundations of Parasitology 6th edition Boston: Mc Graw Hill
Brown 6th edition
Roitt, I, Brostoff, J & Male, D (1993) Immunology 3rd ed London England: Mosby-Year Book Europe
Limited
Chatterjee, K.D. (1980) Parasitology; Med Protozoology & Helminthology Calcutta, India; Chatterjee
Medical Publishers

20

You might also like